แƒžแƒ แƒแƒขแƒแƒšแƒ˜แƒ–แƒฃแƒ แƒ˜ แƒ‘แƒแƒšแƒแƒœแƒกแƒ˜. แƒ›แƒŸแƒแƒ•แƒ”แƒ‘แƒ˜แƒกแƒ แƒ“แƒ แƒคแƒฃแƒซแƒ”แƒ”แƒ‘แƒ˜แƒก แƒžแƒ แƒแƒขแƒแƒšแƒ˜แƒ–แƒฃแƒ แƒ˜ แƒ—แƒ”แƒแƒ แƒ˜แƒ

แƒ–แƒแƒ’แƒแƒ“แƒแƒ“, แƒ‘แƒ แƒแƒœแƒกแƒขแƒ”แƒ“-แƒšแƒแƒฃแƒ แƒ˜แƒก แƒžแƒ แƒแƒขแƒแƒšแƒ˜แƒ–แƒฃแƒ แƒ˜ แƒ—แƒ”แƒแƒ แƒ˜แƒ˜แƒก แƒจแƒ”แƒกแƒแƒ‘แƒแƒ›แƒ˜แƒกแƒแƒ“, แƒ’แƒแƒœแƒขแƒแƒšแƒ”แƒ‘แƒ˜แƒก (4.2) แƒ›แƒ˜แƒฎแƒ”แƒ“แƒ•แƒ˜แƒ—, แƒกแƒฃแƒกแƒขแƒ˜ แƒ›แƒแƒœแƒแƒ‘แƒแƒ–แƒฃแƒ แƒ˜ แƒ›แƒŸแƒแƒ•แƒ˜แƒก แƒ“แƒ˜แƒกแƒแƒชแƒ˜แƒแƒชแƒ˜แƒ˜แƒกแƒ—แƒ•แƒ˜แƒก แƒ’แƒ•แƒแƒฅแƒ•แƒก:

แƒญแƒ”แƒจแƒ›แƒแƒ แƒ˜แƒขแƒ˜ แƒ—แƒ”แƒ แƒ›แƒแƒ“แƒ˜แƒœแƒแƒ›แƒ˜แƒ™แƒฃแƒ แƒ˜ แƒ›แƒฃแƒ“แƒ›แƒ˜แƒ•แƒ˜ TOแƒ”แƒก แƒ‘แƒแƒšแƒแƒœแƒกแƒ˜ แƒ˜แƒฅแƒœแƒ”แƒ‘แƒ

แƒกแƒแƒ“แƒแƒช แƒงแƒ•แƒ”แƒšแƒ แƒแƒฅแƒขแƒ˜แƒ•แƒแƒ‘แƒ แƒแƒ แƒ˜แƒก แƒฌแƒแƒœแƒแƒกแƒฌแƒแƒ แƒฃแƒšแƒ˜. แƒฌแƒแƒ แƒ›แƒแƒ•แƒ˜แƒ“แƒ’แƒ˜แƒœแƒแƒ— แƒ”แƒก แƒ—แƒแƒœแƒแƒคแƒแƒ แƒ“แƒแƒ‘แƒ แƒคแƒแƒ แƒ›แƒ˜แƒ—:

แƒแƒฆแƒœแƒ˜แƒจแƒœแƒ”แƒ—, แƒ แƒแƒ’แƒแƒ แƒช แƒฌแƒ˜แƒœแƒ แƒจแƒ”แƒ›แƒ—แƒฎแƒ•แƒ”แƒ•แƒแƒจแƒ˜, แƒแƒ แƒ˜ แƒ›แƒฃแƒ“แƒ›แƒ˜แƒ•แƒ˜แƒก แƒœแƒแƒ›แƒ แƒแƒ•แƒšแƒ˜ TOแƒ“แƒ a (H 2 O) แƒ›แƒ”แƒจแƒ•แƒ”แƒแƒ‘แƒ˜แƒ— (H 2 O) \u003d consst at T=แƒ™แƒแƒœแƒกแƒข. แƒ›แƒ”แƒ แƒ”

แƒแƒœ แƒ“แƒแƒแƒฎแƒšแƒแƒ”แƒ‘แƒ˜แƒ—:

แƒกแƒแƒ“แƒแƒช แƒงแƒ•แƒ”แƒšแƒ แƒ™แƒแƒœแƒชแƒ”แƒœแƒขแƒ แƒแƒชแƒ˜แƒ แƒฌแƒแƒœแƒแƒกแƒฌแƒแƒ แƒฃแƒšแƒ˜แƒ. แƒแƒฅ แƒแƒ แƒ˜แƒก แƒฆแƒ˜แƒ แƒ”แƒ‘แƒฃแƒšแƒ”แƒ‘แƒ TO แƒแƒ“แƒแƒฃแƒ แƒ”แƒ™แƒ แƒ›แƒŸแƒแƒ•แƒ แƒ“แƒ˜แƒกแƒแƒชแƒ˜แƒแƒชแƒ˜แƒ˜แƒก (แƒ˜แƒแƒœแƒ˜แƒ–แƒแƒชแƒ˜แƒ˜แƒก) แƒ›แƒฃแƒ“แƒ›แƒ˜แƒ•แƒ˜ แƒแƒœ แƒฃแƒ‘แƒ แƒแƒšแƒแƒ“ แƒ›แƒŸแƒแƒ•แƒ˜แƒแƒœแƒแƒ‘แƒ˜แƒก แƒ›แƒฃแƒ“แƒ›แƒ˜แƒ•แƒ˜.

แƒ‘แƒ”แƒ•แƒ แƒ˜ แƒกแƒฃแƒกแƒขแƒ˜ แƒ›แƒŸแƒแƒ•แƒ˜แƒกแƒ—แƒ•แƒ˜แƒก, แƒ แƒ˜แƒชแƒฎแƒ•แƒ˜แƒ—แƒ˜ แƒ›แƒœแƒ˜แƒจแƒ•แƒœแƒ”แƒšแƒแƒ‘แƒ”แƒ‘แƒ˜ TO แƒแƒซแƒแƒšแƒ˜แƒแƒœ แƒ›แƒชแƒ˜แƒ แƒ”แƒ, แƒแƒ›แƒ˜แƒขแƒแƒ› แƒœแƒแƒชแƒ•แƒšแƒแƒ“ TO แƒแƒ•แƒ แƒชแƒ”แƒšแƒ“แƒ”แƒ‘แƒ แƒกแƒ˜แƒซแƒšแƒ˜แƒ”แƒ แƒ˜แƒก แƒ›แƒแƒฉแƒ•แƒ”แƒœแƒ”แƒ‘แƒ”แƒšแƒ˜ (แƒแƒœ แƒฃแƒ‘แƒ แƒแƒšแƒแƒ“ แƒ›แƒแƒฉแƒ•แƒ”แƒœแƒ”แƒ‘แƒ”แƒšแƒ˜):

RK แƒ =- แƒšแƒ’ TO แƒ .

แฒฃแƒคแƒ แƒ TO แƒ(แƒแƒœแƒฃ แƒ แƒแƒช แƒฃแƒคแƒ แƒ แƒžแƒแƒขแƒแƒ แƒแƒ แƒž TO แƒ ), แƒ แƒแƒช แƒฃแƒคแƒ แƒ แƒซแƒšแƒ˜แƒ”แƒ แƒ˜แƒ แƒ›แƒŸแƒแƒ•แƒ.

แƒ›แƒแƒœแƒแƒ‘แƒแƒ–แƒฃแƒ แƒ˜ แƒ›แƒŸแƒแƒ•แƒแƒก HB-แƒ˜แƒก แƒกแƒแƒฌแƒงแƒ˜แƒกแƒ˜ แƒ™แƒแƒœแƒชแƒ”แƒœแƒขแƒ แƒแƒชแƒ˜แƒ แƒฃแƒ“แƒ แƒ˜แƒก แƒฎแƒกแƒœแƒแƒ แƒจแƒ˜ แƒ›แƒ˜แƒกแƒ˜ แƒ“แƒ˜แƒกแƒแƒชแƒ˜แƒแƒชแƒ˜แƒ˜แƒก (แƒ˜แƒแƒœแƒ˜แƒ–แƒแƒชแƒ˜แƒ˜แƒก) แƒฎแƒแƒ แƒ˜แƒกแƒฎแƒก. แƒ›แƒแƒจแƒ˜แƒœ [ะ 3 ะž + ] แƒ“แƒ [ะ’ - ] แƒ˜แƒแƒœแƒ”แƒ‘แƒ˜แƒก แƒฌแƒแƒœแƒแƒกแƒฌแƒแƒ แƒฃแƒšแƒ˜ แƒ™แƒแƒœแƒชแƒ”แƒœแƒขแƒ แƒแƒชแƒ˜แƒ”แƒ‘แƒ˜ แƒขแƒแƒšแƒ˜ แƒ˜แƒฅแƒœแƒ”แƒ‘แƒ [ะ 3 ะž + ] = [ะ’ - ] = แƒแƒก แƒ , แƒฌแƒแƒœแƒแƒกแƒฌแƒแƒ แƒฃแƒšแƒ˜ แƒ›แƒŸแƒแƒ•แƒ˜แƒก แƒ™แƒแƒœแƒชแƒ”แƒœแƒขแƒ แƒแƒชแƒ˜แƒ [HB] = แƒ—แƒแƒœ แƒ - ฮฑ แƒ—แƒแƒœ แƒ = แƒ—แƒแƒœ แƒ(1 - ฮฑ). แƒฌแƒแƒœแƒแƒกแƒฌแƒแƒ แƒฃแƒšแƒ˜ แƒ™แƒแƒœแƒชแƒ”แƒœแƒขแƒ แƒแƒชแƒ˜แƒ”แƒ‘แƒ˜แƒก แƒแƒ› แƒ›แƒœแƒ˜แƒจแƒ•แƒœแƒ”แƒšแƒแƒ‘แƒ”แƒ‘แƒ˜แƒก แƒฉแƒแƒœแƒแƒชแƒ•แƒšแƒ”แƒ‘แƒ˜แƒ— แƒฌแƒแƒœแƒแƒกแƒฌแƒแƒ แƒแƒ‘แƒ˜แƒก แƒ›แƒฃแƒ“แƒ›แƒ˜แƒ•แƒ˜แƒก แƒ’แƒแƒ›แƒแƒฎแƒแƒขแƒฃแƒšแƒ”แƒ‘แƒแƒจแƒ˜ (4.10), แƒ›แƒ˜แƒ•แƒ˜แƒฆแƒ”แƒ‘แƒ—:

แƒ—แƒฃ แƒ™แƒแƒœแƒชแƒ”แƒœแƒขแƒ แƒแƒชแƒ˜แƒ˜แƒก แƒœแƒแƒชแƒ•แƒšแƒแƒ“ แƒ—แƒแƒœ แƒแƒ’แƒแƒ›แƒแƒ˜แƒงแƒ”แƒœแƒ”แƒ— แƒ›แƒ˜แƒกแƒ˜ แƒแƒ แƒ›แƒฎแƒ แƒ˜แƒ•แƒ˜ แƒ•- แƒ’แƒแƒœแƒ–แƒแƒ•แƒ”แƒ‘แƒ (แƒ’แƒแƒœแƒ–แƒแƒ•แƒ”แƒ‘แƒ), แƒ’แƒแƒ›แƒแƒฎแƒแƒขแƒฃแƒšแƒ˜ แƒš/แƒ›แƒแƒšแƒจแƒ˜, V=1/แƒ—แƒแƒœ แƒ , แƒจแƒ”แƒ›แƒ“แƒ”แƒ’ แƒคแƒแƒ แƒ›แƒฃแƒšแƒ TO แƒแƒ’แƒแƒ›แƒแƒ˜แƒงแƒฃแƒ แƒ”แƒ‘แƒ แƒแƒกแƒ”:

แƒ”แƒก แƒ—แƒแƒœแƒแƒคแƒแƒ แƒ“แƒแƒ‘แƒ, แƒ˜แƒกแƒ”แƒ•แƒ” แƒ แƒแƒ’แƒแƒ แƒช แƒ’แƒแƒ›แƒแƒฎแƒแƒขแƒฃแƒšแƒ”แƒ‘แƒ

แƒแƒฆแƒฌแƒ”แƒ แƒ” แƒแƒกแƒขแƒ•แƒแƒšแƒ“แƒ˜แƒก แƒ’แƒแƒœแƒ–แƒแƒ•แƒ”แƒ‘แƒ˜แƒก แƒ™แƒแƒœแƒแƒœแƒ˜ (แƒแƒœ แƒ’แƒแƒœแƒ–แƒแƒ•แƒ”แƒ‘แƒ˜แƒก แƒ™แƒแƒœแƒแƒœแƒ˜).แƒกแƒฃแƒกแƒขแƒ˜ แƒแƒ แƒแƒ‘แƒ˜แƒ—แƒ˜ แƒ”แƒšแƒ”แƒฅแƒขแƒ แƒแƒšแƒ˜แƒขแƒ˜แƒกแƒ—แƒ•แƒ˜แƒก. a1-แƒกแƒ—แƒ•แƒ˜แƒก (แƒขแƒ˜แƒžแƒ˜แƒฃแƒ แƒ˜ แƒจแƒ”แƒ›แƒ—แƒฎแƒ•แƒ”แƒ•แƒ แƒ‘แƒ”แƒ•แƒ  แƒแƒœแƒแƒšแƒ˜แƒขแƒ˜แƒ™แƒฃแƒ  แƒกแƒ˜แƒกแƒขแƒ”แƒ›แƒแƒจแƒ˜)

แƒแƒ“แƒ•แƒ˜แƒšแƒ˜แƒ แƒ˜แƒ›แƒ˜แƒก แƒฉแƒ•แƒ”แƒœแƒ”แƒ‘แƒ, แƒ แƒแƒ› แƒ–แƒแƒ’แƒแƒ“แƒแƒ“, แƒœแƒ”แƒ‘แƒ˜แƒกแƒ›แƒ˜แƒ”แƒ แƒ˜ แƒจแƒ”แƒ›แƒแƒ“แƒ’แƒ”แƒœแƒšแƒแƒ‘แƒ˜แƒก K n A m แƒกแƒฃแƒกแƒขแƒ˜ แƒ”แƒšแƒ”แƒฅแƒขแƒ แƒแƒšแƒ˜แƒขแƒ˜แƒกแƒ—แƒ•แƒ˜แƒก, แƒ แƒแƒ›แƒ”แƒšแƒ˜แƒช แƒ˜แƒจแƒšแƒ”แƒ‘แƒ แƒ˜แƒแƒœแƒ”แƒ‘แƒแƒ“ แƒกแƒฅแƒ”แƒ›แƒ˜แƒก แƒ›แƒ˜แƒฎแƒ”แƒ“แƒ•แƒ˜แƒ—.

K n A m = แƒž TO t+ +แƒขแƒ แƒœ -

แƒแƒกแƒขแƒ•แƒแƒšแƒ“แƒ˜แƒก แƒ’แƒแƒœแƒ–แƒแƒ•แƒ”แƒ‘แƒ˜แƒก แƒ™แƒแƒœแƒแƒœแƒ˜ แƒแƒฆแƒฌแƒ”แƒ แƒ˜แƒšแƒ˜แƒ แƒ›แƒ˜แƒ›แƒแƒ แƒ—แƒ”แƒ‘แƒ˜แƒ—

แƒกแƒแƒ“ แƒ—แƒแƒœ- แƒกแƒฃแƒกแƒขแƒ˜ แƒ”แƒšแƒ”แƒฅแƒขแƒ แƒแƒšแƒ˜แƒขแƒ˜แƒก แƒกแƒแƒฌแƒงแƒ˜แƒกแƒ˜ แƒ™แƒแƒœแƒชแƒ”แƒœแƒขแƒ แƒแƒชแƒ˜แƒ, แƒ›แƒแƒ’แƒแƒšแƒ˜แƒ—แƒแƒ“, แƒกแƒฃแƒกแƒขแƒ˜ แƒ›แƒŸแƒแƒ•แƒ. แƒแƒกแƒ” แƒ แƒแƒ›, แƒแƒ แƒ—แƒแƒคแƒแƒกแƒคแƒแƒ แƒ˜แƒก แƒ›แƒŸแƒแƒ•แƒ˜แƒกแƒ—แƒ•แƒ˜แƒก H 3 RO 4 (แƒž = 3,

แƒ—= 1), แƒ แƒแƒ›แƒ”แƒšแƒ˜แƒช แƒ›แƒ—แƒšแƒ˜แƒแƒœแƒแƒ‘แƒแƒจแƒ˜ แƒ˜แƒจแƒšแƒ”แƒ‘แƒ แƒ˜แƒแƒœแƒ”แƒ‘แƒแƒ“ แƒกแƒฅแƒ”แƒ›แƒ˜แƒก แƒ›แƒ˜แƒฎแƒ”แƒ“แƒ•แƒ˜แƒ—

.

แƒแƒ แƒแƒ‘แƒ˜แƒ—แƒ˜ แƒ”แƒšแƒ”แƒฅแƒขแƒ แƒแƒšแƒ˜แƒขแƒ˜แƒกแƒ—แƒ•แƒ˜แƒก, แƒ™แƒแƒ•แƒจแƒ˜แƒ แƒ˜ แƒ’แƒแƒ“แƒแƒ˜แƒฅแƒชแƒ”แƒ•แƒ (4.11). a1-แƒกแƒ—แƒ•แƒ˜แƒก แƒ’แƒ•แƒแƒฅแƒ•แƒก:

แƒ›แƒแƒ“แƒ˜แƒ— แƒ•แƒ˜แƒžแƒแƒ•แƒแƒ— HB แƒ›แƒแƒœแƒแƒ‘แƒแƒ–แƒฃแƒ แƒ˜ แƒ›แƒŸแƒแƒ•แƒ˜แƒก แƒฎแƒกแƒœแƒแƒ แƒ˜แƒก pH-แƒ˜แƒก แƒฌแƒแƒœแƒแƒกแƒฌแƒแƒ แƒฃแƒšแƒ˜ แƒ›แƒœแƒ˜แƒจแƒ•แƒœแƒ”แƒšแƒแƒ‘แƒ. แƒฌแƒงแƒแƒšแƒ‘แƒแƒ“แƒ˜แƒก แƒ˜แƒแƒœแƒ”แƒ‘แƒ˜แƒก แƒฌแƒแƒœแƒแƒกแƒฌแƒแƒ แƒฃแƒšแƒ˜ แƒ™แƒแƒœแƒชแƒ”แƒœแƒขแƒ แƒแƒชแƒ˜แƒ

แƒแƒฆแƒœแƒ˜แƒจแƒ•แƒœแƒ˜แƒก แƒ’แƒแƒ›แƒแƒงแƒ”แƒœแƒ”แƒ‘แƒ˜แƒ— แƒ›แƒ˜แƒ•แƒ˜แƒฆแƒ”แƒ‘แƒ—:

pH = 0.5 (แƒ’แƒ• TO แƒ+แƒ’แƒ• แƒ—แƒแƒœ แƒ). (4.12)

แƒแƒ›แƒ แƒ˜แƒ’แƒแƒ“, แƒกแƒฃแƒกแƒขแƒ˜ แƒ›แƒแƒœแƒแƒ‘แƒแƒ–แƒฃแƒ แƒ˜ แƒ›แƒŸแƒแƒ•แƒ˜แƒก แƒฎแƒกแƒœแƒแƒ แƒ˜แƒก แƒฌแƒแƒœแƒแƒกแƒฌแƒแƒ แƒแƒ‘แƒ˜แƒก pH-แƒ˜แƒก แƒ’แƒแƒ›แƒแƒกแƒแƒ—แƒ•แƒšแƒ”แƒšแƒแƒ“ แƒแƒฃแƒชแƒ˜แƒšแƒ”แƒ‘แƒ”แƒšแƒ˜แƒ แƒ•แƒ˜แƒชแƒแƒ“แƒ”แƒ— แƒแƒ› แƒ›แƒŸแƒแƒ•แƒ˜แƒก แƒ›แƒŸแƒแƒ•แƒ˜แƒแƒœแƒแƒ‘แƒ˜แƒก แƒ›แƒฃแƒ“แƒ›แƒ˜แƒ•แƒ˜ TO แƒแƒ“แƒ แƒ›แƒ˜แƒกแƒ˜ แƒกแƒแƒฌแƒงแƒ˜แƒกแƒ˜ แƒ™แƒแƒœแƒชแƒ”แƒœแƒขแƒ แƒแƒชแƒ˜แƒ แƒ—แƒแƒœ แƒ .

แƒ’แƒแƒ›แƒแƒ—แƒ•แƒแƒšแƒ”แƒ— แƒซแƒ›แƒแƒ แƒ›แƒŸแƒแƒ•แƒแƒก แƒฎแƒกแƒœแƒแƒ แƒ˜แƒก pH แƒกแƒแƒฌแƒงแƒ˜แƒกแƒ˜ แƒ™แƒแƒœแƒชแƒ”แƒœแƒขแƒ แƒแƒชแƒ˜แƒ˜แƒ— 0,01 แƒ›แƒแƒš/แƒš.

แƒแƒ—แƒแƒฎแƒ˜แƒก แƒขแƒ”แƒ›แƒžแƒ”แƒ แƒแƒขแƒฃแƒ แƒแƒ–แƒ” แƒซแƒ›แƒแƒ แƒ›แƒŸแƒแƒ•แƒ˜แƒกแƒ—แƒ•แƒ˜แƒก TO แƒ = 1.74 10 -5 แƒ“แƒ แƒ’แƒ• TO แƒ = 4,76.

แƒคแƒแƒ แƒ›แƒฃแƒšแƒ˜แƒก แƒ›แƒ˜แƒฎแƒ”แƒ“แƒ•แƒ˜แƒ— (4.12) แƒจแƒ”แƒ’แƒ•แƒ˜แƒซแƒšแƒ˜แƒ แƒ“แƒแƒ•แƒฌแƒ”แƒ แƒแƒ—:

pH = 0.5 (แƒ’แƒ• TO แƒ+แƒ’แƒ• แƒ—แƒแƒœ แƒ) = 0,5(476-0,01) = 0,5(4,76+2) = 3,38.

แƒ›แƒกแƒ’แƒแƒ•แƒกแƒ˜ แƒ’แƒแƒœแƒฎแƒ˜แƒšแƒ•แƒ แƒจแƒ”แƒ˜แƒซแƒšแƒ”แƒ‘แƒ แƒ’แƒแƒœแƒฎแƒแƒ แƒชแƒ˜แƒ”แƒšแƒ“แƒ”แƒก แƒฌแƒแƒœแƒแƒกแƒฌแƒแƒ แƒแƒ‘แƒ˜แƒกแƒ—แƒ•แƒ˜แƒก แƒœแƒ”แƒ‘แƒ˜แƒกแƒ›แƒ˜แƒ”แƒ แƒ˜ แƒกแƒฃแƒกแƒขแƒ˜ แƒแƒ›แƒแƒฎแƒกแƒœแƒ˜แƒกแƒแƒก แƒ›แƒ แƒแƒ•แƒแƒšแƒซแƒ˜แƒ แƒ˜แƒแƒœแƒ˜แƒ›แƒŸแƒแƒ•แƒ”แƒ‘แƒ˜.

แƒžแƒแƒšแƒ˜แƒ‘แƒแƒ–แƒฃแƒ แƒ˜ แƒ›แƒŸแƒแƒ•แƒ”แƒ‘แƒ˜ แƒ˜แƒจแƒšแƒ”แƒ‘แƒ แƒ˜แƒแƒœแƒ”แƒ‘แƒแƒ“ แƒ”แƒขแƒแƒžแƒแƒ‘แƒ แƒ˜แƒ•แƒแƒ“, แƒ แƒแƒ›แƒ“แƒ”แƒœแƒ˜แƒ›แƒ” แƒ”แƒขแƒแƒžแƒแƒ“, แƒ แƒแƒ›แƒ”แƒšแƒ—แƒแƒ’แƒแƒœ แƒ—แƒ˜แƒ—แƒแƒ”แƒฃแƒšแƒ˜ แƒฎแƒแƒกแƒ˜แƒแƒ—แƒ“แƒ”แƒ‘แƒ แƒฌแƒแƒœแƒแƒกแƒฌแƒแƒ แƒแƒ‘แƒ˜แƒก แƒกแƒแƒ™แƒฃแƒ—แƒแƒ แƒ˜ แƒ›แƒฃแƒ“แƒ›แƒ˜แƒ•แƒ˜แƒ—. แƒ”แƒขแƒแƒžแƒแƒ‘แƒ แƒ˜แƒ•แƒ˜ แƒ›แƒŸแƒแƒ•แƒ แƒ“แƒ˜แƒกแƒแƒชแƒ˜แƒแƒชแƒ˜แƒ˜แƒก แƒ›แƒฃแƒ“แƒ›แƒ˜แƒ•แƒ˜.แƒแƒกแƒ” แƒ แƒแƒ›, แƒ›แƒแƒ’แƒแƒšแƒ˜แƒ—แƒแƒ“, แƒแƒ แƒ—แƒแƒ‘แƒแƒ แƒ˜แƒก แƒ›แƒŸแƒแƒ•แƒแƒก H 3 BO 3 แƒฎแƒกแƒœแƒแƒ แƒ”แƒ‘แƒจแƒ˜ แƒ˜แƒฅแƒ›แƒœแƒ”แƒ‘แƒ แƒฌแƒแƒœแƒแƒกแƒฌแƒแƒ แƒแƒ‘แƒ (แƒ›แƒฃแƒ“แƒ›แƒ˜แƒ•แƒ”แƒ‘แƒ˜แƒก แƒ›แƒœแƒ˜แƒจแƒ•แƒœแƒ”แƒšแƒแƒ‘แƒ”แƒ‘แƒ˜ แƒ›แƒแƒชแƒ”แƒ›แƒฃแƒšแƒ˜แƒ 25 ยฐ C- แƒ–แƒ”):

H 3 BO 3 + H 2 O \u003d H 3 O + +, TO 1 =

H 2 O \u003d H 3 O + +, TO 2 =

H 2 O \u003d H 3 O + +, TO 3 =

แƒงแƒแƒ•แƒ”แƒšแƒ˜ แƒ›แƒแƒ›แƒ“แƒ”แƒ•แƒœแƒ แƒ”แƒขแƒแƒžแƒ˜แƒก แƒ›แƒŸแƒแƒ•แƒแƒก แƒ“แƒ˜แƒกแƒแƒชแƒ˜แƒแƒชแƒ˜แƒ˜แƒก แƒ›แƒฃแƒ“แƒ›แƒ˜แƒ•แƒ แƒœแƒแƒ™แƒšแƒ”แƒ‘แƒ˜แƒ แƒฌแƒ˜แƒœแƒ แƒกแƒแƒคแƒ”แƒฎแƒฃแƒ แƒ˜แƒก แƒ“แƒ˜แƒกแƒแƒชแƒ˜แƒแƒชแƒ˜แƒ˜แƒก แƒ›แƒฃแƒ“แƒ›แƒ˜แƒ•แƒ–แƒ” - แƒฉแƒ•แƒ”แƒฃแƒšแƒ”แƒ‘แƒ แƒ˜แƒ•, แƒกแƒ˜แƒ“แƒ˜แƒ“แƒ˜แƒก แƒ แƒแƒ›แƒ“แƒ”แƒœแƒ˜แƒ›แƒ” แƒ แƒ˜แƒ’แƒ˜แƒ—.

แƒงแƒ•แƒ”แƒšแƒ แƒ”แƒขแƒแƒžแƒแƒ‘แƒ แƒ˜แƒ•แƒ˜ แƒ“แƒ˜แƒกแƒแƒชแƒ˜แƒแƒชแƒ˜แƒ˜แƒก แƒ›แƒฃแƒ“แƒ›แƒ˜แƒ•แƒ”แƒ‘แƒ˜แƒก แƒœแƒแƒ›แƒ แƒแƒ•แƒšแƒ˜ แƒขแƒแƒšแƒ˜แƒ แƒ›แƒŸแƒแƒ•แƒแƒก แƒ“แƒ˜แƒกแƒแƒชแƒ˜แƒแƒชแƒ˜แƒ˜แƒก แƒ›แƒ—แƒšแƒ˜แƒแƒœแƒ˜ แƒ›แƒฃแƒ“แƒ›แƒ˜แƒ•แƒ˜แƒก K:

TO 1 TO 2 ...TO แƒž =แƒ™.

แƒแƒ›แƒ แƒ˜แƒ’แƒแƒ“, แƒแƒ“แƒ•แƒ˜แƒšแƒ˜แƒ แƒ˜แƒ›แƒ˜แƒก แƒ“แƒแƒœแƒแƒฎแƒ•แƒ, แƒ แƒแƒ› แƒแƒ แƒ—แƒแƒ‘แƒแƒ แƒ˜แƒก แƒ›แƒŸแƒแƒ•แƒ˜แƒกแƒ—แƒ•แƒ˜แƒก แƒ›แƒœแƒ˜แƒจแƒ•แƒœแƒ”แƒšแƒแƒ‘แƒ แƒแƒฅแƒ•แƒก

TO 1 TO 2 TO 3 =K=

แƒแƒ แƒ˜แƒก แƒฏแƒแƒ›แƒฃแƒ แƒ˜ แƒ›แƒŸแƒแƒ•แƒแƒก แƒ“แƒ˜แƒกแƒแƒชแƒ˜แƒแƒชแƒ˜แƒ˜แƒก แƒ›แƒฃแƒ“แƒ›แƒ˜แƒ•แƒ˜ แƒกแƒฅแƒ”แƒ›แƒ˜แƒก แƒ›แƒ˜แƒฎแƒ”แƒ“แƒ•แƒ˜แƒ—:

4.3.2 แƒกแƒฃแƒกแƒขแƒ˜ แƒคแƒฃแƒซแƒ”แƒ”แƒ‘แƒ˜แƒก แƒฎแƒกแƒœแƒแƒ แƒ”แƒ‘แƒ˜แƒก แƒคแƒฃแƒซแƒ˜แƒก แƒ›แƒฃแƒ“แƒ›แƒ˜แƒ•แƒ˜ แƒ“แƒ pH

แƒ‘แƒ แƒแƒœแƒกแƒขแƒ”แƒ“-แƒšแƒแƒฃแƒ แƒ˜แƒก แƒ›แƒŸแƒแƒ•แƒ”แƒ‘แƒ˜แƒกแƒ แƒ“แƒ แƒคแƒฃแƒซแƒ”แƒ”แƒ‘แƒ˜แƒก แƒžแƒ แƒแƒขแƒแƒšแƒ˜แƒ–แƒฃแƒ แƒ˜ แƒ—แƒ”แƒแƒ แƒ˜แƒ˜แƒก แƒจแƒ”แƒกแƒแƒ‘แƒแƒ›แƒ˜แƒกแƒแƒ“, แƒ–แƒแƒ’แƒแƒ“ แƒจแƒ”แƒ›แƒ—แƒฎแƒ•แƒ”แƒ•แƒแƒจแƒ˜, แƒฌแƒงแƒแƒšแƒฎแƒกแƒœแƒแƒ แƒ”แƒ‘แƒจแƒ˜ แƒ”แƒ แƒ—แƒ˜ แƒ›แƒŸแƒแƒ•แƒ แƒกแƒฃแƒกแƒขแƒ˜ แƒคแƒฃแƒซแƒ˜แƒก B แƒ˜แƒแƒœแƒ˜แƒ–แƒแƒชแƒ˜แƒ˜แƒกแƒ—แƒ•แƒ˜แƒก แƒจแƒ”แƒ˜แƒซแƒšแƒ”แƒ‘แƒ แƒ“แƒแƒฌแƒ”แƒ แƒแƒ—:

B + H 2 O \u003d HB + + OH -

แƒ—แƒฃ แƒคแƒฃแƒซแƒ˜แƒก แƒ˜แƒแƒœแƒ˜แƒ–แƒแƒชแƒ˜แƒ˜แƒก แƒฎแƒแƒ แƒ˜แƒกแƒฎแƒ˜ แƒแƒ แƒ˜แƒก a1, แƒ›แƒแƒจแƒ˜แƒœ แƒ™แƒแƒœแƒชแƒ”แƒœแƒขแƒ แƒแƒชแƒ˜แƒ˜แƒก แƒ›แƒฃแƒ“แƒ›แƒ˜แƒ•แƒ˜ แƒจแƒ”แƒ˜แƒซแƒšแƒ”แƒ‘แƒ แƒ›แƒ˜แƒ•แƒ˜แƒฆแƒแƒ— แƒแƒ› แƒฅแƒ˜แƒ›แƒ˜แƒฃแƒ แƒ˜ แƒฌแƒแƒœแƒแƒกแƒฌแƒแƒ แƒแƒ‘แƒ˜แƒก แƒ›แƒฃแƒ“แƒ›แƒ˜แƒ•แƒแƒ“.

แƒฌแƒ˜แƒœแƒแƒก แƒ›แƒกแƒ’แƒแƒ•แƒกแƒแƒ“, แƒ›แƒ˜แƒ•แƒ˜แƒฆแƒ”แƒ‘แƒ—:

TO = =แƒ™ แƒ‘ = const แƒ แƒแƒ“แƒ”แƒกแƒแƒช แƒ—= แƒ™แƒแƒœแƒกแƒข

แƒ แƒแƒ’แƒแƒ แƒช แƒแƒ แƒ˜ แƒ›แƒฃแƒ“แƒ›แƒ˜แƒ•แƒ˜แƒก แƒœแƒแƒ›แƒ แƒแƒ•แƒšแƒ˜ TO\u003d const แƒ“แƒ [H 2 O] \u003d const.

แƒ แƒแƒแƒ“แƒ”แƒœแƒแƒ‘แƒแƒก แƒ•แƒฃแƒฌแƒแƒ“แƒแƒ— แƒ™ แƒ‘ , แƒ—แƒแƒœแƒแƒ‘แƒแƒ แƒ˜, แƒจแƒ”แƒกแƒแƒ‘แƒแƒ›แƒ˜แƒกแƒแƒ“,

แƒ™ แƒ‘ = , (4.13)

แƒกแƒฃแƒกแƒขแƒ˜ แƒ”แƒ แƒ—แƒ›แƒŸแƒแƒ•แƒ แƒคแƒฃแƒซแƒ˜แƒก แƒ“แƒ˜แƒกแƒแƒชแƒ˜แƒแƒชแƒ˜แƒ˜แƒก (แƒ˜แƒแƒœแƒ˜แƒ–แƒแƒชแƒ˜แƒ˜แƒก) แƒ›แƒฃแƒ“แƒ›แƒ˜แƒ•แƒ˜แƒแƒœแƒฃแƒ‘แƒ แƒแƒšแƒแƒ“ แƒกแƒแƒ‘แƒแƒ–แƒ˜แƒกแƒ แƒ›แƒฃแƒ“แƒ›แƒ˜แƒ•แƒ˜แƒ”แƒก แƒ‘แƒแƒ–แƒ แƒ“แƒ แƒฆแƒ˜แƒ แƒ”แƒ‘แƒฃแƒšแƒ”แƒ‘แƒ

แƒ’แƒ• แƒ™ แƒ‘ = แƒ™ แƒ‘ ,

แƒกแƒ˜แƒซแƒšแƒ˜แƒ”แƒ แƒ˜แƒก แƒ›แƒแƒฉแƒ•แƒ”แƒœแƒ”แƒ‘แƒ”แƒšแƒ˜ (แƒแƒœ แƒฃแƒ‘แƒ แƒแƒšแƒแƒ“ แƒ˜แƒœแƒ“แƒ˜แƒ™แƒแƒขแƒแƒ แƒ˜) แƒกแƒแƒ‘แƒแƒ–แƒ˜แƒกแƒ แƒ›แƒฃแƒ“แƒ›แƒ˜แƒ•แƒ˜แƒก.

แƒแƒกแƒขแƒ•แƒแƒšแƒ“แƒ˜แƒก แƒ’แƒแƒœแƒ–แƒแƒ•แƒ”แƒ‘แƒ˜แƒก แƒ™แƒแƒœแƒแƒœแƒ˜แƒก แƒ›แƒ˜แƒฎแƒ”แƒ“แƒ•แƒ˜แƒ— แƒ’แƒแƒœแƒกแƒแƒฎแƒ˜แƒšแƒ•แƒ”แƒš แƒกแƒแƒฅแƒ›แƒ”แƒจแƒ˜ (แƒ›แƒกแƒ’แƒแƒ•แƒกแƒ˜ แƒ›แƒ˜แƒ›แƒแƒ แƒ—แƒ”แƒ‘แƒ˜แƒ— (4.11))

แƒ™ แƒ‘ =,

แƒกแƒแƒ“ แƒแƒ แƒ˜แƒก แƒ”แƒ แƒ—แƒ˜ แƒ›แƒŸแƒแƒ•แƒ แƒกแƒฃแƒกแƒขแƒ˜ แƒคแƒฃแƒซแƒ˜แƒก แƒ˜แƒแƒœแƒ˜แƒ–แƒแƒชแƒ˜แƒ˜แƒก แƒฎแƒแƒ แƒ˜แƒกแƒฎแƒ˜ แƒ“แƒ แƒแƒ แƒ˜แƒก แƒ›แƒ˜แƒกแƒ˜ แƒกแƒแƒฌแƒงแƒ˜แƒกแƒ˜ แƒ™แƒแƒœแƒชแƒ”แƒœแƒขแƒ แƒแƒชแƒ˜แƒ. แƒ•แƒ˜แƒœแƒแƒ˜แƒ“แƒแƒœ แƒกแƒฃแƒกแƒขแƒ˜ แƒคแƒฃแƒซแƒ˜แƒกแƒ—แƒ•แƒ˜แƒก a1, แƒ›แƒแƒจแƒ˜แƒœ

แƒ›แƒแƒ“แƒ˜แƒ— แƒ•แƒ˜แƒžแƒแƒ•แƒแƒ— แƒ’แƒแƒœแƒกแƒแƒฎแƒ˜แƒšแƒ•แƒ”แƒšแƒ˜ แƒ›แƒแƒœแƒแƒ›แƒŸแƒแƒ•แƒ แƒคแƒฃแƒซแƒ˜แƒก แƒฌแƒงแƒแƒšแƒฎแƒกแƒœแƒแƒ แƒ˜แƒก แƒฌแƒแƒœแƒแƒกแƒฌแƒแƒ แƒแƒ‘แƒ˜แƒก pH แƒ›แƒœแƒ˜แƒจแƒ•แƒœแƒ”แƒšแƒแƒ‘แƒ แƒแƒ—แƒแƒฎแƒ˜แƒก แƒขแƒ”แƒ›แƒžแƒ”แƒ แƒแƒขแƒฃแƒ แƒแƒ–แƒ”. แƒคแƒแƒ แƒ›แƒฃแƒšแƒ˜แƒก แƒจแƒ”แƒกแƒแƒ‘แƒแƒ›แƒ˜แƒกแƒแƒ“ (4.7) แƒ’แƒ•แƒแƒฅแƒ•แƒก:

pH = p TO แƒ• - rOH = 14 - rOH.

แƒ›แƒแƒ“แƒ˜แƒ— แƒ’แƒแƒœแƒ•แƒกแƒแƒ–แƒฆแƒ•แƒ แƒแƒ— pOH = [OH - ] แƒ›แƒœแƒ˜แƒจแƒ•แƒœแƒ”แƒšแƒแƒ‘แƒ. แƒชแƒฎแƒแƒ“แƒ˜แƒ

[OH -] = =

แƒ˜แƒœแƒ“แƒ˜แƒ™แƒแƒขแƒแƒ แƒ”แƒ‘แƒ˜แƒก แƒ’แƒแƒ›แƒแƒงแƒ”แƒœแƒ”แƒ‘แƒ˜แƒ— pON = [OH - ], แƒ’แƒ• TO แƒ‘ =แƒ™ แƒ‘แƒ“แƒ

p = , แƒ•แƒ˜แƒฆแƒ”แƒ‘แƒ—: pOH = 0,5 (แƒ’แƒ• TO แƒ‘+ p). แƒแƒ› แƒ’แƒแƒ›แƒแƒฎแƒแƒขแƒ•แƒ˜แƒก แƒฉแƒแƒœแƒแƒชแƒ•แƒšแƒ”แƒ‘แƒ˜แƒ— pH-แƒ˜แƒก แƒ–แƒ”แƒ›แƒแƒแƒฆแƒœแƒ˜แƒจแƒœแƒฃแƒš แƒคแƒแƒ แƒ›แƒฃแƒšแƒแƒจแƒ˜, แƒ›แƒ˜แƒ•แƒ“แƒ˜แƒ•แƒแƒ แƒ— แƒฃแƒ แƒ—แƒ˜แƒ”แƒ แƒ—แƒแƒ‘แƒแƒ›แƒ“แƒ”

pH \u003d 14 - pOH \u003d 14 - 0.5 (p TO แƒ‘+ p).

แƒแƒกแƒ” แƒ แƒแƒ›, แƒฌแƒแƒœแƒแƒกแƒฌแƒแƒ แƒแƒ‘แƒ˜แƒก pH แƒ›แƒœแƒ˜แƒจแƒ•แƒœแƒ”แƒšแƒแƒ‘แƒ แƒกแƒฃแƒกแƒขแƒ˜ แƒ”แƒ แƒ—แƒ›แƒŸแƒแƒ•แƒ แƒคแƒฃแƒซแƒ˜แƒก แƒฎแƒกแƒœแƒแƒ แƒจแƒ˜ แƒจแƒ”แƒ˜แƒซแƒšแƒ”แƒ‘แƒ แƒ’แƒแƒ›แƒแƒ˜แƒ—แƒ•แƒแƒšแƒแƒก แƒคแƒแƒ แƒ›แƒฃแƒšแƒ˜แƒก แƒ’แƒแƒ›แƒแƒงแƒ”แƒœแƒ”แƒ‘แƒ˜แƒ— (4.15):

pH = 14 - 0,5 (แƒ’แƒ• TO แƒ‘+ p). (4.15)

แƒ’แƒแƒ›แƒแƒ—แƒ•แƒแƒšแƒ”แƒ— pH 0,01 แƒ›แƒแƒš/แƒš แƒแƒ›แƒ˜แƒแƒ™แƒ˜แƒก แƒฌแƒงแƒแƒšแƒฎแƒกแƒœแƒแƒ แƒจแƒ˜, แƒ แƒ˜แƒกแƒ—แƒ•แƒ˜แƒกแƒแƒช แƒแƒ—แƒแƒฎแƒ˜แƒก แƒขแƒ”แƒ›แƒžแƒ”แƒ แƒแƒขแƒฃแƒ แƒแƒ–แƒ” TO แƒ‘= แƒ“แƒ แƒ’แƒ• TO แƒ‘ = 4,76.

แƒแƒ›แƒ˜แƒแƒ™แƒ˜แƒก แƒฌแƒงแƒแƒšแƒฎแƒกแƒœแƒแƒ แƒจแƒ˜ แƒฌแƒแƒœแƒแƒกแƒฌแƒแƒ แƒแƒ‘แƒ แƒ“แƒแƒ›แƒงแƒแƒ แƒ”แƒ‘แƒฃแƒšแƒ˜แƒ:

แƒ แƒแƒ›แƒ”แƒšแƒ˜แƒช แƒซแƒ˜แƒ แƒ˜แƒ—แƒแƒ“แƒแƒ“ แƒ’แƒแƒ“แƒแƒฎแƒ แƒ˜แƒšแƒ˜แƒ แƒ›แƒแƒ แƒชแƒฎแƒœแƒ˜แƒ•, แƒ˜แƒกแƒ” แƒ แƒแƒ› แƒแƒ›แƒ˜แƒแƒ™แƒ˜แƒก แƒ˜แƒแƒœแƒ˜แƒ–แƒแƒชแƒ˜แƒ˜แƒก แƒฎแƒแƒ แƒ˜แƒกแƒฎแƒ˜ แƒแƒ แƒ˜แƒก . แƒแƒ›แƒ˜แƒขแƒแƒ›, pH แƒ›แƒœแƒ˜แƒจแƒ•แƒœแƒ”แƒšแƒแƒ‘แƒ˜แƒก แƒ’แƒแƒ›แƒแƒกแƒแƒ—แƒ•แƒšแƒ”แƒšแƒแƒ“, แƒจแƒ”แƒ’แƒ˜แƒซแƒšแƒ˜แƒแƒ— แƒ’แƒแƒ›แƒแƒ˜แƒงแƒ”แƒœแƒแƒ— แƒ™แƒแƒ•แƒจแƒ˜แƒ แƒ˜ (4.15):

pH = 14 - 0,5 (แƒ’แƒ• TO แƒ‘+ p) =

แƒ›แƒกแƒ’แƒแƒ•แƒกแƒ˜ แƒ’แƒแƒœแƒฎแƒ˜แƒšแƒ•แƒ แƒจแƒ”แƒ˜แƒซแƒšแƒ”แƒ‘แƒ แƒ’แƒแƒœแƒฎแƒแƒ แƒชแƒ˜แƒ”แƒšแƒ“แƒ”แƒก แƒœแƒ”แƒ‘แƒ˜แƒกแƒ›แƒ˜แƒ”แƒ แƒ˜ แƒกแƒฃแƒกแƒขแƒ˜แƒกแƒ—แƒ•แƒ˜แƒก แƒžแƒแƒšแƒ˜แƒ›แƒŸแƒแƒ•แƒแƒกแƒแƒคแƒฃแƒซแƒ•แƒ”แƒšแƒ˜. แƒ›แƒแƒ แƒ—แƒแƒšแƒ˜แƒ, แƒแƒ› แƒจแƒ”แƒ›แƒ—แƒฎแƒ•แƒ”แƒ•แƒแƒจแƒ˜ แƒฃแƒคแƒ แƒ แƒ แƒ—แƒฃแƒšแƒ˜ แƒ’แƒแƒ›แƒแƒœแƒแƒ—แƒฅแƒ•แƒแƒ›แƒ”แƒ‘แƒ˜ แƒ›แƒ˜แƒ˜แƒฆแƒ”แƒ‘แƒ.

แƒกแƒฃแƒกแƒขแƒ˜ แƒžแƒแƒšแƒ˜แƒ›แƒŸแƒแƒ•แƒฃแƒ แƒ˜ แƒคแƒฃแƒซแƒ”แƒ”แƒ‘แƒ˜, แƒกแƒฃแƒกแƒขแƒ˜ แƒžแƒแƒšแƒ˜แƒ‘แƒแƒ–แƒฃแƒ แƒ˜ แƒ›แƒŸแƒแƒ•แƒ”แƒ‘แƒ˜แƒก แƒ›แƒกแƒ’แƒแƒ•แƒกแƒแƒ“, แƒ˜แƒจแƒšแƒ”แƒ‘แƒ แƒ”แƒขแƒแƒžแƒแƒ‘แƒ แƒ˜แƒ•แƒแƒ“ แƒ“แƒ แƒ—แƒ˜แƒ—แƒแƒ”แƒฃแƒš แƒ“แƒ˜แƒกแƒแƒชแƒ˜แƒแƒชแƒ˜แƒ˜แƒก แƒกแƒแƒคแƒ”แƒฎแƒฃแƒ แƒก แƒแƒกแƒ”แƒ•แƒ” แƒแƒฅแƒ•แƒก แƒคแƒฃแƒซแƒ˜แƒก แƒ”แƒขแƒแƒžแƒแƒ‘แƒ แƒ˜แƒ•แƒ˜ แƒ“แƒ˜แƒกแƒแƒชแƒ˜แƒแƒชแƒ˜แƒ˜แƒก แƒ›แƒฃแƒ“แƒ›แƒ˜แƒ•แƒ˜ - แƒ”แƒขแƒแƒžแƒแƒ‘แƒ แƒ˜แƒ•แƒ˜ แƒ‘แƒแƒ–แƒ˜แƒกแƒฃแƒ แƒแƒ‘แƒ˜แƒก แƒ›แƒฃแƒ“แƒ›แƒ˜แƒ•แƒ˜.

แƒ›แƒแƒ’แƒแƒšแƒ˜แƒ—แƒแƒ“, แƒขแƒงแƒ•แƒ˜แƒ˜แƒก แƒฐแƒ˜แƒ“แƒ แƒแƒฅแƒกแƒ˜แƒ“แƒ˜ Pb (OH) 2 แƒฌแƒงแƒแƒšแƒฎแƒกแƒœแƒแƒ แƒ”แƒ‘แƒจแƒ˜ แƒ˜แƒจแƒšแƒ”แƒ‘แƒ แƒ˜แƒแƒœแƒ”แƒ‘แƒแƒ“ แƒแƒ  แƒ”แƒขแƒแƒžแƒแƒ“:

แƒ˜แƒ’แƒ˜แƒ•แƒ” แƒฌแƒแƒœแƒแƒกแƒฌแƒแƒ แƒแƒ‘แƒ แƒจแƒ”แƒ˜แƒซแƒšแƒ”แƒ‘แƒ แƒกแƒฎแƒ•แƒแƒ’แƒ•แƒแƒ แƒแƒ“ แƒ“แƒแƒ˜แƒฌแƒ”แƒ แƒแƒก, แƒ“แƒแƒ˜แƒชแƒ•แƒแƒœ (แƒžแƒ แƒแƒขแƒแƒšแƒ˜แƒ–แƒฃแƒ แƒ˜ แƒ—แƒ”แƒแƒ แƒ˜แƒ˜แƒก แƒคแƒแƒ แƒ’แƒšแƒ”แƒ‘แƒจแƒ˜) แƒคแƒฃแƒซแƒ˜แƒก แƒ’แƒแƒœแƒ›แƒแƒ แƒขแƒ”แƒ‘แƒ, แƒ แƒแƒ’แƒแƒ แƒช แƒœแƒ˜แƒ•แƒ—แƒ˜แƒ”แƒ แƒ”แƒ‘แƒ, แƒ แƒแƒ›แƒ”แƒšแƒ˜แƒช แƒแƒ›แƒแƒ’แƒ แƒ”แƒ‘แƒก แƒžแƒ แƒแƒขแƒแƒœแƒก, แƒแƒ› แƒจแƒ”แƒ›แƒ—แƒฎแƒ•แƒ”แƒ•แƒแƒจแƒ˜, แƒ˜แƒฆแƒ”แƒ‘แƒก แƒ›แƒแƒก แƒฌแƒงแƒšแƒ˜แƒก แƒ›แƒแƒšแƒ”แƒ™แƒฃแƒšแƒ˜แƒ“แƒแƒœ:

แƒแƒ› แƒจแƒ”แƒ›แƒ—แƒฎแƒ•แƒ”แƒ•แƒแƒจแƒ˜, แƒ”แƒขแƒแƒžแƒแƒ‘แƒ แƒ˜แƒ•แƒ˜ แƒกแƒแƒ‘แƒแƒ–แƒ˜แƒกแƒ แƒ›แƒฃแƒ“แƒ›แƒ˜แƒ•แƒ”แƒ‘แƒ˜ แƒจแƒ”แƒ˜แƒซแƒšแƒ”แƒ‘แƒ แƒฌแƒแƒ แƒ›แƒแƒ“แƒ’แƒ”แƒœแƒ˜แƒšแƒ˜ แƒ˜แƒงแƒแƒก แƒ แƒแƒ’แƒแƒ แƒช:

แƒแƒ› แƒฌแƒแƒœแƒแƒกแƒฌแƒแƒ แƒแƒ‘แƒ˜แƒก แƒแƒกแƒ”แƒ—แƒ˜ แƒฉแƒแƒœแƒแƒฌแƒ”แƒ แƒ˜แƒ—, แƒ•แƒแƒ แƒแƒฃแƒ“แƒแƒ‘แƒ”แƒœ, แƒ แƒแƒ› แƒžแƒ แƒแƒขแƒแƒœแƒ˜ แƒฌแƒงแƒšแƒ˜แƒก แƒ›แƒแƒšแƒ”แƒ™แƒฃแƒšแƒ˜แƒ“แƒแƒœ แƒ’แƒแƒ“แƒแƒ“แƒ˜แƒก แƒฐแƒ˜แƒ“แƒ แƒแƒฅแƒกแƒ˜แƒšแƒ˜แƒก แƒฏแƒ’แƒฃแƒคแƒจแƒ˜ แƒฌแƒงแƒšแƒ˜แƒก แƒ›แƒแƒšแƒ”แƒ™แƒฃแƒšแƒ˜แƒก แƒฌแƒแƒ แƒ›แƒแƒฅแƒ›แƒœแƒ˜แƒ— (), แƒ แƒ˜แƒก แƒจแƒ”แƒ“แƒ”แƒ’แƒแƒ“แƒแƒช แƒฌแƒงแƒšแƒ˜แƒก แƒ›แƒแƒšแƒ”แƒ™แƒฃแƒšแƒ”แƒ‘แƒ˜แƒก แƒ แƒแƒแƒ“แƒ”แƒœแƒแƒ‘แƒ แƒขแƒงแƒ•แƒ˜แƒ˜แƒก (II) แƒแƒขแƒแƒ›แƒ—แƒแƒœ แƒแƒฎแƒšแƒแƒก. แƒ˜แƒ–แƒ แƒ“แƒ”แƒ‘แƒ แƒ”แƒ แƒ—แƒ˜แƒ—, แƒฎแƒแƒšแƒ แƒขแƒงแƒ•แƒ˜แƒ˜แƒก (II) แƒแƒขแƒแƒ›แƒ—แƒแƒœ แƒ“แƒแƒ™แƒแƒ•แƒจแƒ˜แƒ แƒ”แƒ‘แƒฃแƒšแƒ˜ แƒฐแƒ˜แƒ“แƒ แƒแƒฅแƒกแƒ˜แƒšแƒ˜แƒก แƒฏแƒ’แƒฃแƒคแƒ”แƒ‘แƒ˜แƒก แƒ แƒแƒแƒ“แƒ”แƒœแƒแƒ‘แƒ แƒแƒกแƒ”แƒ•แƒ” แƒ›แƒชแƒ˜แƒ แƒ“แƒ”แƒ‘แƒ แƒ—แƒ˜แƒ—แƒ แƒ“แƒ˜แƒกแƒแƒชแƒ˜แƒแƒชแƒ˜แƒ˜แƒก แƒกแƒแƒคแƒ”แƒฎแƒฃแƒ แƒ–แƒ”.

แƒ›แƒฃแƒจแƒแƒแƒ‘แƒ TO 1 TO 2 =K=[Pb 2+] [OH -] 2 / [Pb (OH) 2] =

2.865 แƒกแƒแƒ“แƒแƒช TO- แƒกแƒ แƒฃแƒšแƒ˜ แƒ“แƒ˜แƒกแƒแƒชแƒ˜แƒแƒชแƒ˜แƒ˜แƒก แƒ›แƒฃแƒ“แƒ›แƒ˜แƒ•แƒ˜ แƒกแƒฅแƒ”แƒ›แƒ˜แƒก แƒ›แƒ˜แƒฎแƒ”แƒ“แƒ•แƒ˜แƒ—

แƒแƒœ แƒกแƒฎแƒ•แƒ แƒกแƒฅแƒ”แƒ›แƒ˜แƒก แƒ›แƒ˜แƒฎแƒ”แƒ“แƒ•แƒ˜แƒ—

แƒ แƒแƒช แƒกแƒแƒ‘แƒแƒšแƒแƒ แƒฏแƒแƒ›แƒจแƒ˜ แƒ˜แƒฌแƒ•แƒ”แƒ•แƒก แƒ˜แƒ›แƒแƒ•แƒ” แƒจแƒ”แƒ“แƒ”แƒ’แƒก.

แƒ™แƒ˜แƒ“แƒ”แƒ• แƒ”แƒ แƒ—แƒ˜ แƒ›แƒแƒ’แƒแƒšแƒ˜แƒ—แƒ˜แƒ แƒแƒ แƒ’แƒแƒœแƒฃแƒšแƒ˜ แƒคแƒฃแƒซแƒ” แƒ”แƒ—แƒ˜แƒšแƒ”แƒœแƒ“แƒ˜แƒแƒ›แƒ˜แƒœแƒ˜, แƒ แƒแƒ›แƒ”แƒšแƒ˜แƒช แƒ’แƒแƒ“แƒ˜แƒก แƒ˜แƒแƒœแƒ˜แƒ–แƒแƒชแƒ˜แƒแƒก แƒฌแƒงแƒแƒšแƒฎแƒกแƒœแƒแƒ แƒจแƒ˜ แƒแƒ  แƒ”แƒขแƒแƒžแƒแƒ“. แƒžแƒ˜แƒ แƒ•แƒ”แƒšแƒ˜ แƒ”แƒขแƒแƒžแƒ˜:

แƒ›แƒ”แƒแƒ แƒ” แƒœแƒแƒ‘แƒ˜แƒฏแƒ˜:

แƒ›แƒฃแƒจแƒแƒแƒ‘แƒ -

แƒ›แƒ—แƒšแƒ˜แƒแƒœแƒ˜ แƒ“แƒ˜แƒกแƒแƒชแƒ˜แƒแƒชแƒ˜แƒ˜แƒก แƒ›แƒฃแƒ“แƒ›แƒ˜แƒ•แƒ˜. แƒ˜แƒก แƒ”แƒ›แƒ—แƒฎแƒ•แƒ”แƒ•แƒ แƒ‘แƒแƒšแƒแƒœแƒกแƒก

แƒฌแƒแƒœแƒแƒกแƒฌแƒแƒ แƒแƒ‘แƒ˜แƒก แƒ›แƒฃแƒ“แƒ›แƒ˜แƒ•แƒ”แƒ‘แƒ˜แƒก แƒ แƒ˜แƒชแƒฎแƒ•แƒ˜แƒ—แƒ˜ แƒ›แƒœแƒ˜แƒจแƒ•แƒœแƒ”แƒšแƒแƒ‘แƒ”แƒ‘แƒ˜ แƒ›แƒแƒชแƒ”แƒ›แƒฃแƒšแƒ˜แƒ แƒ–แƒ”แƒ›แƒแƒ— แƒแƒ—แƒแƒฎแƒ˜แƒก แƒขแƒ”แƒ›แƒžแƒ”แƒ แƒแƒขแƒฃแƒ แƒแƒ–แƒ”.

แƒ แƒแƒ’แƒแƒ แƒช แƒžแƒแƒšแƒ˜แƒ‘แƒแƒ–แƒฃแƒ แƒ˜ แƒ›แƒŸแƒแƒ•แƒ”แƒ‘แƒ˜แƒก แƒจแƒ”แƒ›แƒ—แƒฎแƒ•แƒ”แƒ•แƒแƒจแƒ˜, แƒกแƒฃแƒกแƒขแƒ˜ แƒžแƒแƒšแƒ˜แƒ›แƒŸแƒแƒ•แƒฃแƒ แƒ˜ แƒคแƒฃแƒซแƒ˜แƒกแƒ—แƒ•แƒ˜แƒก, แƒงแƒแƒ•แƒ”แƒšแƒ˜ แƒ›แƒแƒ›แƒ“แƒ”แƒ•แƒœแƒ แƒกแƒแƒคแƒ”แƒฎแƒฃแƒ แƒ˜แƒก แƒ“แƒ˜แƒกแƒแƒชแƒ˜แƒแƒชแƒ˜แƒ˜แƒก แƒ›แƒฃแƒ“แƒ›แƒ˜แƒ•แƒ˜ แƒฉแƒ•แƒ”แƒฃแƒšแƒ”แƒ‘แƒ แƒ˜แƒ• แƒ แƒแƒ›แƒ“แƒ”แƒœแƒ˜แƒ›แƒ” แƒ แƒ˜แƒ’แƒ˜แƒ— แƒœแƒแƒ™แƒšแƒ”แƒ‘แƒ˜แƒ แƒฌแƒ˜แƒœแƒ แƒกแƒแƒคแƒ”แƒฎแƒฃแƒ แƒ˜แƒก แƒ“แƒ˜แƒกแƒแƒชแƒ˜แƒแƒชแƒ˜แƒ˜แƒก แƒ›แƒฃแƒ“แƒ›แƒ˜แƒ•แƒ–แƒ”.

แƒ›แƒแƒ’แƒ˜แƒ“แƒแƒ–แƒ”. 4.2 แƒ’แƒ•แƒ˜แƒฉแƒ•แƒ”แƒœแƒ”แƒ‘แƒก แƒ–แƒแƒ’แƒ˜แƒ”แƒ แƒ—แƒ˜ แƒกแƒฃแƒกแƒขแƒ˜ แƒ›แƒŸแƒแƒ•แƒ˜แƒกแƒ แƒ“แƒ แƒคแƒฃแƒซแƒ˜แƒก แƒ›แƒŸแƒแƒ•แƒ˜แƒแƒœแƒแƒ‘แƒ˜แƒก แƒ“แƒ แƒคแƒฃแƒซแƒ”แƒแƒ‘แƒ˜แƒก แƒ›แƒฃแƒ“แƒ›แƒ˜แƒ•แƒ”แƒ‘แƒ˜แƒก แƒ แƒ˜แƒชแƒฎแƒ•แƒ˜แƒ—แƒ˜ แƒ›แƒœแƒ˜แƒจแƒ•แƒœแƒ”แƒšแƒแƒ‘แƒ”แƒ‘แƒก.

แƒชแƒฎแƒ แƒ˜แƒšแƒ˜ 4.2. แƒญแƒ”แƒจแƒ›แƒแƒ แƒ˜แƒขแƒ˜ แƒ—แƒ”แƒ แƒ›แƒแƒ“แƒ˜แƒœแƒแƒ›แƒ˜แƒ™แƒฃแƒ แƒ˜ แƒ˜แƒแƒœแƒ˜แƒ–แƒแƒชแƒ˜แƒ˜แƒก แƒ›แƒฃแƒ“แƒ›แƒ˜แƒ•แƒ”แƒ‘แƒ˜ แƒ–แƒแƒ’แƒ˜แƒ”แƒ แƒ—แƒ˜ แƒ›แƒŸแƒแƒ•แƒ˜แƒกแƒ แƒ“แƒ แƒคแƒฃแƒซแƒ˜แƒก แƒฌแƒงแƒแƒšแƒฎแƒกแƒœแƒแƒ แƒ”แƒ‘แƒจแƒ˜.

TO แƒ- แƒ›แƒŸแƒแƒ•แƒ˜แƒแƒœแƒแƒ‘แƒ˜แƒก แƒ›แƒฃแƒ“แƒ›แƒ˜แƒ•แƒ˜, TO แƒ‘- แƒกแƒแƒ‘แƒแƒ–แƒ˜แƒกแƒ แƒ›แƒฃแƒ“แƒ›แƒ˜แƒ•แƒ˜,

TO 1 - แƒ“แƒ˜แƒกแƒแƒชแƒ˜แƒแƒชแƒ˜แƒ˜แƒก แƒ›แƒฃแƒ“แƒ›แƒ˜แƒ•แƒ˜ แƒžแƒ˜แƒ แƒ•แƒ”แƒšแƒ˜ แƒ”แƒขแƒแƒžแƒ˜แƒกแƒ—แƒ•แƒ˜แƒก,

TO 2 - แƒ“แƒ˜แƒกแƒแƒชแƒ˜แƒแƒชแƒ˜แƒ˜แƒก แƒ›แƒฃแƒ“แƒ›แƒ˜แƒ•แƒ˜ แƒ›แƒ”แƒแƒ แƒ” แƒ”แƒขแƒแƒžแƒ˜แƒกแƒ—แƒ•แƒ˜แƒก แƒ“แƒ แƒ.แƒจ.

pH แƒ›แƒœแƒ˜แƒจแƒ•แƒœแƒ”แƒšแƒแƒ‘แƒ



แƒฌแƒงแƒแƒšแƒ˜, แƒ แƒแƒ’แƒแƒ แƒช แƒกแƒฃแƒกแƒขแƒ˜ แƒ”แƒšแƒ”แƒฅแƒขแƒ แƒแƒšแƒ˜แƒขแƒ˜, แƒ›แƒชแƒ˜แƒ แƒ” แƒ แƒแƒแƒ“แƒ”แƒœแƒแƒ‘แƒ˜แƒ— แƒ’แƒแƒœแƒ˜แƒชแƒ“แƒ˜แƒก แƒ˜แƒแƒœแƒ˜แƒ–แƒแƒชแƒ˜แƒแƒก:

H 2 O โ†” H + + OH -.

แƒฌแƒงแƒแƒšแƒฎแƒกแƒœแƒแƒ แƒจแƒ˜ แƒ˜แƒแƒœแƒ”แƒ‘แƒ˜ แƒ’แƒแƒœแƒ˜แƒชแƒ“แƒ˜แƒแƒœ แƒ“แƒแƒขแƒ”แƒœแƒ˜แƒแƒœแƒ”แƒ‘แƒแƒก (แƒฌแƒง.)

แƒฌแƒงแƒแƒšแƒก แƒแƒฎแƒแƒกแƒ˜แƒแƒ—แƒ”แƒ‘แƒก แƒžแƒ แƒแƒขแƒแƒšแƒ˜แƒ–แƒฃแƒ แƒ˜ แƒแƒ›แƒคแƒแƒขแƒ”แƒ แƒ˜แƒฃแƒšแƒแƒ‘แƒ. แƒฌแƒงแƒšแƒ˜แƒก แƒ—แƒ•แƒ˜แƒ—แƒ˜แƒแƒœแƒ˜แƒ–แƒแƒชแƒ˜แƒ˜แƒก แƒ แƒ”แƒแƒฅแƒชแƒ˜แƒ (แƒแƒ•แƒขแƒแƒžแƒ แƒแƒขแƒแƒšแƒ˜แƒ–แƒ˜), แƒ แƒแƒ›แƒšแƒ˜แƒก แƒ“แƒ แƒแƒกแƒแƒช แƒžแƒ แƒแƒขแƒแƒœแƒ˜ แƒฌแƒงแƒšแƒ˜แƒก แƒ”แƒ แƒ—แƒ˜ แƒ›แƒแƒšแƒ”แƒ™แƒฃแƒšแƒ˜แƒ“แƒแƒœ (แƒ›แƒŸแƒแƒ•แƒ) แƒ’แƒแƒ“แƒแƒ“แƒ˜แƒก แƒฌแƒงแƒšแƒ˜แƒก แƒ›แƒ”แƒแƒ แƒ” แƒ›แƒแƒšแƒ”แƒ™แƒฃแƒšแƒแƒจแƒ˜ (แƒ‘แƒแƒ–แƒแƒจแƒ˜), แƒแƒฆแƒฌแƒ”แƒ แƒ˜แƒšแƒ˜แƒ แƒ’แƒแƒœแƒขแƒแƒšแƒ”แƒ‘แƒ˜แƒ—:

H 2 O + H 2 O โ†” H 3 O + + OH -.

แƒฌแƒงแƒšแƒ˜แƒก แƒแƒ•แƒขแƒแƒžแƒ แƒแƒขแƒแƒšแƒ˜แƒ–แƒ˜แƒก แƒฌแƒแƒœแƒแƒกแƒฌแƒแƒ แƒแƒ‘แƒ˜แƒก แƒ›แƒฃแƒ“แƒ›แƒ˜แƒ•แƒ˜ แƒขแƒแƒšแƒ˜แƒ:

แƒ›แƒแƒกแƒ˜แƒก แƒ›แƒแƒฅแƒ›แƒ”แƒ“แƒ”แƒ‘แƒ˜แƒก แƒ™แƒแƒœแƒแƒœแƒ˜ แƒ›แƒแƒฅแƒ›แƒ”แƒ“แƒ”แƒ‘แƒก แƒ˜แƒแƒœแƒ˜แƒ–แƒแƒชแƒ˜แƒ˜แƒก แƒ›แƒฃแƒ“แƒ›แƒ˜แƒ•แƒ–แƒ”:

แƒกแƒแƒ“แƒแƒช a แƒแƒ แƒ˜แƒก แƒแƒฅแƒขแƒ˜แƒ•แƒแƒ‘แƒ.

แƒ›แƒแƒ™แƒšแƒ”แƒ“, H 3 O +-แƒ˜แƒก แƒœแƒแƒชแƒ•แƒšแƒแƒ“ แƒ›แƒŸแƒแƒ•แƒ-แƒขแƒฃแƒขแƒแƒ•แƒแƒœแƒ˜ แƒฌแƒแƒœแƒแƒกแƒฌแƒแƒ แƒแƒ‘แƒแƒจแƒ˜ แƒ•แƒฌแƒ”แƒ แƒ—

แƒ•แƒ˜แƒœแƒแƒ˜แƒ“แƒแƒœ แƒฌแƒงแƒแƒšแƒ˜ แƒ“แƒ˜แƒ“แƒ˜ แƒกแƒ˜แƒญแƒแƒ แƒ‘แƒ˜แƒ— แƒฎแƒกแƒœแƒแƒ แƒจแƒ˜แƒ แƒ“แƒ แƒ›แƒชแƒ˜แƒ แƒ” แƒ แƒแƒแƒ“แƒ”แƒœแƒแƒ‘แƒ˜แƒ— แƒ’แƒแƒœแƒ˜แƒชแƒ“แƒ˜แƒก แƒ˜แƒแƒœแƒ˜แƒ–แƒแƒชแƒ˜แƒแƒก, แƒจแƒ”แƒ˜แƒซแƒšแƒ”แƒ‘แƒ แƒแƒฆแƒ˜แƒœแƒ˜แƒจแƒœแƒแƒก, แƒ แƒแƒ› แƒ›แƒ˜แƒกแƒ˜ แƒ™แƒแƒœแƒชแƒ”แƒœแƒขแƒ แƒแƒชแƒ˜แƒ แƒ›แƒฃแƒ“แƒ›แƒ˜แƒ•แƒ˜แƒ แƒ“แƒ แƒฃแƒ“แƒ แƒ˜แƒก 55,6 แƒ›แƒแƒšแƒ˜ (1000 แƒ’: 18 แƒ’/แƒ›แƒแƒš \u003d 56 แƒ›แƒแƒšแƒ˜) แƒšแƒ˜แƒขแƒ  แƒฌแƒงแƒแƒšแƒ–แƒ”.

แƒแƒ›แƒ แƒ˜แƒ’แƒแƒ“, K แƒ“แƒ (H 2 O) แƒžแƒ แƒแƒ“แƒฃแƒฅแƒขแƒ˜ แƒ“แƒ แƒฌแƒงแƒšแƒ˜แƒก แƒ™แƒแƒœแƒชแƒ”แƒœแƒขแƒ แƒแƒชแƒ˜แƒ แƒแƒ แƒ˜แƒก 1.8 10 -16 แƒ›แƒแƒš / แƒš 55.6 แƒ›แƒแƒš / แƒš \u003d 10 -14 แƒ›แƒแƒšแƒ˜ 2 / แƒš 2. แƒแƒ›แƒ แƒ˜แƒ’แƒแƒ“, \u003d 10 -14 (25 ยฐ C แƒขแƒ”แƒ›แƒžแƒ”แƒ แƒแƒขแƒฃแƒ แƒแƒ–แƒ”) แƒแƒ แƒ˜แƒก แƒ›แƒฃแƒ“แƒ›แƒ˜แƒ•แƒ˜ แƒ›แƒœแƒ˜แƒจแƒ•แƒœแƒ”แƒšแƒแƒ‘แƒ, แƒแƒฆแƒ˜แƒœแƒ˜แƒจแƒœแƒ”แƒ‘แƒ Kwแƒ“แƒ แƒ“แƒแƒฃแƒ แƒ”แƒ™แƒ แƒฌแƒงแƒšแƒ˜แƒก แƒแƒ•แƒขแƒแƒžแƒ แƒแƒขแƒแƒšแƒ˜แƒ–แƒ˜แƒก แƒ›แƒฃแƒ“แƒ›แƒ˜แƒ•แƒ˜. แƒ–แƒแƒ’แƒฏแƒ”แƒ  แƒ˜แƒกแƒ˜แƒœแƒ˜ แƒ˜แƒงแƒ”แƒœแƒ”แƒ‘แƒ”แƒœ แƒ›แƒแƒซแƒ•แƒ”แƒšแƒ”แƒ‘แƒฃแƒš แƒกแƒแƒฎแƒ”แƒšแƒก - แƒฌแƒงแƒšแƒ˜แƒก แƒ˜แƒแƒœแƒฃแƒ  แƒžแƒ แƒแƒ“แƒฃแƒฅแƒขแƒก.

แƒฎแƒกแƒœแƒแƒ แƒ”แƒ‘แƒก, แƒ แƒแƒ›แƒšแƒ”แƒ‘แƒจแƒ˜แƒช แƒฌแƒงแƒแƒšแƒ‘แƒแƒ“แƒ˜แƒก แƒ˜แƒแƒœแƒ”แƒ‘แƒ˜แƒก แƒ“แƒ แƒฐแƒ˜แƒ“แƒ แƒแƒฅแƒกแƒ˜แƒ“แƒ˜แƒก แƒ˜แƒแƒœแƒ”แƒ‘แƒ˜แƒก แƒ™แƒแƒœแƒชแƒ”แƒœแƒขแƒ แƒแƒชแƒ˜แƒ แƒ”แƒ แƒ—แƒœแƒแƒ˜แƒ แƒ˜แƒ, แƒœแƒ”แƒ˜แƒขแƒ แƒแƒšแƒฃแƒ  แƒฎแƒกแƒœแƒแƒ แƒ”แƒ‘แƒก แƒฃแƒฌแƒแƒ“แƒ”แƒ‘แƒ”แƒœ = = = 10 -7 แƒ›แƒแƒš/แƒš. แƒ›แƒŸแƒแƒ•แƒ” แƒฎแƒกแƒœแƒแƒ แƒ”แƒ‘แƒจแƒ˜ > , > 10 -7 แƒ›แƒแƒš/แƒš, แƒฎแƒแƒšแƒ แƒขแƒฃแƒขแƒ”แƒจแƒ˜ > , > 10 -7 แƒ›แƒแƒš/แƒš.



แƒกแƒ˜แƒ›แƒแƒ แƒขแƒ˜แƒ•แƒ˜แƒกแƒ—แƒ•แƒ˜แƒก, แƒกแƒแƒคแƒฃแƒซแƒ•แƒšแƒแƒ“ แƒแƒฆแƒ”แƒ‘แƒฃแƒšแƒ˜แƒ pH แƒ›แƒœแƒ˜แƒจแƒ•แƒœแƒ”แƒšแƒแƒ‘แƒ - แƒฌแƒงแƒแƒšแƒ‘แƒแƒ“แƒ˜แƒก แƒ˜แƒแƒœแƒ”แƒ‘แƒ˜แƒก แƒ™แƒแƒœแƒชแƒ”แƒœแƒขแƒ แƒแƒชแƒ˜แƒ˜แƒก แƒแƒ—แƒแƒ‘แƒ˜แƒ—แƒ˜ แƒšแƒแƒ’แƒแƒ แƒ˜แƒ—แƒ›แƒ˜, แƒแƒฆแƒ”แƒ‘แƒฃแƒšแƒ˜ แƒกแƒแƒžแƒ˜แƒ แƒ˜แƒกแƒžแƒ˜แƒ แƒ แƒœแƒ˜แƒจแƒœแƒ˜แƒ—: pH \u003d -lg.

แฒกแƒแƒ˜แƒœแƒขแƒ”แƒ แƒ”แƒกแƒ แƒคแƒแƒฅแƒขแƒ”แƒ‘แƒ˜:

แƒ˜แƒ–แƒแƒฐแƒ˜แƒ“แƒ แƒ˜แƒ˜แƒก แƒ›แƒ“แƒ’แƒแƒ›แƒแƒ แƒ”แƒแƒ‘แƒ˜แƒก แƒ“แƒแƒ แƒฆแƒ•แƒ”แƒ•แƒ ( pH แƒ›แƒฃแƒ“แƒ›แƒ˜แƒ•แƒแƒ‘แƒ) แƒแƒฆแƒ˜แƒœแƒ˜แƒจแƒœแƒ”แƒ‘แƒ แƒ’แƒฃแƒš-แƒกแƒ˜แƒกแƒฎแƒšแƒซแƒแƒ แƒฆแƒ•แƒ—แƒ แƒ“แƒแƒแƒ•แƒแƒ“แƒ”แƒ‘แƒ”แƒ‘แƒจแƒ˜, แƒ˜แƒจแƒ”แƒ›แƒ˜แƒ˜แƒ—, แƒจแƒแƒฅแƒ แƒ˜แƒแƒœแƒ˜ แƒ“แƒ˜แƒแƒ‘แƒ”แƒขแƒ˜แƒ— (แƒ’แƒแƒœแƒ•แƒ˜แƒ—แƒแƒ แƒ“แƒ”แƒ‘แƒ แƒแƒชแƒ˜แƒ“แƒแƒ–แƒ˜). แƒ›แƒŸแƒแƒ•แƒ-แƒขแƒฃแƒขแƒแƒ•แƒแƒœแƒ˜ แƒ‘แƒแƒšแƒแƒœแƒกแƒ˜ แƒ˜แƒœแƒแƒ แƒฉแƒฃแƒœแƒ”แƒ‘แƒก แƒกแƒฃแƒœแƒ—แƒฅแƒ•แƒแƒก, แƒจแƒแƒ แƒ“แƒ•แƒแƒก, แƒแƒคแƒšแƒ˜แƒแƒœแƒแƒ‘แƒแƒก. แƒ”แƒก แƒกแƒ˜แƒกแƒขแƒ”แƒ›แƒ”แƒ‘แƒ˜ แƒœแƒ”แƒšแƒ แƒ›แƒฃแƒจแƒแƒแƒ‘แƒ”แƒœ แƒ“แƒ แƒ›แƒŸแƒแƒ•แƒ” แƒ“แƒ แƒขแƒฃแƒขแƒ” แƒ›แƒ”แƒขแƒแƒ‘แƒแƒšแƒฃแƒ แƒ˜ แƒžแƒ แƒแƒ“แƒฃแƒฅแƒขแƒ”แƒ‘แƒ˜แƒก แƒ“แƒแƒฃแƒงแƒแƒ•แƒœแƒ”แƒ‘แƒ”แƒšแƒ˜ แƒ’แƒแƒœแƒ”แƒ˜แƒขแƒ แƒแƒšแƒ”แƒ‘แƒ แƒฎแƒ“แƒ”แƒ‘แƒ แƒกแƒฎแƒ”แƒฃแƒšแƒ˜แƒก แƒ‘แƒฃแƒคแƒ”แƒ แƒฃแƒšแƒ˜ แƒกแƒ˜แƒกแƒขแƒ”แƒ›แƒ”แƒ‘แƒ˜แƒก แƒ›แƒ˜แƒ”แƒ . แƒ˜แƒ–แƒแƒฐแƒ˜แƒ“แƒ แƒ˜แƒ˜แƒก แƒ›แƒ“แƒ’แƒแƒ›แƒแƒ แƒ”แƒแƒ‘แƒ แƒฃแƒ–แƒ แƒฃแƒœแƒ•แƒ”แƒšแƒงแƒแƒคแƒ˜แƒšแƒ˜แƒ แƒ›แƒ—แƒ”แƒšแƒ˜ แƒ แƒ˜แƒ’แƒ˜ แƒคแƒ˜แƒ–แƒ˜แƒ™แƒแƒฅแƒ˜แƒ›แƒ˜แƒฃแƒ แƒ˜ แƒ“แƒ แƒคแƒ˜แƒ–แƒ˜แƒแƒšแƒแƒ’แƒ˜แƒฃแƒ แƒ˜ แƒ›แƒ”แƒฅแƒแƒœแƒ˜แƒ–แƒ›แƒ”แƒ‘แƒ˜แƒก แƒ”แƒ แƒ—แƒแƒ‘แƒšแƒ˜แƒ•แƒ˜ แƒ›แƒแƒฅแƒ›แƒ”แƒ“แƒ”แƒ‘แƒ˜แƒ—. แƒ‘แƒฃแƒคแƒ”แƒ แƒฃแƒšแƒ˜ แƒ›แƒแƒฅแƒ›แƒ”แƒ“แƒ”แƒ‘แƒ แƒฃแƒ–แƒ แƒฃแƒœแƒ•แƒ”แƒšแƒงแƒแƒคแƒ˜แƒšแƒ˜แƒ แƒ แƒแƒ›แƒ“แƒ”แƒœแƒ˜แƒ›แƒ” แƒžแƒ แƒแƒขแƒแƒšแƒ˜แƒ–แƒฃแƒ แƒ˜ แƒฌแƒแƒœแƒแƒกแƒฌแƒแƒ แƒแƒ‘แƒ˜แƒก แƒ™แƒแƒ›แƒ‘แƒ˜แƒœแƒแƒชแƒ˜แƒ˜แƒ—.

แƒ›แƒŸแƒแƒ•แƒ”แƒ‘แƒ˜แƒก แƒกแƒ˜แƒซแƒšแƒ˜แƒ”แƒ แƒ” แƒ’แƒแƒœแƒ˜แƒกแƒแƒ–แƒฆแƒ•แƒ แƒ”แƒ‘แƒ แƒžแƒ แƒแƒขแƒแƒœแƒ˜แƒก แƒ“แƒแƒœแƒแƒชแƒ˜แƒ˜แƒก แƒฃแƒœแƒแƒ แƒ˜แƒ—.แƒแƒ› แƒฃแƒœแƒแƒ แƒ˜แƒก แƒกแƒแƒ–แƒแƒ›แƒ˜แƒ แƒ›แƒŸแƒแƒ•แƒ˜แƒแƒœแƒแƒ‘แƒ˜แƒก แƒ›แƒฃแƒ“แƒ›แƒ˜แƒ•แƒ˜ (Ka).

แƒ แƒแƒช แƒฃแƒคแƒ แƒ แƒ“แƒ˜แƒ“แƒ˜แƒ แƒ›แƒŸแƒแƒ•แƒ˜แƒแƒœแƒแƒ‘แƒ˜แƒก แƒ›แƒฃแƒ“แƒ›แƒ˜แƒ•แƒ˜, แƒ›แƒ˜แƒ— แƒฃแƒคแƒ แƒ แƒซแƒšแƒ˜แƒ”แƒ แƒ˜แƒ แƒ›แƒŸแƒแƒ•แƒ.แƒ›แƒแƒ’แƒแƒšแƒ˜แƒ—แƒแƒ“, แƒซแƒ›แƒแƒ แƒ›แƒŸแƒแƒ•แƒ แƒฃแƒคแƒ แƒ แƒซแƒšแƒ˜แƒ”แƒ แƒ˜แƒ แƒ•แƒ˜แƒ“แƒ แƒ” แƒฐแƒ˜แƒ“แƒ แƒแƒชแƒ˜แƒแƒœแƒ›แƒŸแƒแƒ•แƒ, แƒ แƒแƒ“แƒ’แƒแƒœ Ka (CH 3 COOH) \u003d 1.74 10 -5, Ka (HCN) \u003d 1 10 -9. แƒ’แƒแƒ›แƒแƒ—แƒ•แƒšแƒ”แƒ‘แƒ˜แƒกแƒ แƒ“แƒ แƒฉแƒแƒฌแƒ”แƒ แƒ˜แƒก แƒ›แƒแƒฎแƒ”แƒ แƒฎแƒ”แƒ‘แƒฃแƒšแƒแƒ‘แƒ˜แƒกแƒ—แƒ•แƒ˜แƒก แƒ˜แƒกแƒ˜แƒœแƒ˜ แƒฎแƒจแƒ˜แƒ แƒแƒ“ แƒ˜แƒงแƒ”แƒœแƒ”แƒ‘แƒ”แƒœ แƒแƒ แƒ แƒ—แƒแƒ•แƒแƒ“ แƒ›แƒฃแƒ“แƒ›แƒ˜แƒ•แƒ”แƒ‘แƒก, แƒแƒ แƒแƒ›แƒ”แƒ“ แƒ›แƒแƒ— แƒฃแƒแƒ แƒงแƒแƒคแƒ˜แƒ— แƒแƒ—แƒแƒ‘แƒ˜แƒ—แƒ˜ แƒšแƒแƒ’แƒแƒ แƒ˜แƒ—แƒ›แƒ”แƒ‘แƒก: pKa = -lgKa. pKa แƒ›แƒœแƒ˜แƒจแƒ•แƒœแƒ”แƒšแƒแƒ‘แƒ แƒ”แƒฌแƒแƒ“แƒ”แƒ‘แƒ แƒ›แƒŸแƒแƒ•แƒ แƒกแƒ˜แƒซแƒšแƒ˜แƒ”แƒ แƒ”.แƒ แƒแƒช แƒฃแƒคแƒ แƒ แƒ“แƒ˜แƒ“แƒ˜แƒ pKa แƒ›แƒœแƒ˜แƒจแƒ•แƒœแƒ”แƒšแƒแƒ‘แƒ, แƒ›แƒ˜แƒ— แƒฃแƒคแƒ แƒ แƒกแƒฃแƒกแƒขแƒ˜แƒ แƒ›แƒŸแƒแƒ•แƒ.

แƒซแƒšแƒ˜แƒ”แƒ แƒ˜ แƒ›แƒŸแƒแƒ•แƒ”แƒ‘แƒ˜ แƒ—แƒ˜แƒ—แƒฅแƒ›แƒ˜แƒก แƒ›แƒ—แƒšแƒ˜แƒแƒœแƒแƒ“ แƒฉแƒฃแƒฅแƒœแƒ˜แƒแƒœ แƒ—แƒแƒ•แƒ˜แƒแƒœแƒ— แƒžแƒ แƒแƒขแƒแƒœแƒก แƒฌแƒงแƒšแƒ˜แƒก แƒ›แƒแƒšแƒ”แƒ™แƒฃแƒšแƒ”แƒ‘แƒก, แƒแƒ›แƒ˜แƒขแƒแƒ› แƒฎแƒกแƒœแƒแƒ แƒจแƒ˜ แƒแƒ แƒกแƒ”แƒ‘แƒฃแƒšแƒ˜ แƒ›แƒŸแƒแƒ•แƒ แƒกแƒ˜แƒœแƒแƒ›แƒ“แƒ•แƒ˜แƒšแƒ”แƒจแƒ˜ แƒแƒ แƒ˜แƒก แƒฐแƒ˜แƒ“แƒ แƒแƒœแƒ˜แƒฃแƒ›แƒ˜แƒก แƒ˜แƒแƒœแƒ˜.

แƒแƒ›แƒแƒกแƒ—แƒแƒœ แƒ“แƒแƒ™แƒแƒ•แƒจแƒ˜แƒ แƒ”แƒ‘แƒ˜แƒ—, แƒซแƒšแƒ˜แƒ”แƒ แƒ˜ แƒ›แƒแƒœแƒแƒ‘แƒแƒ–แƒฃแƒ แƒ˜ แƒ›แƒŸแƒแƒ•แƒ˜แƒก แƒฎแƒกแƒœแƒแƒ แƒ˜แƒก pH-แƒ˜แƒก แƒ’แƒแƒ›แƒแƒ—แƒ•แƒšแƒ˜แƒกแƒแƒก, แƒžแƒ แƒแƒขแƒแƒœแƒ”แƒ‘แƒ˜แƒก แƒ™แƒแƒœแƒชแƒ”แƒœแƒขแƒ แƒแƒชแƒ˜แƒ แƒฃแƒขแƒแƒšแƒ“แƒ”แƒ‘แƒ แƒ›แƒŸแƒแƒ•แƒแƒก แƒ™แƒแƒœแƒชแƒ”แƒœแƒขแƒ แƒแƒชแƒ˜แƒแƒก.

แƒ’(H 3 O +) = แƒ’(HB).

แƒกแƒฃแƒกแƒขแƒ˜ แƒ›แƒŸแƒแƒ•แƒ”แƒ‘แƒ˜แƒก แƒฎแƒกแƒœแƒแƒ แƒ”แƒ‘แƒจแƒ˜ แƒฐแƒ˜แƒ“แƒ แƒแƒœแƒ˜แƒฃแƒ›แƒ˜แƒก แƒ˜แƒแƒœแƒ”แƒ‘แƒ˜แƒก แƒ™แƒแƒœแƒชแƒ”แƒœแƒขแƒ แƒแƒชแƒ˜แƒ แƒ’แƒแƒชแƒ˜แƒšแƒ”แƒ‘แƒ˜แƒ— แƒ“แƒแƒ‘แƒแƒšแƒ˜แƒ, แƒ•แƒ˜แƒ“แƒ แƒ” แƒ›แƒŸแƒแƒ•แƒแƒก แƒ™แƒแƒœแƒชแƒ”แƒœแƒขแƒ แƒแƒชแƒ˜แƒ. แƒ˜แƒ’แƒ˜ แƒ’แƒแƒ›แƒแƒ˜แƒ—แƒ•แƒšแƒ”แƒ‘แƒ แƒกแƒแƒคแƒฃแƒซแƒ•แƒ”แƒšแƒ–แƒ”

แƒแƒ› แƒ’แƒแƒœแƒขแƒแƒšแƒ”แƒ‘แƒ˜แƒก แƒแƒ แƒ˜แƒ•แƒ” แƒœแƒแƒฌแƒ˜แƒšแƒ˜ แƒ˜แƒซแƒšแƒ”แƒ•แƒ แƒคแƒแƒ แƒ›แƒฃแƒšแƒแƒก แƒกแƒฃแƒกแƒขแƒ˜ แƒ›แƒŸแƒแƒ•แƒ”แƒ‘แƒ˜แƒก แƒฎแƒกแƒœแƒแƒ แƒ”แƒ‘แƒ˜แƒก pH-แƒ˜แƒก แƒ’แƒแƒ›แƒแƒกแƒแƒ—แƒ•แƒšแƒ”แƒšแƒแƒ“: pH = 0,5 (pKa - lg แƒ’(HB)).


แƒกแƒฃแƒกแƒขแƒ˜ แƒ›แƒŸแƒแƒ•แƒ”แƒ‘แƒ˜แƒก แƒ“แƒ˜แƒกแƒแƒชแƒ˜แƒแƒชแƒ˜แƒ˜แƒก แƒ›แƒฃแƒ“แƒ›แƒ˜แƒ•แƒ”แƒ‘แƒ˜

แƒ›แƒŸแƒแƒ•แƒ

TO แƒ

แƒ  TO แƒ=-แƒšแƒ’ TO แƒ

แƒแƒ–แƒแƒขแƒแƒ•แƒแƒœแƒ˜

แƒแƒ›แƒ˜แƒœแƒแƒแƒชแƒ˜แƒฃแƒ แƒ˜

แƒ‘แƒ”แƒœแƒ–แƒแƒฃแƒ แƒ˜

แƒ‘แƒแƒ แƒ˜ (แƒแƒ แƒ—แƒแƒ‘แƒแƒ แƒฃแƒšแƒ˜)

แƒขแƒ”แƒขแƒ แƒแƒ‘แƒแƒ แƒœแƒแƒ˜แƒ

แƒžแƒ แƒแƒขแƒแƒšแƒ˜แƒ–แƒฃแƒ แƒ˜ แƒ แƒ”แƒแƒฅแƒชแƒ˜แƒ”แƒ‘แƒ˜แƒก แƒกแƒแƒฎแƒ”แƒ”แƒ‘แƒ˜.

MU "แƒ’แƒแƒ“แƒแƒฌแƒงแƒ•แƒ”แƒขแƒ˜แƒšแƒ”แƒ‘แƒ”แƒ‘แƒ˜" แƒ’แƒ• 52-55

แƒฌแƒงแƒšแƒ˜แƒก แƒแƒ•แƒขแƒแƒžแƒ แƒแƒขแƒแƒšแƒ˜แƒ–แƒ˜. แƒฌแƒงแƒšแƒ˜แƒก แƒ˜แƒแƒœแƒฃแƒ แƒ˜ แƒžแƒ แƒแƒ“แƒฃแƒฅแƒขแƒ˜.MU "แƒ’แƒแƒ“แƒแƒฌแƒงแƒ•แƒ”แƒขแƒ˜แƒšแƒ”แƒ‘แƒ”แƒ‘แƒ˜ยป แƒ’แƒ•แƒ”แƒ แƒ“แƒ˜ 56

แƒฌแƒงแƒšแƒ˜แƒก แƒ›แƒแƒšแƒ”แƒ™แƒฃแƒšแƒ”แƒ‘แƒ˜แƒก แƒ›แƒชแƒ˜แƒ แƒ” แƒœแƒแƒฌแƒ˜แƒšแƒ˜ แƒงแƒแƒ•แƒ”แƒšแƒ—แƒ•แƒ˜แƒก แƒ˜แƒแƒœแƒฃแƒ  แƒ›แƒ“แƒ’แƒแƒ›แƒแƒ แƒ”แƒแƒ‘แƒแƒจแƒ˜แƒ, แƒ—แƒฃแƒ›แƒชแƒ แƒซแƒแƒšแƒ˜แƒแƒœ แƒกแƒฃแƒกแƒขแƒ˜ แƒ”แƒšแƒ”แƒฅแƒขแƒ แƒแƒšแƒ˜แƒขแƒ˜แƒ. แƒฌแƒงแƒšแƒ˜แƒก แƒ˜แƒแƒœแƒ˜แƒ–แƒแƒชแƒ˜แƒ แƒ“แƒ แƒจแƒ”แƒ›แƒ“แƒ’แƒแƒ›แƒ˜ แƒ“แƒ˜แƒกแƒแƒชแƒ˜แƒแƒชแƒ˜แƒ, แƒ แƒแƒ’แƒแƒ แƒช แƒฃแƒ™แƒ•แƒ” แƒแƒฆแƒ˜แƒœแƒ˜แƒจแƒœแƒ, แƒแƒฆแƒฌแƒ”แƒ แƒ˜แƒšแƒ˜แƒ แƒ›แƒŸแƒแƒ•แƒ-แƒขแƒฃแƒขแƒแƒ•แƒแƒœแƒ˜ แƒ“แƒ˜แƒกแƒžแƒ แƒแƒžแƒแƒ แƒชแƒ˜แƒแƒชแƒ˜แƒ˜แƒก แƒแƒœ แƒแƒ•แƒขแƒแƒžแƒ แƒแƒขแƒแƒšแƒ˜แƒ–แƒ˜แƒก แƒžแƒ แƒแƒขแƒแƒšแƒ˜แƒ–แƒฃแƒ แƒ˜ แƒ แƒ”แƒแƒฅแƒชแƒ˜แƒ˜แƒก แƒ’แƒแƒœแƒขแƒแƒšแƒ”แƒ‘แƒ˜แƒ—.

แƒฌแƒงแƒแƒšแƒ˜ แƒซแƒแƒšแƒ˜แƒแƒœ แƒกแƒฃแƒกแƒขแƒ˜ แƒ”แƒšแƒ”แƒฅแƒขแƒ แƒแƒšแƒ˜แƒขแƒ˜แƒ, แƒแƒ›แƒ˜แƒขแƒแƒ› แƒ›แƒ˜แƒฆแƒ”แƒ‘แƒฃแƒšแƒ˜ แƒ™แƒแƒœแƒ˜แƒฃแƒ’แƒแƒขแƒฃแƒ แƒ˜ แƒ›แƒŸแƒแƒ•แƒ แƒ“แƒ แƒ™แƒแƒœแƒ˜แƒฃแƒ’แƒ˜แƒ แƒ”แƒ‘แƒฃแƒšแƒ˜ แƒ‘แƒแƒ–แƒ แƒซแƒšแƒ˜แƒ”แƒ แƒ˜แƒ. แƒแƒ›แƒ˜แƒขแƒแƒ›, แƒแƒ› แƒžแƒ แƒแƒขแƒแƒšแƒ˜แƒ–แƒฃแƒ แƒ˜ แƒ แƒ”แƒแƒฅแƒชแƒ˜แƒ˜แƒก แƒฌแƒแƒœแƒแƒกแƒฌแƒแƒ แƒแƒ‘แƒ แƒ’แƒแƒ“แƒแƒขแƒแƒœแƒ˜แƒšแƒ˜แƒ แƒ›แƒแƒ แƒชแƒฎแƒœแƒ˜แƒ•.

แƒแƒ› แƒฌแƒแƒœแƒแƒกแƒฌแƒแƒ แƒแƒ‘แƒ˜แƒก แƒ›แƒฃแƒ“แƒ›แƒ˜แƒ•แƒ˜ K แƒฃแƒ“แƒ แƒ˜แƒก =

แƒฌแƒงแƒšแƒ˜แƒก แƒ˜แƒแƒœแƒ”แƒ‘แƒ˜แƒก แƒ™แƒแƒœแƒชแƒ”แƒœแƒขแƒ แƒแƒชแƒ˜แƒ˜แƒก แƒžแƒ แƒแƒ“แƒฃแƒฅแƒขแƒ˜แƒก แƒ แƒแƒแƒ“แƒ”แƒœแƒแƒ‘แƒ แƒ˜แƒ•แƒ˜ แƒ›แƒœแƒ˜แƒจแƒ•แƒœแƒ”แƒšแƒแƒ‘แƒ ร— แƒแƒ แƒ˜แƒก แƒฌแƒงแƒšแƒ˜แƒก แƒ˜แƒแƒœแƒฃแƒ แƒ˜ แƒžแƒ แƒแƒ“แƒฃแƒฅแƒขแƒ˜.

แƒฃแƒ“แƒ แƒ˜แƒก: ร— = K แƒขแƒแƒšแƒ˜. ร— 2 = 1 ร— 10 - 14

แƒแƒ›แƒ˜แƒขแƒแƒ›: K H 2O \u003d ร— \u003d 10 - 14 แƒแƒœ แƒ’แƒแƒ›แƒแƒ แƒขแƒ˜แƒ•แƒ”แƒ‘แƒฃแƒšแƒ˜ K H 2O \u003d ร— \u003d 10 - 14

K H 2 O แƒแƒ แƒ˜แƒก แƒฌแƒงแƒšแƒ˜แƒก แƒ˜แƒแƒœแƒฃแƒ แƒ˜ แƒžแƒ แƒแƒ“แƒฃแƒฅแƒขแƒ˜, แƒฌแƒงแƒšแƒ˜แƒก แƒแƒ•แƒขแƒแƒžแƒ แƒแƒขแƒแƒšแƒ˜แƒ–แƒ˜แƒก แƒ›แƒฃแƒ“แƒ›แƒ˜แƒ•แƒ˜ แƒแƒœ แƒฃแƒ‘แƒ แƒแƒšแƒแƒ“ แƒฌแƒงแƒšแƒ˜แƒก แƒ›แƒฃแƒ“แƒ›แƒ˜แƒ•แƒ˜. K H 2 O แƒ“แƒแƒ›แƒแƒ™แƒ˜แƒ“แƒ”แƒ‘แƒฃแƒšแƒ˜แƒ แƒขแƒ”แƒ›แƒžแƒ”แƒ แƒแƒขแƒฃแƒ แƒแƒ–แƒ”. แƒขแƒ”แƒ›แƒžแƒ”แƒ แƒแƒขแƒฃแƒ แƒ˜แƒก แƒ›แƒแƒขแƒ”แƒ‘แƒแƒกแƒ—แƒแƒœ แƒ”แƒ แƒ—แƒแƒ“, แƒ˜แƒก แƒ˜แƒ–แƒ แƒ“แƒ”แƒ‘แƒ.

แƒฅแƒ˜แƒ›แƒ˜แƒฃแƒ แƒแƒ“ แƒกแƒฃแƒคแƒ—แƒ แƒฌแƒงแƒแƒšแƒจแƒ˜ = = = 1ร—10 โ€“ 7 . แƒ”แƒก แƒแƒ แƒ˜แƒก แƒœแƒ”แƒ˜แƒขแƒ แƒแƒšแƒฃแƒ แƒ˜ แƒ’แƒแƒ แƒ”แƒ›แƒ.

แƒฎแƒกแƒœแƒแƒ แƒจแƒ˜ แƒจแƒ”แƒ˜แƒซแƒšแƒ”แƒ‘แƒ แƒ˜แƒงแƒแƒก > - แƒ’แƒแƒ แƒ”แƒ›แƒ แƒ›แƒŸแƒแƒ•แƒ”แƒ แƒแƒœ< โ€“ ัั€ะตะดะฐ ั‰ะตะปะพั‡ะฝะฐั

= ; =

pH แƒ›แƒœแƒ˜แƒจแƒ•แƒœแƒ”แƒšแƒแƒ‘แƒ

แƒฎแƒกแƒœแƒแƒ แƒ”แƒ‘แƒ˜แƒก แƒ›แƒŸแƒแƒ•แƒ˜แƒแƒœแƒแƒ‘แƒ˜แƒก แƒ’แƒแƒกแƒแƒ–แƒแƒ›แƒแƒ“ แƒ’แƒแƒ›แƒแƒ˜แƒงแƒ”แƒœแƒ”แƒ— แƒฌแƒงแƒแƒšแƒ‘แƒแƒ“แƒ˜แƒก แƒ˜แƒแƒœแƒ˜แƒก แƒ™แƒแƒœแƒชแƒ”แƒœแƒขแƒ แƒแƒชแƒ˜แƒ˜แƒก แƒ›แƒแƒฉแƒ•แƒ”แƒœแƒ”แƒ‘แƒ”แƒšแƒ˜ pH.

แƒฌแƒงแƒแƒšแƒ‘แƒแƒ“แƒ˜แƒก แƒ˜แƒœแƒ“แƒ”แƒฅแƒกแƒ˜ แƒแƒ แƒ˜แƒก แƒ›แƒœแƒ˜แƒจแƒ•แƒœแƒ”แƒšแƒแƒ‘แƒ แƒฎแƒกแƒœแƒแƒ แƒจแƒ˜ แƒ—แƒแƒ•แƒ˜แƒกแƒฃแƒคแƒแƒšแƒ˜ แƒฌแƒงแƒแƒšแƒ‘แƒแƒ“แƒ˜แƒก แƒ˜แƒแƒœแƒ”แƒ‘แƒ˜แƒก แƒ™แƒแƒœแƒชแƒ”แƒœแƒขแƒ แƒแƒชแƒ˜แƒ˜แƒก แƒฃแƒแƒ แƒงแƒแƒคแƒ˜แƒ—แƒ˜ แƒแƒ—แƒแƒ‘แƒ˜แƒ—แƒ˜ แƒšแƒแƒ’แƒแƒ แƒ˜แƒ—แƒ›แƒ˜แƒก แƒขแƒแƒšแƒ˜.

pH = โ€“ lg โ‡’ = 10 โ€“ pH

แƒœแƒ”แƒ˜แƒขแƒ แƒแƒšแƒฃแƒ  แƒ’แƒแƒ แƒ”แƒ›แƒแƒจแƒ˜ pH = 7

แƒ›แƒŸแƒแƒ•แƒ” pH-แƒ–แƒ”< 7

แƒขแƒฃแƒขแƒ” pH-แƒจแƒ˜ > 7

แƒกแƒแƒจแƒฃแƒแƒšแƒ แƒกแƒแƒคแƒฃแƒซแƒ•แƒšแƒ˜แƒแƒœแƒแƒ‘แƒ˜แƒก แƒ“แƒแƒกแƒแƒฎแƒแƒกแƒ˜แƒแƒ—แƒ”แƒ‘แƒšแƒแƒ“ แƒ’แƒแƒ›แƒแƒ˜แƒงแƒ”แƒœแƒ”แƒ‘แƒ แƒฐแƒ˜แƒ“แƒ แƒแƒฅแƒกแƒ˜แƒšแƒ˜แƒก แƒ˜แƒœแƒ“แƒ”แƒฅแƒกแƒ˜ pOH.

pOH \u003d - lg [OH -] โ‡’ [OH -] \u003d 10 - pOH

pH + pOH = 14 รž pH = 14 - pOH แƒ“แƒ pOH = 14 - pH

แƒ›แƒŸแƒแƒ•แƒ”แƒ‘แƒ˜แƒกแƒ แƒ“แƒ แƒคแƒฃแƒซแƒ”แƒ”แƒ‘แƒ˜แƒก แƒฎแƒกแƒœแƒแƒ แƒ”แƒ‘แƒ˜แƒกแƒ—แƒ•แƒ˜แƒก pH-แƒ˜แƒก แƒ’แƒแƒ›แƒแƒกแƒแƒ—แƒ•แƒšแƒ”แƒšแƒ˜ แƒคแƒแƒ แƒ›แƒฃแƒšแƒ”แƒ‘แƒ˜.

pH = โ€“ แƒšแƒ’

  1. แƒซแƒšแƒ˜แƒ”แƒ แƒ˜ แƒ›แƒŸแƒแƒ•แƒ”แƒ‘แƒ˜: \u003d C (1/z แƒ›แƒŸแƒแƒ•แƒ”แƒ‘แƒ˜)

แƒ’แƒแƒ›แƒแƒ•แƒ—แƒ•แƒแƒšแƒแƒ— HCl แƒฎแƒกแƒœแƒแƒ แƒ˜แƒก pH ะก(HCl) = 0,1 แƒ›แƒแƒš/แƒš แƒ›แƒ˜แƒกแƒ˜ แƒกแƒ แƒฃแƒšแƒ˜ แƒ“แƒ˜แƒกแƒแƒชแƒ˜แƒแƒชแƒ˜แƒ˜แƒก แƒžแƒ˜แƒ แƒแƒ‘แƒ”แƒ‘แƒจแƒ˜.

C(HCl) = 0,1 แƒ›แƒแƒš/แƒš; pH \u003d - lg 0.1 \u003d 1

2. แƒซแƒšแƒ˜แƒ”แƒ แƒ˜ แƒคแƒฃแƒซแƒ”แƒ”แƒ‘แƒ˜: [OH - ] \u003d C (1/z แƒคแƒฃแƒซแƒ”แƒ”แƒ‘แƒ˜)

แƒ’แƒแƒ›แƒแƒ—แƒ•แƒแƒšแƒ”แƒ— NaOH แƒฎแƒกแƒœแƒแƒ แƒ˜แƒก pH แƒ˜แƒ›แƒแƒ•แƒ” แƒžแƒ˜แƒ แƒแƒ‘แƒ”แƒ‘แƒจแƒ˜.

C(NaOH) = 0,1 แƒ›แƒแƒš/แƒš; = = 10 โ€“ 13; pH \u003d - lg 10 - 13 \u003d 13

3. แƒกแƒฃแƒกแƒขแƒ˜ แƒ›แƒŸแƒแƒ•แƒ”แƒ‘แƒ˜

แƒ’แƒแƒ›แƒแƒ—แƒ•แƒแƒšแƒ”แƒ— แƒซแƒ›แƒแƒ แƒ›แƒŸแƒแƒ•แƒแƒก แƒฎแƒกแƒœแƒแƒ แƒ˜แƒก pH แƒ›แƒแƒšแƒฃแƒ แƒ˜ แƒ™แƒแƒœแƒชแƒ”แƒœแƒขแƒ แƒแƒชแƒ˜แƒ˜แƒ— 0,5 แƒ›แƒแƒš/แƒš. CH 3COOH-แƒ›แƒ“แƒ” \u003d 1.8 ร— 10 - 5.

3ร—10 - 3

pH \u003d - lg 3 ร— 10 - 3 \u003d 2.5

4. แƒกแƒฃแƒกแƒขแƒ˜ แƒ‘แƒแƒ–แƒ”แƒ‘แƒ˜

แƒ’แƒแƒ›แƒแƒ—แƒ•แƒแƒšแƒ”แƒ— แƒแƒ›แƒ˜แƒแƒ™แƒ˜แƒก แƒฎแƒกแƒœแƒแƒ แƒ˜แƒก pH 0,2 แƒ›แƒแƒš/แƒš แƒ›แƒแƒšแƒฃแƒ แƒ˜ แƒ™แƒแƒœแƒชแƒ”แƒœแƒขแƒ แƒแƒชแƒ˜แƒ˜แƒ—.



K NH 3 \u003d 1,76 ร— 10 - 5

1.88ร—10 - 3

0,53 ร— 10 - 11; pH \u003d - lg 0,53 ร— 10 - 11 \u003d 11,3

5. C (H +) \u003d [H +] \u003d 10 - pH

pH = 7, [H + ] = 10 - 7

แƒแƒ แƒกแƒ”แƒ‘แƒแƒ‘แƒก pH-แƒ˜แƒก แƒ’แƒแƒœแƒกแƒแƒ–แƒฆแƒ•แƒ แƒ˜แƒก แƒกแƒฎแƒ•แƒแƒ“แƒแƒกแƒฎแƒ•แƒ แƒ›แƒ”แƒ—แƒแƒ“แƒ˜: แƒ˜แƒœแƒ“แƒ˜แƒ™แƒแƒขแƒแƒ แƒ”แƒ‘แƒ˜แƒกแƒ แƒ“แƒ แƒ˜แƒแƒœแƒแƒ›แƒ”แƒ แƒฃแƒšแƒ˜ แƒ›แƒแƒฌแƒงแƒแƒ‘แƒ˜แƒšแƒแƒ‘แƒ”แƒ‘แƒ˜แƒก แƒ’แƒแƒ›แƒแƒงแƒ”แƒœแƒ”แƒ‘แƒ˜แƒ—.

pH แƒ›แƒœแƒ˜แƒจแƒ•แƒœแƒ”แƒšแƒแƒ‘แƒ แƒแƒ แƒ’แƒแƒœแƒ˜แƒ–แƒ›แƒ˜แƒก แƒฅแƒ˜แƒ›แƒ˜แƒฃแƒ แƒ˜ แƒ แƒ”แƒแƒฅแƒชแƒ˜แƒ”แƒ‘แƒ˜แƒกแƒ แƒ“แƒ แƒ‘แƒ˜แƒแƒฅแƒ˜แƒ›แƒ˜แƒฃแƒ แƒ˜ แƒžแƒ แƒแƒชแƒ”แƒกแƒ”แƒ‘แƒ˜แƒกแƒ—แƒ•แƒ˜แƒก.

แƒ‘แƒ”แƒ•แƒ แƒ˜ แƒ แƒ”แƒแƒฅแƒชแƒ˜แƒ แƒ’แƒแƒ แƒ™แƒ•แƒ”แƒฃแƒšแƒ˜ แƒ›แƒ˜แƒ›แƒแƒ แƒ—แƒฃแƒšแƒ”แƒ‘แƒ˜แƒ— แƒ’แƒแƒกแƒแƒ’แƒ แƒซแƒ”แƒšแƒ”แƒ‘แƒšแƒแƒ“ แƒ›แƒแƒ˜แƒ—แƒฎแƒแƒ•แƒก แƒ’แƒแƒ แƒ”แƒ›แƒแƒก แƒ›แƒ™แƒแƒชแƒ แƒแƒ“ แƒ’แƒแƒœแƒกแƒแƒ–แƒฆแƒ•แƒ แƒฃแƒš pH แƒ›แƒœแƒ˜แƒจแƒ•แƒœแƒ”แƒšแƒแƒ‘แƒแƒก.

แƒฉแƒ•แƒ”แƒฃแƒšแƒ”แƒ‘แƒ แƒ˜แƒ•, แƒฏแƒแƒœแƒกแƒแƒฆ แƒกแƒฎแƒ”แƒฃแƒšแƒจแƒ˜ แƒ‘แƒ˜แƒแƒšแƒแƒ’แƒ˜แƒฃแƒ แƒ˜ แƒกแƒ˜แƒ—แƒฎแƒ”แƒ”แƒ‘แƒ˜แƒก แƒฃแƒ›แƒ”แƒขแƒ”แƒกแƒแƒ‘แƒ˜แƒก แƒ’แƒแƒ แƒ”แƒ›แƒแƒก แƒ แƒ”แƒแƒฅแƒชแƒ˜แƒ แƒœแƒ”แƒ˜แƒขแƒ แƒแƒšแƒฃแƒ แƒ—แƒแƒœ แƒแƒฎแƒšแƒแƒกแƒแƒ.

แƒกแƒ˜แƒกแƒฎแƒšแƒ˜ - 7,4

แƒœแƒ”แƒ แƒฌแƒงแƒ•แƒ˜ - 6,6

แƒœแƒแƒฌแƒšแƒแƒ•แƒ˜แƒก แƒฌแƒ•แƒ”แƒœแƒ˜ - 6,4

แƒœแƒแƒฆแƒ•แƒ”แƒšแƒ˜ - 6,9

แƒจแƒแƒ แƒ“แƒ˜ - 5,6

แƒ™แƒฃแƒญแƒ˜แƒก แƒฌแƒ•แƒ”แƒœแƒ˜: แƒ) แƒ›แƒแƒกแƒ•แƒ”แƒœแƒ”แƒ‘แƒ˜แƒก แƒ“แƒ แƒแƒก - 7.3

แƒ‘) แƒ›แƒแƒœแƒ”แƒšแƒ”แƒ‘แƒ˜แƒก แƒ›แƒ“แƒ’แƒแƒ›แƒแƒ แƒ”แƒแƒ‘แƒแƒจแƒ˜ - 1,5-2

pH-แƒ˜แƒก แƒœแƒแƒ แƒ›แƒ˜แƒ“แƒแƒœ แƒ’แƒแƒ“แƒแƒฎแƒ แƒแƒก แƒแƒฅแƒ•แƒก แƒ“แƒ˜แƒแƒ’แƒœแƒแƒกแƒขแƒ˜แƒ™แƒฃแƒ แƒ˜ (แƒ“แƒแƒแƒ•แƒแƒ“แƒ”แƒ‘แƒ˜แƒก แƒ’แƒแƒœแƒกแƒแƒ–แƒฆแƒ•แƒ แƒ) แƒ“แƒ แƒžแƒ แƒแƒ’แƒœแƒแƒ–แƒฃแƒšแƒ˜ (แƒ“แƒแƒแƒ•แƒแƒ“แƒ”แƒ‘แƒ˜แƒก แƒ›แƒ˜แƒ›แƒ“แƒ˜แƒœแƒแƒ แƒ”แƒแƒ‘แƒ).

แƒแƒชแƒ˜แƒ“แƒแƒ–แƒ˜ แƒแƒ แƒ˜แƒก pH-แƒ˜แƒก แƒชแƒ•แƒšแƒ แƒ›แƒŸแƒแƒ•แƒ” แƒ›แƒฎแƒแƒ แƒ”แƒก, pH แƒ›แƒชแƒ˜แƒ แƒ“แƒ”แƒ‘แƒ, แƒฌแƒงแƒแƒšแƒ‘แƒแƒ“แƒ˜แƒก แƒ˜แƒแƒœแƒ”แƒ‘แƒ˜แƒก แƒ™แƒแƒœแƒชแƒ”แƒœแƒขแƒ แƒแƒชแƒ˜แƒ แƒ˜แƒ–แƒ แƒ“แƒ”แƒ‘แƒ.

แƒแƒšแƒ™แƒแƒšแƒแƒ–แƒ˜ - pH แƒ’แƒแƒ“แƒแƒ“แƒ˜แƒก แƒขแƒฃแƒขแƒ” แƒ แƒ”แƒ’แƒ˜แƒแƒœแƒจแƒ˜, pH แƒ˜แƒ–แƒ แƒ“แƒ”แƒ‘แƒ, แƒฌแƒงแƒแƒšแƒ‘แƒแƒ“แƒ˜แƒก แƒ˜แƒแƒœแƒ”แƒ‘แƒ˜แƒก แƒ™แƒแƒœแƒชแƒ”แƒœแƒขแƒ แƒแƒชแƒ˜แƒ แƒ›แƒชแƒ˜แƒ แƒ“แƒ”แƒ‘แƒ.

แƒกแƒ˜แƒกแƒฎแƒšแƒ˜แƒก pH-แƒ˜แƒก แƒ“แƒ แƒแƒ”แƒ‘แƒ˜แƒ—แƒ˜ แƒ’แƒแƒ“แƒแƒฎแƒ แƒ แƒœแƒแƒ แƒ›แƒ˜แƒ“แƒแƒœ แƒ›แƒ”แƒแƒ—แƒ”แƒ“แƒ”แƒ‘แƒ˜แƒ— แƒ˜แƒฌแƒ•แƒ”แƒ•แƒก แƒแƒ แƒ’แƒแƒœแƒ˜แƒ–แƒ›แƒจแƒ˜ แƒกแƒ”แƒ แƒ˜แƒแƒ–แƒฃแƒš แƒ“แƒแƒ แƒฆแƒ•แƒ”แƒ•แƒ”แƒ‘แƒก. แƒกแƒ˜แƒกแƒฎแƒšแƒ˜แƒก pH-แƒ˜แƒก แƒฎแƒแƒœแƒ’แƒ แƒซแƒšแƒ˜แƒ•แƒ˜ แƒ›แƒ”แƒ แƒงแƒ”แƒแƒ‘แƒ แƒจแƒ”แƒ˜แƒซแƒšแƒ”แƒ‘แƒ แƒคแƒแƒขแƒแƒšแƒฃแƒ แƒ˜ แƒ˜แƒงแƒแƒก. แƒกแƒ˜แƒกแƒฎแƒšแƒ˜แƒก pH-แƒ˜แƒก แƒ’แƒแƒ“แƒแƒฎแƒ แƒ”แƒ‘แƒ˜ แƒจแƒ”แƒ˜แƒซแƒšแƒ”แƒ‘แƒ แƒ˜แƒงแƒแƒก 6,8 - 8, แƒชแƒ•แƒšแƒ˜แƒšแƒ”แƒ‘แƒ”แƒ‘แƒ˜ แƒแƒ› แƒ˜แƒœแƒขแƒ”แƒ แƒ•แƒแƒšแƒ˜แƒก แƒ’แƒแƒ แƒ”แƒ— แƒœแƒ”แƒ‘แƒ˜แƒกแƒ›แƒ˜แƒ”แƒ แƒ˜ แƒ›แƒ˜แƒ›แƒแƒ แƒ—แƒฃแƒšแƒ”แƒ‘แƒ˜แƒ— แƒจแƒ”แƒฃแƒ—แƒแƒ•แƒกแƒ”แƒ‘แƒ”แƒšแƒ˜แƒ แƒกแƒ˜แƒชแƒแƒชแƒฎแƒšแƒ”แƒกแƒ—แƒแƒœ.

แƒ™แƒแƒ›แƒ‘แƒ˜แƒœแƒ˜แƒ แƒ”แƒ‘แƒฃแƒšแƒ˜ แƒ“แƒ แƒ˜แƒ–แƒแƒšแƒ˜แƒ แƒ”แƒ‘แƒฃแƒšแƒ˜ แƒžแƒ แƒแƒขแƒแƒšแƒ˜แƒ–แƒฃแƒ แƒ˜ แƒฌแƒแƒœแƒแƒกแƒฌแƒแƒ แƒแƒ‘แƒ.

แƒžแƒ แƒแƒขแƒแƒšแƒ˜แƒ–แƒฃแƒ แƒ˜ แƒžแƒ แƒแƒชแƒ”แƒกแƒ”แƒ‘แƒ˜ แƒจแƒ”แƒฅแƒชแƒ”แƒ•แƒแƒ“แƒ˜ แƒ แƒ”แƒแƒฅแƒชแƒ˜แƒ”แƒ‘แƒ˜แƒ. แƒžแƒ แƒแƒขแƒแƒšแƒ˜แƒ–แƒฃแƒ แƒ˜ แƒฌแƒแƒœแƒแƒกแƒฌแƒแƒ แƒแƒ‘แƒ แƒ›แƒ˜แƒ™แƒ”แƒ แƒซแƒแƒ”แƒ‘แƒฃแƒšแƒ˜แƒ แƒกแƒฃแƒกแƒขแƒ˜ แƒ›แƒŸแƒแƒ•แƒ”แƒ‘แƒ˜แƒกแƒ แƒ“แƒ แƒคแƒฃแƒซแƒ”แƒ”แƒ‘แƒ˜แƒก แƒฌแƒแƒ แƒ›แƒแƒฅแƒ›แƒœแƒ˜แƒกแƒ™แƒ”แƒœ. แƒ˜แƒกแƒ˜แƒœแƒ˜ แƒจแƒ”แƒ˜แƒซแƒšแƒ”แƒ‘แƒ แƒฉแƒแƒ˜แƒ—แƒ•แƒแƒšแƒแƒก, แƒ แƒแƒ’แƒแƒ แƒช แƒ™แƒแƒœแƒ™แƒฃแƒ แƒ”แƒœแƒชแƒ˜แƒ แƒกแƒฎแƒ•แƒแƒ“แƒแƒกแƒฎแƒ•แƒ แƒกแƒ˜แƒซแƒšแƒ˜แƒ”แƒ แƒ˜แƒก แƒคแƒฃแƒซแƒ”แƒ”แƒ‘แƒก แƒจแƒแƒ แƒ˜แƒก แƒžแƒ แƒแƒขแƒแƒœแƒ˜แƒก แƒคแƒšแƒแƒ‘แƒ˜แƒกแƒ—แƒ•แƒ˜แƒก. แƒ˜แƒกแƒ˜แƒœแƒ˜ แƒกแƒแƒฃแƒ‘แƒ แƒแƒ‘แƒ”แƒœ แƒ˜แƒ–แƒแƒšแƒ˜แƒ แƒ”แƒ‘แƒฃแƒš แƒ“แƒ แƒ™แƒแƒ›แƒ‘แƒ˜แƒœแƒ˜แƒ แƒ”แƒ‘แƒฃแƒš แƒฌแƒแƒœแƒแƒกแƒฌแƒแƒ แƒแƒ‘แƒแƒ–แƒ”.

แƒ—แƒฃ แƒ แƒแƒ›แƒ“แƒ”แƒœแƒ˜แƒ›แƒ” แƒ”แƒ แƒ—แƒ“แƒ แƒแƒฃแƒšแƒแƒ“ แƒแƒ แƒกแƒ”แƒ‘แƒฃแƒšแƒ˜ แƒฌแƒแƒœแƒแƒกแƒฌแƒแƒ แƒแƒ‘แƒ แƒ”แƒ แƒ—แƒ›แƒแƒœแƒ”แƒ—แƒ˜แƒกแƒ’แƒแƒœ แƒ“แƒแƒ›แƒแƒฃแƒ™แƒ˜แƒ“แƒ”แƒ‘แƒ”แƒšแƒ˜แƒ, แƒ›แƒแƒ— แƒ˜แƒ–แƒแƒšแƒ˜แƒ แƒ”แƒ‘แƒฃแƒšแƒก แƒฃแƒฌแƒแƒ“แƒ”แƒ‘แƒ”แƒœ. แƒฌแƒแƒœแƒแƒกแƒฌแƒแƒ แƒแƒ‘แƒ˜แƒก แƒชแƒ•แƒšแƒ˜แƒšแƒ”แƒ‘แƒ แƒ”แƒ แƒ— แƒ›แƒแƒ—แƒ’แƒแƒœแƒจแƒ˜ แƒแƒ  แƒ˜แƒฌแƒ•แƒ”แƒ•แƒก แƒฌแƒแƒœแƒแƒกแƒฌแƒแƒ แƒแƒ‘แƒ˜แƒก แƒžแƒแƒ–แƒ˜แƒชแƒ˜แƒ˜แƒก แƒชแƒ•แƒšแƒ˜แƒšแƒ”แƒ‘แƒแƒก แƒ›แƒ”แƒแƒ แƒ”แƒจแƒ˜.

แƒ—แƒฃ แƒ”แƒ แƒ—-แƒ”แƒ แƒ— แƒ›แƒแƒ—แƒ’แƒแƒœแƒจแƒ˜ แƒฌแƒแƒœแƒแƒกแƒฌแƒแƒ แƒแƒ‘แƒ˜แƒก แƒชแƒ•แƒšแƒ˜แƒšแƒ”แƒ‘แƒ แƒ˜แƒฌแƒ•แƒ”แƒ•แƒก แƒฌแƒแƒœแƒแƒกแƒฌแƒแƒ แƒแƒ‘แƒ˜แƒก แƒชแƒ•แƒšแƒ˜แƒšแƒ”แƒ‘แƒแƒก แƒ›แƒ”แƒแƒ แƒ”แƒจแƒ˜, แƒ›แƒแƒจแƒ˜แƒœ แƒฉแƒ•แƒ”แƒœ แƒ•แƒกแƒแƒฃแƒ‘แƒ แƒแƒ‘แƒ— แƒ™แƒแƒ›แƒ‘แƒ˜แƒœแƒ˜แƒ แƒ”แƒ‘แƒฃแƒš (แƒ™แƒแƒœแƒ˜แƒฃแƒ’แƒแƒขแƒ”แƒ‘แƒฃแƒš, แƒ™แƒแƒœแƒ™แƒฃแƒ แƒ”แƒœแƒขแƒฃแƒš) แƒฌแƒแƒœแƒแƒกแƒฌแƒแƒ แƒแƒ‘แƒแƒ–แƒ”. แƒ™แƒแƒ›แƒ‘แƒ˜แƒœแƒ˜แƒ แƒ”แƒ‘แƒฃแƒšแƒ˜ แƒฌแƒแƒœแƒแƒกแƒฌแƒแƒ แƒแƒ‘แƒ˜แƒก แƒ›แƒฅแƒแƒœแƒ” แƒกแƒ˜แƒกแƒขแƒ”แƒ›แƒ”แƒ‘แƒจแƒ˜ แƒ“แƒแƒ›แƒ˜แƒœแƒแƒœแƒขแƒฃแƒ แƒ˜ แƒžแƒ แƒแƒชแƒ”แƒกแƒ˜แƒ แƒ˜แƒก, แƒ แƒแƒ›แƒ”แƒšแƒ˜แƒช แƒฎแƒแƒกแƒ˜แƒแƒ—แƒ“แƒ”แƒ‘แƒ แƒฌแƒแƒœแƒแƒกแƒฌแƒแƒ แƒแƒ‘แƒ˜แƒก แƒ›แƒฃแƒ“แƒ›แƒ˜แƒ•แƒ˜แƒก แƒฃแƒคแƒ แƒ แƒ“แƒ˜แƒ“แƒ˜ แƒ›แƒœแƒ˜แƒจแƒ•แƒœแƒ”แƒšแƒแƒ‘แƒ˜แƒ—.

แƒ›แƒ”แƒแƒ แƒ” แƒžแƒ แƒแƒชแƒ”แƒกแƒ˜ แƒ˜แƒฅแƒœแƒ”แƒ‘แƒ แƒฃแƒžแƒ˜แƒ แƒแƒขแƒ”แƒกแƒ˜, แƒ แƒแƒ“แƒ’แƒแƒœ แƒ›แƒ˜แƒกแƒ˜ แƒฌแƒแƒœแƒแƒกแƒฌแƒแƒ แƒแƒ‘แƒ˜แƒก แƒ›แƒฃแƒ“แƒ›แƒ˜แƒ•แƒ˜ แƒฃแƒคแƒ แƒ แƒ›แƒ”แƒขแƒ˜แƒ แƒ•แƒ˜แƒ“แƒ แƒ” แƒžแƒ˜แƒ แƒ•แƒ”แƒšแƒ˜ แƒžแƒ แƒแƒชแƒ”แƒกแƒ˜แƒก แƒฌแƒแƒœแƒแƒกแƒฌแƒแƒ แƒแƒ‘แƒ˜แƒก แƒ›แƒฃแƒ“แƒ›แƒ˜แƒ•แƒ˜. แƒฌแƒแƒœแƒแƒกแƒฌแƒแƒ แƒแƒ‘แƒ แƒ›แƒ”แƒแƒ แƒ” แƒžแƒ แƒแƒชแƒ”แƒกแƒจแƒ˜ แƒฃแƒคแƒ แƒ แƒ›แƒ”แƒขแƒแƒ“ แƒ›แƒแƒ แƒฏแƒ•แƒœแƒ˜แƒ• แƒแƒ แƒ˜แƒก แƒ’แƒแƒ“แƒแƒขแƒแƒœแƒ˜แƒšแƒ˜, แƒ แƒแƒ“แƒ’แƒแƒœ แƒ›แƒ”แƒ—แƒ˜แƒšแƒแƒ›แƒ˜แƒœแƒ˜ แƒฃแƒคแƒ แƒ แƒซแƒšแƒ˜แƒ”แƒ แƒ˜ แƒ‘แƒแƒ–แƒแƒ แƒ•แƒ˜แƒ“แƒ แƒ” แƒแƒ›แƒ˜แƒแƒ™แƒ˜, NH 4 + แƒฃแƒคแƒ แƒ แƒซแƒšแƒ˜แƒ”แƒ แƒ˜ แƒ›แƒŸแƒแƒ•แƒแƒ แƒ•แƒ˜แƒ“แƒ แƒ” CH 3 NH 3 +.

แƒ“แƒแƒกแƒ™แƒ•แƒœแƒ: แƒฃแƒคแƒ แƒ แƒซแƒšแƒ˜แƒ”แƒ แƒ˜ แƒคแƒฃแƒซแƒ” แƒ—แƒ แƒ’แƒฃแƒœแƒแƒ•แƒก แƒกแƒฃแƒกแƒขแƒ˜ แƒคแƒฃแƒซแƒ˜แƒก แƒ˜แƒแƒœแƒ˜แƒ–แƒแƒชแƒ˜แƒแƒก. แƒแƒ›แƒ˜แƒขแƒแƒ›, แƒ แƒแƒ“แƒ”แƒกแƒแƒช แƒแƒ›แƒ˜แƒแƒ™แƒ˜แƒกแƒ แƒ“แƒ แƒ›แƒ”แƒ—แƒ˜แƒšแƒแƒ›แƒ˜แƒœแƒ˜แƒก แƒœแƒแƒ แƒ”แƒ•แƒก แƒ”แƒ›แƒแƒขแƒ”แƒ‘แƒ แƒ›แƒชแƒ˜แƒ แƒ” แƒ แƒแƒแƒ“แƒ”แƒœแƒแƒ‘แƒ˜แƒ— แƒ›แƒแƒ แƒ˜แƒšแƒ›แƒŸแƒแƒ•แƒ, แƒ›แƒ”แƒ—แƒ˜แƒšแƒแƒ›แƒ˜แƒœแƒ˜ แƒซแƒ˜แƒ แƒ˜แƒ—แƒแƒ“แƒแƒ“ แƒžแƒ แƒแƒขแƒแƒœแƒแƒชแƒ˜แƒแƒก แƒ’แƒแƒœแƒ˜แƒชแƒ“แƒ˜แƒก.

แƒ“แƒ แƒแƒกแƒ”แƒ•แƒ”: แƒฃแƒซแƒšแƒ˜แƒ”แƒ แƒ”แƒกแƒ˜ แƒ›แƒŸแƒแƒ•แƒ แƒ—แƒ แƒ’แƒฃแƒœแƒแƒ•แƒก แƒกแƒฃแƒกแƒขแƒ˜ แƒ›แƒŸแƒแƒ•แƒ”แƒ‘แƒ˜แƒก แƒ˜แƒแƒœแƒ˜แƒ–แƒแƒชแƒ˜แƒแƒก. แƒแƒกแƒ” แƒ แƒแƒ›, แƒ›แƒแƒ แƒ˜แƒšแƒ›แƒŸแƒแƒ•แƒ, แƒ แƒแƒ›แƒ”แƒšแƒ˜แƒช แƒ’แƒ•แƒฎแƒ•แƒ“แƒ”แƒ‘แƒ แƒ™แƒฃแƒญแƒ˜แƒก แƒฌแƒ•แƒ”แƒœแƒจแƒ˜, แƒแƒคแƒ”แƒ แƒฎแƒ”แƒ‘แƒก แƒซแƒ›แƒแƒ แƒ›แƒŸแƒแƒ•แƒแƒก (แƒ›แƒแƒ“แƒ˜แƒก แƒกแƒแƒ™แƒ•แƒ”แƒ‘แƒ—แƒแƒœ แƒ”แƒ แƒ—แƒแƒ“) แƒแƒœ แƒแƒชแƒ”แƒขแƒ˜แƒšแƒกแƒแƒšแƒ˜แƒชแƒ˜แƒšแƒ˜แƒก แƒ›แƒŸแƒแƒ•แƒแƒก (แƒฌแƒแƒ›แƒแƒšแƒ˜) แƒ˜แƒแƒœแƒ˜แƒ–แƒแƒชแƒ˜แƒแƒก.

______________________________________________________________

แƒแƒ›แƒ แƒ˜แƒ’แƒแƒ“, แƒแƒ› แƒ—แƒ”แƒแƒ แƒ˜แƒ˜แƒก แƒ›แƒ˜แƒฎแƒ”แƒ“แƒ•แƒ˜แƒ— แƒ›แƒŸแƒแƒ•แƒ แƒแƒ แƒ˜แƒก แƒœแƒ”แƒ‘แƒ˜แƒกแƒ›แƒ˜แƒ”แƒ แƒ˜ แƒœแƒ˜แƒ•แƒ—แƒ˜แƒ”แƒ แƒ”แƒ‘แƒ, แƒ แƒแƒ›แƒšแƒ˜แƒก แƒ›แƒแƒšแƒ”แƒ™แƒฃแƒšแƒ”แƒ‘แƒก (แƒ˜แƒแƒœแƒ”แƒ‘แƒ˜แƒก แƒฉแƒแƒ—แƒ•แƒšแƒ˜แƒ—) แƒจแƒ”แƒฃแƒซแƒšแƒ˜แƒแƒ— แƒžแƒ แƒแƒขแƒแƒœแƒ˜แƒก แƒ“แƒแƒœแƒแƒชแƒ˜แƒ, แƒ”.แƒ˜. แƒ˜แƒงแƒแƒ•แƒ˜ แƒžแƒ แƒแƒขแƒแƒœแƒ˜แƒก แƒ“แƒแƒœแƒแƒ แƒ˜; แƒคแƒฃแƒซแƒ” แƒแƒ แƒ˜แƒก แƒœแƒ”แƒ‘แƒ˜แƒกแƒ›แƒ˜แƒ”แƒ แƒ˜ แƒœแƒ˜แƒ•แƒ—แƒ˜แƒ”แƒ แƒ”แƒ‘แƒ, แƒ แƒแƒ›แƒšแƒ˜แƒก แƒ›แƒแƒšแƒ”แƒ™แƒฃแƒšแƒ”แƒ‘แƒก (แƒ˜แƒแƒœแƒ”แƒ‘แƒ˜แƒก แƒฉแƒแƒ—แƒ•แƒšแƒ˜แƒ—) แƒจแƒ”แƒฃแƒซแƒšแƒ˜แƒแƒ— แƒžแƒ แƒแƒขแƒแƒœแƒ˜แƒก แƒ›แƒ˜แƒ›แƒแƒ’แƒ แƒ”แƒ‘แƒ, แƒ”.แƒ˜. แƒ˜แƒงแƒแƒก แƒžแƒ แƒแƒขแƒแƒœแƒ˜แƒก แƒ›แƒ˜แƒ›แƒฆแƒ”แƒ‘แƒ˜; แƒแƒ›แƒคแƒแƒšแƒ˜แƒขแƒ˜ แƒแƒ แƒ˜แƒก แƒœแƒ”แƒ‘แƒ˜แƒกแƒ›แƒ˜แƒ”แƒ แƒ˜ แƒœแƒ˜แƒ•แƒ—แƒ˜แƒ”แƒ แƒ”แƒ‘แƒ, แƒ แƒแƒ›แƒ”แƒšแƒ˜แƒช แƒแƒ แƒ˜แƒก แƒžแƒ แƒแƒขแƒแƒœแƒ”แƒ‘แƒ˜แƒก แƒ“แƒแƒœแƒแƒ แƒ˜แƒช แƒ“แƒ แƒ›แƒ˜แƒ›แƒฆแƒ”แƒ‘แƒ˜แƒช.

แƒ”แƒก แƒ—แƒ”แƒแƒ แƒ˜แƒ แƒฎแƒกแƒœแƒ˜แƒก แƒแƒ แƒ แƒ›แƒฎแƒแƒšแƒแƒ“ แƒœแƒ”แƒ˜แƒขแƒ แƒแƒšแƒฃแƒ แƒ˜ แƒ›แƒแƒšแƒ”แƒ™แƒฃแƒšแƒ”แƒ‘แƒ˜แƒก, แƒแƒ แƒแƒ›แƒ”แƒ“ แƒ˜แƒแƒœแƒ”แƒ‘แƒ˜แƒก แƒ›แƒŸแƒแƒ•แƒ-แƒขแƒฃแƒขแƒแƒ•แƒแƒœ แƒ—แƒ•แƒ˜แƒกแƒ”แƒ‘แƒ”แƒ‘แƒก. แƒ›แƒŸแƒแƒ•แƒ, แƒ แƒแƒ›แƒ”แƒšแƒ˜แƒช แƒฉแƒฃแƒฅแƒœแƒ˜แƒก แƒžแƒ แƒแƒขแƒแƒœแƒก, แƒ˜แƒฅแƒชแƒ”แƒ•แƒ แƒคแƒฃแƒซแƒ”แƒ“, แƒ แƒแƒ›แƒ”แƒšแƒ˜แƒช แƒจแƒ”แƒ แƒฌแƒงแƒ›แƒฃแƒšแƒ˜แƒ แƒแƒ› แƒ›แƒŸแƒแƒ•แƒแƒกแƒ—แƒแƒœ. "แƒ›แƒŸแƒแƒ•แƒ" แƒ“แƒ "แƒ‘แƒแƒ–แƒ˜แƒก" แƒชแƒœแƒ”แƒ‘แƒ”แƒ‘แƒ˜ แƒคแƒแƒ แƒ“แƒแƒ‘แƒ˜แƒ—แƒ˜ แƒชแƒœแƒ”แƒ‘แƒ”แƒ‘แƒ˜แƒ, แƒ แƒแƒ“แƒ’แƒแƒœ แƒ”แƒ แƒ—แƒกแƒ แƒ“แƒ แƒ˜แƒ›แƒแƒ•แƒ” แƒœแƒแƒฌแƒ˜แƒšแƒแƒ™แƒ”แƒ‘แƒก - แƒ›แƒแƒšแƒ”แƒ™แƒฃแƒšแƒ”แƒ‘แƒก แƒแƒœ แƒ˜แƒแƒœแƒ”แƒ‘แƒก - แƒจแƒ”แƒฃแƒซแƒšแƒ˜แƒแƒ— แƒ’แƒแƒ›แƒแƒแƒ•แƒšแƒ˜แƒœแƒแƒœ แƒ แƒแƒ’แƒแƒ แƒช แƒซแƒ˜แƒ แƒ˜แƒ—แƒแƒ“แƒ˜, แƒแƒกแƒ”แƒ•แƒ” แƒ›แƒŸแƒแƒ•แƒ” แƒ—แƒ•แƒ˜แƒกแƒ”แƒ‘แƒ”แƒ‘แƒ˜, แƒ”แƒก แƒ“แƒแƒ›แƒแƒ™แƒ˜แƒ“แƒ”แƒ‘แƒฃแƒšแƒ˜แƒ แƒžแƒแƒ แƒขแƒœแƒ˜แƒแƒ แƒ–แƒ”.

แƒžแƒ แƒแƒขแƒแƒšแƒ˜แƒ–แƒฃแƒ  แƒฌแƒแƒœแƒแƒกแƒฌแƒแƒ แƒแƒ‘แƒแƒจแƒ˜ แƒฌแƒแƒ แƒ›แƒแƒ˜แƒฅแƒ›แƒœแƒ”แƒ‘แƒ แƒ›แƒŸแƒแƒ•แƒ-แƒขแƒฃแƒขแƒแƒ•แƒแƒœแƒ˜ แƒฌแƒงแƒ•แƒ˜แƒšแƒ˜. แƒžแƒ แƒแƒขแƒแƒœแƒ˜แƒก แƒ—แƒ”แƒแƒ แƒ˜แƒ˜แƒก แƒ›แƒ˜แƒฎแƒ”แƒ“แƒ•แƒ˜แƒ—, แƒฐแƒ˜แƒ“แƒ แƒแƒšแƒ˜แƒ–แƒ˜แƒก, แƒ˜แƒแƒœแƒ˜แƒ–แƒแƒชแƒ˜แƒ˜แƒกแƒ แƒ“แƒ แƒœแƒ”แƒ˜แƒขแƒ แƒแƒšแƒ˜แƒ–แƒแƒชแƒ˜แƒ˜แƒก แƒ แƒ”แƒแƒฅแƒชแƒ˜แƒ”แƒ‘แƒ˜ แƒแƒ  แƒ’แƒแƒœแƒ˜แƒฎแƒ˜แƒšแƒ”แƒ‘แƒ แƒ แƒแƒ’แƒแƒ แƒช แƒ’แƒแƒœแƒกแƒแƒ™แƒฃแƒ—แƒ แƒ”แƒ‘แƒฃแƒšแƒ˜ แƒคแƒ”แƒœแƒแƒ›แƒ”แƒœแƒ˜, แƒแƒ แƒแƒ›แƒ”แƒ“ แƒ’แƒแƒœแƒ˜แƒฎแƒ˜แƒšแƒ”แƒ‘แƒ แƒžแƒ แƒแƒขแƒแƒœแƒ”แƒ‘แƒ˜แƒก แƒฉแƒ•แƒ”แƒฃแƒšแƒ”แƒ‘แƒ แƒ˜แƒ• แƒ’แƒแƒ“แƒแƒกแƒ•แƒšแƒแƒ“ แƒ›แƒŸแƒแƒ•แƒ˜แƒ“แƒแƒœ แƒคแƒฃแƒซแƒ”แƒ–แƒ”.

แƒœแƒแƒฌแƒ˜แƒšแƒแƒ™แƒ˜ A แƒฌแƒแƒ แƒ›แƒแƒ˜แƒฅแƒ›แƒœแƒ”แƒ‘แƒ แƒฌแƒงแƒแƒšแƒ‘แƒแƒ“แƒ˜แƒก แƒ˜แƒแƒœแƒ˜แƒก แƒ’แƒแƒ›แƒแƒงแƒแƒคแƒ˜แƒก แƒจแƒ”แƒ›แƒ“แƒ”แƒ’

แƒ”แƒฌแƒแƒ“แƒ”แƒ‘แƒ แƒ›แƒแƒชแƒ”แƒ›แƒฃแƒš แƒ›แƒŸแƒแƒ•แƒแƒกแƒ—แƒแƒœ แƒจแƒ”แƒ”แƒ แƒ—แƒ”แƒ‘แƒฃแƒš แƒคแƒฃแƒซแƒ”แƒก, แƒ แƒแƒ“แƒ’แƒแƒœ. แƒ›แƒแƒก แƒจแƒ”แƒฃแƒซแƒšแƒ˜แƒ H + แƒ˜แƒแƒœแƒ˜แƒก แƒฎแƒ”แƒšแƒแƒฎแƒšแƒ แƒ›แƒ˜แƒ‘แƒ›แƒ แƒกแƒแƒ™แƒฃแƒ—แƒแƒ  แƒ—แƒแƒ•แƒก.
แƒžแƒ แƒแƒขแƒแƒšแƒ˜แƒ–แƒฃแƒ แƒ˜ แƒ—แƒ”แƒแƒ แƒ˜แƒ˜แƒก แƒ›แƒ˜แƒฎแƒ”แƒ“แƒ•แƒ˜แƒ—, แƒ›แƒŸแƒแƒ•แƒ”แƒ‘แƒ˜ แƒ“แƒ แƒคแƒฃแƒซแƒ”แƒ”แƒ‘แƒ˜ แƒจแƒ”แƒ˜แƒซแƒšแƒ”แƒ‘แƒ แƒ˜แƒงแƒแƒก แƒกแƒแƒ›แƒ˜ แƒกแƒแƒฎแƒ˜แƒก: แƒœแƒ”แƒ˜แƒขแƒ แƒแƒšแƒฃแƒ แƒ˜, แƒแƒœแƒ˜แƒแƒœแƒฃแƒ แƒ˜ แƒ“แƒ แƒ™แƒแƒขแƒ˜แƒแƒœแƒฃแƒ แƒ˜. แƒžแƒ˜แƒ แƒ•แƒ”แƒšแƒ˜แƒก แƒ แƒแƒšแƒก แƒแƒกแƒ แƒฃแƒšแƒ”แƒ‘แƒ”แƒœ แƒœแƒ”แƒ˜แƒขแƒ แƒแƒšแƒฃแƒ แƒ˜ แƒ›แƒแƒšแƒ”แƒ™แƒฃแƒšแƒ”แƒ‘แƒ˜, แƒ แƒแƒ›แƒšแƒ”แƒ‘แƒกแƒแƒช แƒจแƒ”แƒฃแƒซแƒšแƒ˜แƒแƒ— H + แƒ˜แƒแƒœแƒ˜แƒก แƒ›แƒ˜แƒชแƒ”แƒ›แƒ แƒแƒœ แƒ›แƒ˜แƒ›แƒแƒ’แƒ แƒ”แƒ‘แƒ, แƒ›แƒแƒ’แƒแƒšแƒ˜แƒ—แƒแƒ“: HCl, H 2 SO 4, HNO 3 (แƒ›แƒŸแƒแƒ•แƒ”แƒ‘แƒ˜); NH 3, CH 3 -O-CH 3 (แƒ‘แƒแƒ–แƒ”แƒ‘แƒ˜). แƒแƒœแƒ˜แƒแƒœแƒฃแƒ แƒ˜ แƒคแƒฃแƒซแƒ”แƒ”แƒ‘แƒ˜ แƒ“แƒ แƒ›แƒŸแƒแƒ•แƒ”แƒ‘แƒ˜แƒแƒ แƒ˜แƒก แƒฃแƒแƒ แƒงแƒแƒคแƒ˜แƒ—แƒแƒ“ แƒ“แƒแƒ›แƒฃแƒฎแƒขแƒฃแƒšแƒ˜ แƒ˜แƒแƒœแƒ”แƒ‘แƒ˜, แƒ›แƒแƒ’แƒแƒšแƒ˜แƒ—แƒแƒ“: HSO 4 -, HPO 4 2-, HS - (แƒ›แƒŸแƒแƒ•แƒ”แƒ‘แƒ˜); OH -, Cl -, NO 3 - (แƒ‘แƒแƒ–แƒ”แƒ‘แƒ˜). แƒ แƒแƒšแƒ”แƒ‘แƒจแƒ˜ แƒ™แƒแƒ—แƒ˜แƒแƒœแƒฃแƒ แƒ˜ แƒคแƒฃแƒซแƒ”แƒ”แƒ‘แƒ˜ แƒ“แƒ แƒ›แƒŸแƒแƒ•แƒ”แƒ‘แƒ˜แƒ“แƒแƒ“แƒ”แƒ‘แƒ˜แƒ—แƒแƒ“ แƒ“แƒแƒ›แƒฃแƒฎแƒขแƒฃแƒšแƒ˜ แƒ˜แƒแƒœแƒ”แƒ‘แƒ˜ แƒ›แƒแƒฅแƒ›แƒ”แƒ“แƒ”แƒ‘แƒ”แƒœ, แƒ›แƒแƒ’แƒแƒšแƒ˜แƒ—แƒแƒ“: NH 4 +, H 3 O + (แƒ›แƒŸแƒแƒ•แƒ”แƒ‘แƒ˜); H 2 Nโ€“NH 3 +, H 2 Nโ€“ (CH 2) 2 โ€“NH 3 + (แƒ‘แƒแƒ–แƒ”แƒ‘แƒ˜). แƒ‘แƒ”แƒ•แƒ  แƒœแƒแƒฌแƒ˜แƒšแƒแƒ™แƒก (แƒ แƒแƒ’แƒแƒ แƒช แƒ›แƒแƒšแƒ”แƒ™แƒฃแƒšแƒ”แƒ‘แƒก, แƒแƒกแƒ”แƒ•แƒ” แƒ˜แƒแƒœแƒ”แƒ‘แƒก) แƒ’แƒแƒแƒฉแƒœแƒ˜แƒ แƒแƒ›แƒคแƒแƒขแƒ”แƒ แƒฃแƒšแƒ˜ แƒ—แƒ•แƒ˜แƒกแƒ”แƒ‘แƒ”แƒ‘แƒ˜, แƒ”.แƒ˜. แƒžแƒ˜แƒ แƒแƒ‘แƒ”แƒ‘แƒ˜แƒ“แƒแƒœ แƒ’แƒแƒ›แƒแƒ›แƒ“แƒ˜แƒœแƒแƒ แƒ”, แƒ›แƒแƒ— แƒจแƒ”แƒฃแƒซแƒšแƒ˜แƒแƒ— แƒ˜แƒ›แƒแƒฅแƒ›แƒ”แƒ“แƒแƒœ แƒ แƒแƒ’แƒแƒ แƒช แƒ›แƒŸแƒแƒ•แƒ, แƒแƒกแƒ”แƒ•แƒ” แƒ‘แƒแƒ–แƒ˜แƒก แƒกแƒแƒฎแƒ˜แƒ—, แƒ›แƒแƒ’แƒแƒšแƒ˜แƒ—แƒแƒ“: H 2 O, NH 3, HSO 4 -, H 2 Nโ€“NH 3 + แƒ“แƒ แƒ.แƒจ. แƒแƒ› แƒœแƒแƒ”แƒ แƒ—แƒ”แƒ‘แƒก แƒแƒ›แƒคแƒ˜แƒžแƒ แƒแƒขแƒ˜แƒ™แƒ˜ แƒแƒœ แƒแƒ›แƒคแƒแƒšแƒ˜แƒขแƒ”แƒ‘แƒ˜ แƒ”แƒฌแƒแƒ“แƒ”แƒ‘แƒ. แƒ›แƒ˜แƒฃแƒฎแƒ”แƒ“แƒแƒ•แƒแƒ“ แƒ˜แƒ›แƒ˜แƒกแƒ, แƒ แƒแƒ› แƒ‘แƒ แƒแƒœแƒกแƒขแƒ”แƒ“-แƒšแƒแƒฃแƒ แƒ˜แƒก แƒ—แƒ”แƒแƒ แƒ˜แƒ แƒฃแƒคแƒ แƒ แƒกแƒ แƒฃแƒšแƒงแƒแƒคแƒ˜แƒšแƒ˜แƒ แƒ•แƒ˜แƒ“แƒ แƒ” แƒแƒ แƒ”แƒœแƒ˜แƒฃแƒกแƒ˜แƒก แƒ—แƒ”แƒแƒ แƒ˜แƒ, แƒ›แƒแƒก แƒแƒกแƒ”แƒ•แƒ” แƒแƒฅแƒ•แƒก แƒ’แƒแƒ แƒ™แƒ•แƒ”แƒฃแƒšแƒ˜ แƒœแƒแƒ™แƒšแƒแƒ•แƒแƒœแƒ”แƒ‘แƒ”แƒ‘แƒ˜ แƒ“แƒ แƒแƒ  แƒแƒ แƒ˜แƒก แƒงแƒแƒ•แƒšแƒ˜แƒกแƒ›แƒแƒ›แƒชแƒ•แƒ”แƒšแƒ˜. แƒแƒกแƒ” แƒ แƒแƒ›, แƒ˜แƒก แƒแƒ  แƒ’แƒแƒ›แƒแƒ˜แƒงแƒ”แƒœแƒ”แƒ‘แƒ แƒ‘แƒ”แƒ•แƒ  แƒœแƒ˜แƒ•แƒ—แƒ˜แƒ”แƒ แƒ”แƒ‘แƒแƒ–แƒ”, แƒ แƒแƒ›แƒšแƒ”แƒ‘แƒ˜แƒช แƒแƒ•แƒšแƒ”แƒœแƒ”แƒœ แƒ›แƒŸแƒแƒ•แƒแƒก แƒคแƒฃแƒœแƒฅแƒชแƒ˜แƒแƒก, แƒ›แƒแƒ’แƒ แƒแƒ› แƒแƒ  แƒจแƒ”แƒ˜แƒชแƒแƒ•แƒก H + แƒ˜แƒแƒœแƒ”แƒ‘แƒก แƒ›แƒแƒ— แƒจแƒ”แƒ›แƒแƒ“แƒ’แƒ”แƒœแƒšแƒแƒ‘แƒแƒจแƒ˜, แƒ›แƒแƒ’แƒแƒšแƒ˜แƒ—แƒแƒ“: BCl 3, AlCl 3, BF 3, FeCl 3 แƒ“แƒ แƒ.แƒจ.

แƒกแƒแƒ“แƒแƒช: K a โ€“ แƒ›แƒŸแƒแƒ•แƒ˜แƒแƒœแƒแƒ‘แƒ˜แƒก แƒ›แƒฃแƒ“แƒ›แƒ˜แƒ•แƒ˜; K p แƒแƒ แƒ˜แƒก แƒฌแƒแƒœแƒแƒกแƒฌแƒแƒ แƒแƒ‘แƒ˜แƒก แƒ›แƒฃแƒ“แƒ›แƒ˜แƒ•แƒ˜.

แƒ˜แƒฅ แƒ›แƒŸแƒแƒ•แƒ แƒฃแƒคแƒ แƒ แƒซแƒšแƒ˜แƒ”แƒ แƒ˜แƒ, แƒ›แƒ˜แƒ— แƒฃแƒคแƒ แƒ แƒ“แƒ˜แƒ“แƒ˜แƒ แƒ›แƒŸแƒแƒ•แƒ˜แƒแƒœแƒแƒ‘แƒ˜แƒก แƒ›แƒฃแƒ“แƒ›แƒ˜แƒ•แƒ˜. แƒฎแƒจแƒ˜แƒ แƒแƒ“ แƒ’แƒแƒ›แƒแƒ˜แƒงแƒ”แƒœแƒ”แƒ‘แƒ pKa แƒ›แƒœแƒ˜แƒจแƒ•แƒœแƒ”แƒšแƒแƒ‘แƒ”แƒ‘แƒ˜. แƒ แƒแƒช แƒฃแƒคแƒ แƒ แƒ›แƒชแƒ˜แƒ แƒ”แƒ pKa แƒ›แƒœแƒ˜แƒจแƒ•แƒœแƒ”แƒšแƒแƒ‘แƒ, แƒ›แƒ˜แƒ— แƒฃแƒคแƒ แƒ แƒซแƒšแƒ˜แƒ”แƒ แƒ˜แƒ แƒ›แƒŸแƒแƒ•แƒ.

pK a = -lgK แƒ

แƒ›แƒแƒ’แƒแƒšแƒ˜แƒ—แƒแƒ“, แƒคแƒ”แƒœแƒแƒšแƒ˜แƒก pK a = 10, แƒ”แƒ—แƒแƒœแƒแƒšแƒ˜แƒก pK a = 16. แƒ”แƒก แƒœแƒ˜แƒจแƒœแƒแƒ•แƒก, แƒ แƒแƒ› แƒคแƒ”แƒœแƒแƒšแƒ˜ แƒกแƒ˜แƒ“แƒ˜แƒ“แƒ˜แƒก แƒ”แƒฅแƒ•แƒกแƒ˜ แƒ แƒ˜แƒ’แƒ˜แƒ— (แƒ›แƒ˜แƒšแƒ˜แƒแƒœแƒฏแƒ”แƒ ) แƒฃแƒคแƒ แƒ แƒซแƒšแƒ˜แƒ”แƒ แƒ˜ แƒ›แƒŸแƒแƒ•แƒแƒ, แƒ•แƒ˜แƒ“แƒ แƒ” แƒ”แƒ—แƒ˜แƒšแƒ˜แƒก แƒกแƒžแƒ˜แƒ แƒขแƒ˜.

แƒกแƒแƒคแƒฃแƒซแƒ•แƒšแƒ˜แƒแƒœแƒแƒ‘แƒ แƒจแƒ”แƒ˜แƒซแƒšแƒ”แƒ‘แƒ แƒ’แƒแƒ›แƒแƒ˜แƒฎแƒแƒขแƒแƒก pK b-แƒ˜แƒ—.

RKb = 14 - pKแƒ

แƒ›แƒœแƒ˜แƒจแƒ•แƒœแƒ”แƒšแƒแƒ•แƒแƒœแƒ˜แƒ แƒ’แƒ•แƒแƒฎแƒกแƒแƒ•แƒ“แƒ”แƒก, แƒ แƒแƒ› แƒฌแƒงแƒšแƒ˜แƒก pKa แƒแƒ แƒ˜แƒก 15.7. แƒงแƒ•แƒ”แƒšแƒ แƒœแƒ˜แƒ•แƒ—แƒ˜แƒ”แƒ แƒ”แƒ‘แƒแƒก, แƒ แƒแƒ›แƒ”แƒšแƒกแƒแƒช แƒแƒฅแƒ•แƒก pKa แƒฌแƒงแƒแƒšแƒ–แƒ” แƒ›แƒ”แƒขแƒ˜, แƒแƒ  แƒจแƒ”แƒฃแƒซแƒšแƒ˜แƒ แƒ’แƒแƒ›แƒแƒแƒ•แƒšแƒ˜แƒœแƒแƒก แƒ›แƒŸแƒแƒ•แƒ” แƒ—แƒ•แƒ˜แƒกแƒ”แƒ‘แƒ”แƒ‘แƒ˜ แƒฌแƒงแƒแƒšแƒฎแƒกแƒœแƒแƒ แƒ”แƒ‘แƒจแƒ˜. แƒฌแƒงแƒแƒšแƒ˜, แƒ แƒแƒ’แƒแƒ แƒช แƒฃแƒคแƒ แƒ แƒซแƒšแƒ˜แƒ”แƒ แƒ˜ แƒ›แƒŸแƒแƒ•แƒ, แƒแƒคแƒ”แƒ แƒฎแƒ”แƒ‘แƒก แƒกแƒฃแƒกแƒขแƒ˜ แƒ›แƒŸแƒแƒ•แƒ”แƒ‘แƒ˜แƒก แƒ“แƒ˜แƒกแƒแƒชแƒ˜แƒแƒชแƒ˜แƒแƒก. แƒ•แƒ˜แƒœแƒแƒ˜แƒ“แƒแƒœ แƒแƒ แƒ’แƒแƒœแƒฃแƒšแƒ˜ แƒœแƒแƒ”แƒ แƒ—แƒ”แƒ‘แƒ˜แƒก แƒฃแƒ›แƒ”แƒขแƒ”แƒกแƒแƒ‘แƒ˜แƒก แƒ›แƒŸแƒแƒ•แƒ” แƒ—แƒ•แƒ˜แƒกแƒ”แƒ‘แƒ”แƒ‘แƒ˜ แƒ‘แƒ”แƒ•แƒ แƒฏแƒ”แƒ  แƒกแƒฃแƒกแƒขแƒ˜แƒ, แƒ•แƒ˜แƒ“แƒ แƒ” แƒฌแƒงแƒšแƒ˜แƒกแƒ, แƒจแƒ”แƒ›แƒฃแƒจแƒแƒ•แƒ”แƒ‘แƒฃแƒšแƒ˜แƒ แƒžแƒแƒšแƒแƒ แƒแƒ’แƒ แƒแƒคแƒ˜แƒฃแƒšแƒ˜ แƒ›แƒ˜แƒ“แƒ’แƒแƒ›แƒ แƒ›แƒแƒ—แƒ˜ แƒ›แƒŸแƒแƒ•แƒ˜แƒแƒœแƒแƒ‘แƒ˜แƒก แƒจแƒ”แƒกแƒแƒคแƒแƒกแƒ”แƒ‘แƒšแƒแƒ“ (I.P. Beletskaya et al.). แƒ˜แƒก แƒ˜แƒซแƒšแƒ”แƒ•แƒ แƒ›แƒŸแƒแƒ•แƒ˜แƒแƒœแƒแƒ‘แƒ˜แƒก แƒจแƒ”แƒคแƒแƒกแƒ”แƒ‘แƒแƒก pKa = 50-แƒ›แƒ“แƒ”, แƒ—แƒฃแƒ›แƒชแƒ แƒซแƒแƒšแƒ˜แƒแƒœ แƒกแƒฃแƒกแƒขแƒ˜ แƒ›แƒŸแƒแƒ•แƒ”แƒ‘แƒ˜แƒกแƒ—แƒ•แƒ˜แƒก pKa แƒ›แƒœแƒ˜แƒจแƒ•แƒœแƒ”แƒšแƒแƒ‘แƒ”แƒ‘แƒ˜ แƒจแƒ”แƒ˜แƒซแƒšแƒ”แƒ‘แƒ แƒจแƒ”แƒคแƒแƒกแƒ“แƒ”แƒก แƒ›แƒฎแƒแƒšแƒแƒ“ แƒซแƒแƒšแƒ˜แƒแƒœ แƒฃแƒฎแƒ”แƒจแƒแƒ“.

แƒ›แƒŸแƒแƒ•แƒ˜แƒแƒœแƒแƒ‘แƒ˜แƒก แƒฎแƒแƒ แƒ˜แƒกแƒฎแƒแƒ‘แƒ แƒ˜แƒ•แƒ˜ แƒจแƒ”แƒคแƒแƒกแƒ”แƒ‘แƒ แƒซแƒแƒšแƒ–แƒ” แƒ›แƒœแƒ˜แƒจแƒ•แƒœแƒ”แƒšแƒแƒ•แƒแƒœแƒ˜แƒ แƒ แƒแƒ’แƒแƒ แƒช แƒกแƒขแƒ แƒฃแƒฅแƒขแƒฃแƒ แƒ˜แƒก แƒ›แƒกแƒ’แƒแƒ•แƒกแƒ˜ แƒœแƒ˜แƒ•แƒ—แƒ˜แƒ”แƒ แƒ”แƒ‘แƒ”แƒ‘แƒ˜แƒก แƒกแƒ”แƒ แƒ˜แƒแƒจแƒ˜, แƒแƒกแƒ”แƒ•แƒ” แƒกแƒฎแƒ•แƒแƒ“แƒแƒกแƒฎแƒ•แƒ แƒ™แƒšแƒแƒกแƒ˜แƒก แƒœแƒแƒ”แƒ แƒ—แƒ”แƒ‘แƒ˜แƒกแƒ—แƒ•แƒ˜แƒก. แƒ›แƒŸแƒแƒ•แƒ˜แƒก แƒฃแƒœแƒแƒ แƒ˜ แƒžแƒ แƒแƒขแƒแƒœแƒ˜แƒก แƒจแƒ”แƒ›แƒแƒฌแƒ˜แƒ แƒฃแƒšแƒแƒ‘แƒแƒกแƒ—แƒแƒœ แƒแƒ แƒ˜แƒก แƒ“แƒแƒ™แƒแƒ•แƒจแƒ˜แƒ แƒ”แƒ‘แƒฃแƒšแƒ˜ แƒ›แƒ˜แƒฆแƒ”แƒ‘แƒฃแƒšแƒ˜ แƒแƒœแƒ˜แƒแƒœแƒ˜แƒก แƒกแƒขแƒแƒ‘แƒ˜แƒšแƒฃแƒ แƒแƒ‘แƒแƒกแƒ—แƒแƒœ. แƒ แƒแƒช แƒฃแƒคแƒ แƒ แƒกแƒขแƒแƒ‘แƒ˜แƒšแƒฃแƒ แƒ˜แƒ แƒ›แƒ˜แƒฆแƒ”แƒ‘แƒฃแƒšแƒ˜ แƒแƒœแƒ˜แƒแƒœแƒ˜, แƒ›แƒ˜แƒ— แƒœแƒแƒ™แƒšแƒ”แƒ‘แƒ˜แƒ แƒ›แƒ˜แƒกแƒ˜ แƒกแƒฃแƒ แƒ•แƒ˜แƒšแƒ˜ แƒ“แƒแƒ˜แƒžแƒงแƒ แƒแƒก แƒžแƒ แƒแƒขแƒแƒœแƒ˜ แƒฃแƒ™แƒแƒœ แƒ“แƒ แƒ’แƒแƒ“แƒแƒ˜แƒฅแƒชแƒ”แƒก แƒœแƒ”แƒ˜แƒขแƒ แƒแƒšแƒฃแƒ  แƒ›แƒแƒšแƒ”แƒ™แƒฃแƒšแƒแƒ“. แƒแƒœแƒ˜แƒแƒœแƒ˜แƒก แƒจแƒ”แƒ“แƒแƒ แƒ”แƒ‘แƒ˜แƒ—แƒ˜ แƒกแƒขแƒแƒ‘แƒ˜แƒšแƒฃแƒ แƒแƒ‘แƒ˜แƒก แƒจแƒ”แƒคแƒแƒกแƒ”แƒ‘แƒ˜แƒกแƒแƒก แƒ›แƒฎแƒ”แƒ“แƒ•แƒ”แƒšแƒแƒ‘แƒแƒจแƒ˜ แƒฃแƒœแƒ“แƒ แƒ˜แƒฅแƒœแƒแƒก แƒ›แƒ˜แƒฆแƒ”แƒ‘แƒฃแƒšแƒ˜ แƒ แƒแƒ›แƒ“แƒ”แƒœแƒ˜แƒ›แƒ” แƒคแƒแƒฅแƒขแƒแƒ แƒ˜.

แƒแƒขแƒแƒ›แƒ˜แƒก แƒ‘แƒฃแƒœแƒ”แƒ‘แƒ, แƒ แƒแƒ›แƒ”แƒšแƒ˜แƒช แƒแƒซแƒšแƒ”แƒ•แƒก แƒžแƒ แƒแƒขแƒแƒœแƒก.แƒ แƒแƒช แƒฃแƒคแƒ แƒ แƒแƒ“แƒ•แƒ˜แƒšแƒแƒ“ แƒ™แƒแƒ แƒ’แƒแƒ•แƒก แƒแƒขแƒแƒ›แƒ˜ แƒžแƒ แƒแƒขแƒแƒœแƒก, แƒ›แƒ˜แƒ— แƒฃแƒคแƒ แƒ แƒ›แƒแƒฆแƒแƒšแƒ˜แƒ แƒ›แƒ˜แƒกแƒ˜ แƒ”แƒšแƒ”แƒฅแƒขแƒ แƒแƒœแƒ”แƒ’แƒแƒขแƒ˜แƒฃแƒ แƒแƒ‘แƒ แƒ“แƒ แƒžแƒแƒšแƒแƒ แƒ˜แƒ–แƒ”แƒ‘แƒ. แƒแƒ›แƒ แƒ˜แƒ’แƒแƒ“, แƒ›แƒŸแƒแƒ•แƒ”แƒ‘แƒ˜แƒก แƒกแƒ”แƒ แƒ˜แƒแƒจแƒ˜, แƒ“แƒ˜แƒกแƒแƒชแƒ˜แƒแƒชแƒ˜แƒ˜แƒก แƒฃแƒœแƒแƒ แƒ˜ แƒ›แƒชแƒ˜แƒ แƒ“แƒ”แƒ‘แƒ แƒจแƒ”แƒ›แƒ“แƒ”แƒ’แƒœแƒแƒ˜แƒ แƒแƒ“:

S-แƒ—>O-H>-N-แƒ—>C-แƒฐ

แƒ”แƒก แƒกแƒ”แƒ แƒ˜แƒ แƒกแƒ แƒฃแƒšแƒงแƒแƒคแƒ˜แƒšแƒแƒ“ แƒ”แƒ›แƒ—แƒฎแƒ•แƒ”แƒ•แƒ แƒžแƒ”แƒ แƒ˜แƒแƒ“แƒฃแƒšแƒ˜ แƒชแƒฎแƒ แƒ˜แƒšแƒ˜แƒ“แƒแƒœ แƒชแƒœแƒแƒ‘แƒ˜แƒšแƒ˜ แƒแƒขแƒแƒ›แƒ”แƒ‘แƒ˜แƒก แƒ—แƒ•แƒ˜แƒกแƒ”แƒ‘แƒ”แƒ‘แƒก.

แƒ’แƒแƒ แƒ”แƒ›แƒแƒก แƒ’แƒแƒ•แƒšแƒ”แƒœแƒ.แƒ—แƒฃ แƒกแƒขแƒ แƒฃแƒฅแƒขแƒฃแƒ แƒ˜แƒ— แƒ›แƒกแƒ’แƒแƒ•แƒกแƒ˜ แƒœแƒ˜แƒ•แƒ—แƒ˜แƒ”แƒ แƒ”แƒ‘แƒ”แƒ‘แƒ˜ แƒจแƒ”แƒ“แƒแƒ แƒ”แƒ‘แƒฃแƒšแƒ˜แƒ, แƒจแƒ”แƒคแƒแƒกแƒ”แƒ‘แƒ แƒฎแƒแƒ แƒชแƒ˜แƒ”แƒšแƒ“แƒ”แƒ‘แƒ แƒ”แƒšแƒ”แƒฅแƒขแƒ แƒแƒœแƒ˜แƒก แƒกแƒ˜แƒ›แƒ™แƒ•แƒ แƒ˜แƒ•แƒ˜แƒก แƒจแƒ”แƒ“แƒแƒ แƒ”แƒ‘แƒ˜แƒ— แƒ˜แƒ› แƒแƒขแƒแƒ›แƒ–แƒ”, แƒ แƒแƒ›แƒ”แƒšแƒ›แƒแƒช แƒจแƒ”แƒ›แƒแƒ˜แƒฆแƒ แƒžแƒ แƒแƒขแƒแƒœแƒ˜. แƒงแƒ•แƒ”แƒšแƒ แƒกแƒขแƒ แƒฃแƒฅแƒขแƒฃแƒ แƒฃแƒšแƒ˜ แƒคแƒแƒฅแƒขแƒแƒ แƒ˜, แƒ แƒแƒ›แƒ”แƒšแƒ˜แƒช แƒฎแƒ”แƒšแƒก แƒฃแƒฌแƒงแƒแƒ‘แƒก แƒ›แƒฃแƒฎแƒขแƒ˜แƒก แƒจแƒ”แƒ›แƒชแƒ˜แƒ แƒ”แƒ‘แƒแƒก, แƒแƒกแƒขแƒแƒ‘แƒ˜แƒšแƒฃแƒ แƒ”แƒ‘แƒก แƒแƒœแƒ˜แƒแƒœแƒก, แƒฎแƒแƒšแƒ แƒ›แƒฃแƒฎแƒขแƒ˜แƒก แƒ›แƒแƒขแƒ”แƒ‘แƒ แƒ“แƒ”แƒกแƒขแƒแƒ‘แƒ˜แƒšแƒ˜แƒ–แƒแƒชแƒ˜แƒแƒก แƒแƒฎแƒ“แƒ”แƒœแƒก. แƒแƒ›แƒ แƒ˜แƒ’แƒแƒ“, แƒงแƒ•แƒ”แƒšแƒ แƒ›แƒ˜แƒ›แƒฆแƒ”แƒ‘แƒ˜ แƒ–แƒ แƒ“แƒ˜แƒก แƒ›แƒŸแƒแƒ•แƒ˜แƒแƒœแƒแƒ‘แƒแƒก, แƒงแƒ•แƒ”แƒšแƒ แƒ“แƒแƒœแƒแƒ แƒ˜ แƒแƒ›แƒชแƒ˜แƒ แƒ”แƒ‘แƒก แƒ›แƒแƒก.

แƒ”แƒก แƒฎแƒ“แƒ”แƒ‘แƒ แƒ˜แƒ›แƒ˜แƒกแƒ“แƒ แƒ›แƒ˜แƒฃแƒฎแƒ”แƒ“แƒแƒ•แƒแƒ“, แƒ—แƒฃ แƒ แƒ แƒ”แƒคแƒ”แƒฅแƒขแƒ˜ แƒ˜แƒฌแƒ•แƒ”แƒ•แƒก แƒ”แƒšแƒ”แƒฅแƒขแƒ แƒแƒœแƒ˜แƒก แƒ’แƒแƒ“แƒแƒชแƒ”แƒ›แƒ˜แƒก (แƒ˜แƒœแƒ“แƒฃแƒฅแƒชแƒ˜แƒฃแƒ แƒ˜ แƒ—แƒฃ แƒ›แƒ”แƒ–แƒแƒ›แƒ”แƒ แƒฃแƒšแƒ˜) แƒ”แƒšแƒ”แƒฅแƒขแƒ แƒแƒœแƒ˜แƒก แƒกแƒ˜แƒ›แƒ™แƒ•แƒ แƒ˜แƒ•แƒ˜แƒก แƒ’แƒแƒ“แƒแƒœแƒแƒฌแƒ˜แƒšแƒ”แƒ‘แƒแƒก.

แƒ’แƒแƒ“แƒแƒญแƒ แƒ˜แƒก แƒ”แƒคแƒ”แƒฅแƒขแƒ˜.แƒฎแƒกแƒœแƒแƒ แƒ˜ (แƒ’แƒแƒ›แƒฎแƒกแƒœแƒ”แƒšแƒ˜แƒก แƒ›แƒแƒšแƒ”แƒ™แƒฃแƒšแƒ”แƒ‘แƒ—แƒแƒœ แƒฃแƒ แƒ—แƒ˜แƒ”แƒ แƒ—แƒฅแƒ›แƒ”แƒ“แƒ”แƒ‘แƒ) แƒ–แƒ แƒ“แƒ˜แƒก แƒแƒœแƒ˜แƒแƒœแƒ˜แƒก แƒกแƒขแƒแƒ‘แƒ˜แƒšแƒฃแƒ แƒแƒ‘แƒแƒก แƒแƒœแƒ˜แƒแƒœแƒกแƒ แƒ“แƒ แƒ’แƒแƒ›แƒฎแƒกแƒœแƒ”แƒšแƒ˜แƒก แƒ›แƒแƒšแƒ”แƒ™แƒฃแƒšแƒ”แƒ‘แƒก แƒจแƒแƒ แƒ˜แƒก แƒญแƒแƒ แƒ‘แƒ˜ แƒ”แƒšแƒ”แƒฅแƒขแƒ แƒแƒœแƒ˜แƒก แƒกแƒ˜แƒ›แƒ™แƒ•แƒ แƒ˜แƒ•แƒ˜แƒก แƒ’แƒแƒ“แƒแƒœแƒแƒฌแƒ˜แƒšแƒ”แƒ‘แƒ˜แƒก แƒ’แƒแƒ›แƒ. แƒ–แƒแƒ’แƒแƒ“แƒแƒ“, แƒœแƒ˜แƒ›แƒฃแƒจแƒ˜ แƒแƒกแƒ”แƒ—แƒ˜แƒ:

แƒ แƒแƒช แƒฃแƒคแƒ แƒ แƒžแƒแƒšแƒแƒ แƒฃแƒšแƒ˜แƒ แƒ’แƒแƒ›แƒฎแƒกแƒœแƒ”แƒšแƒ˜, แƒ›แƒ˜แƒ— แƒฃแƒคแƒ แƒ แƒซแƒšแƒ˜แƒ”แƒ แƒ˜แƒ แƒฎแƒกแƒœแƒแƒ แƒ˜;

แƒ แƒแƒช แƒฃแƒคแƒ แƒ แƒžแƒแƒขแƒแƒ แƒแƒ แƒ˜แƒแƒœแƒ˜, แƒ›แƒ˜แƒ— แƒฃแƒ™แƒ”แƒ—แƒ”แƒกแƒแƒ“ แƒ˜แƒฎแƒกแƒœแƒ”แƒ‘แƒ แƒ˜แƒ’แƒ˜.

แƒกแƒแƒคแƒฃแƒซแƒ•แƒšแƒ˜แƒแƒœแƒแƒ‘แƒ แƒ‘แƒ แƒแƒœแƒกแƒขแƒ”แƒ“แƒ˜แƒก แƒ›แƒ˜แƒฎแƒ”แƒ“แƒ•แƒ˜แƒ— แƒแƒ แƒ˜แƒก แƒœแƒ˜แƒ•แƒ—แƒ˜แƒ”แƒ แƒ”แƒ‘แƒ˜แƒก แƒฃแƒœแƒแƒ แƒ˜, แƒฃแƒ–แƒ แƒฃแƒœแƒ•แƒ”แƒšแƒงแƒแƒก แƒ—แƒแƒ•แƒ˜แƒกแƒ˜ แƒฌแƒงแƒ•แƒ˜แƒšแƒ˜ แƒ”แƒšแƒ”แƒฅแƒขแƒ แƒแƒœแƒ”แƒ‘แƒ˜ แƒžแƒ แƒแƒขแƒแƒœแƒ—แƒแƒœ แƒฃแƒ แƒ—แƒ˜แƒ”แƒ แƒ—แƒฅแƒ›แƒ”แƒ“แƒ”แƒ‘แƒ˜แƒกแƒ—แƒ•แƒ˜แƒก. แƒ แƒแƒ’แƒแƒ แƒช แƒฌแƒ”แƒกแƒ˜, แƒ”แƒก แƒแƒ แƒ˜แƒก แƒœแƒ˜แƒ•แƒ—แƒ˜แƒ”แƒ แƒ”แƒ‘แƒ”แƒ‘แƒ˜, แƒ แƒแƒ›แƒšแƒ”แƒ‘แƒ˜แƒช แƒจแƒ”แƒ˜แƒชแƒแƒ•แƒก แƒ›แƒแƒšแƒ”แƒ™แƒฃแƒšแƒแƒจแƒ˜ แƒแƒ–แƒแƒขแƒ˜แƒก, แƒŸแƒแƒœแƒ’แƒ‘แƒแƒ“แƒ˜แƒก แƒ“แƒ แƒ’แƒแƒ’แƒ˜แƒ แƒ“แƒ˜แƒก แƒแƒขแƒแƒ›แƒ”แƒ‘แƒก.

แƒ แƒแƒช แƒฃแƒคแƒ แƒ แƒกแƒฃแƒกแƒขแƒ˜แƒ แƒซแƒ˜แƒ แƒ˜แƒ—แƒแƒ“แƒ˜ แƒชแƒ”แƒœแƒขแƒ แƒ˜ แƒ˜แƒœแƒแƒฎแƒแƒ•แƒก แƒ”แƒšแƒ”แƒฅแƒขแƒ แƒแƒœแƒ”แƒ‘แƒ˜แƒก แƒฌแƒงแƒ•แƒ˜แƒšแƒก, แƒ›แƒ˜แƒ— แƒฃแƒคแƒ แƒ แƒ›แƒแƒฆแƒแƒšแƒ˜แƒ แƒ‘แƒแƒ–แƒ˜แƒกแƒฃแƒ แƒแƒ‘แƒ. แฒ›แƒ˜แƒงแƒแƒšแƒ”แƒ‘แƒ˜แƒ—

R3-N >R2O>R2แƒก

แƒกแƒแƒ‘แƒแƒ–แƒ˜แƒกแƒ แƒ›แƒชแƒ˜แƒ แƒ“แƒ”แƒ‘แƒ. แƒ”แƒก แƒ—แƒแƒœแƒ›แƒ˜แƒ›แƒ“แƒ”แƒ•แƒ แƒแƒ‘แƒ แƒแƒ“แƒ•แƒ˜แƒšแƒ˜ แƒ“แƒแƒกแƒแƒ›แƒแƒฎแƒกแƒแƒ•แƒ แƒ”แƒ‘แƒ”แƒšแƒ˜แƒ "NOS" แƒ›แƒœแƒ”แƒ›แƒแƒœแƒฃแƒ แƒ˜ แƒฌแƒ”แƒกแƒ˜แƒก แƒ’แƒแƒ›แƒแƒงแƒ”แƒœแƒ”แƒ‘แƒ˜แƒ—.

แƒแƒ แƒกแƒ”แƒ‘แƒแƒ‘แƒก แƒ™แƒแƒ•แƒจแƒ˜แƒ แƒ˜ แƒ‘แƒ แƒแƒœแƒกแƒขแƒ”แƒ“แƒ˜แƒก แƒคแƒฃแƒซแƒ”แƒ”แƒ‘แƒก แƒจแƒแƒ แƒ˜แƒก: แƒแƒœแƒ˜แƒแƒœแƒ”แƒ‘แƒ˜ แƒฃแƒคแƒ แƒ แƒซแƒšแƒ˜แƒ”แƒ แƒ˜ แƒคแƒฃแƒซแƒ”แƒ”แƒ‘แƒ˜แƒ, แƒ•แƒ˜แƒ“แƒ แƒ” แƒจแƒ”แƒกแƒแƒ‘แƒแƒ›แƒ˜แƒกแƒ˜ แƒœแƒ”แƒ˜แƒขแƒ แƒแƒšแƒฃแƒ แƒ˜ แƒ›แƒแƒšแƒ”แƒ™แƒฃแƒšแƒ”แƒ‘แƒ˜. แƒ›แƒแƒ’แƒแƒšแƒ˜แƒ—แƒแƒ“, แƒฐแƒ˜แƒ“แƒ แƒแƒฅแƒกแƒ˜แƒ“แƒ˜แƒก แƒแƒœแƒ˜แƒแƒœแƒ˜ (-OH) แƒฃแƒคแƒ แƒ แƒซแƒšแƒ˜แƒ”แƒ แƒ˜ แƒ‘แƒแƒ–แƒแƒ แƒ•แƒ˜แƒ“แƒ แƒ” แƒฌแƒงแƒแƒšแƒ˜ (H 2 O). แƒ แƒแƒ“แƒ”แƒกแƒแƒช แƒคแƒฃแƒซแƒ” แƒฃแƒ แƒ—แƒ˜แƒ”แƒ แƒ—แƒฅแƒ›แƒ”แƒ“แƒ”แƒ‘แƒก แƒžแƒ แƒแƒขแƒแƒœแƒ—แƒแƒœ, แƒแƒœแƒ˜แƒฃแƒ›แƒ˜แƒก แƒ™แƒแƒ—แƒ˜แƒแƒœแƒ”แƒ‘แƒ˜ แƒจแƒ”แƒ˜แƒซแƒšแƒ”แƒ‘แƒ แƒฌแƒแƒ แƒ›แƒแƒ˜แƒฅแƒ›แƒœแƒแƒก:

R 3 O + - แƒแƒฅแƒกแƒแƒœแƒ˜แƒฃแƒ›แƒ˜แƒก แƒ™แƒแƒขแƒ˜แƒแƒœแƒ˜;

NR 4 + - แƒแƒ›แƒแƒœแƒ˜แƒฃแƒ›แƒ˜แƒก แƒ™แƒแƒขแƒ˜แƒแƒœแƒ˜;

ยท R 3 S + - แƒกแƒฃแƒšแƒคแƒแƒœแƒ˜แƒฃแƒ›แƒ˜แƒก แƒ™แƒแƒขแƒ˜แƒแƒœแƒ˜.

แƒ›แƒกแƒ’แƒแƒ•แƒกแƒ˜ แƒกแƒขแƒ แƒฃแƒฅแƒขแƒฃแƒ แƒ˜แƒก แƒ›แƒฅแƒแƒœแƒ” แƒœแƒ˜แƒ•แƒ—แƒ˜แƒ”แƒ แƒ”แƒ‘แƒ”แƒ‘แƒจแƒ˜ แƒ‘แƒแƒ–แƒ˜แƒกแƒฃแƒ แƒแƒ‘แƒ˜แƒก แƒฎแƒแƒ แƒ˜แƒกแƒฎแƒแƒ‘แƒ แƒ˜แƒ•แƒ˜ แƒจแƒ”แƒคแƒแƒกแƒ”แƒ‘แƒ แƒฎแƒแƒ แƒชแƒ˜แƒ”แƒšแƒ“แƒ”แƒ‘แƒ แƒ˜แƒ›แƒแƒ•แƒ” แƒšแƒแƒ’แƒ˜แƒ™แƒ˜แƒ—, แƒ แƒแƒ’แƒแƒ แƒช แƒ›แƒŸแƒแƒ•แƒ˜แƒแƒœแƒแƒ‘แƒ˜แƒก แƒจแƒ”แƒคแƒแƒกแƒ”แƒ‘แƒ, แƒ›แƒแƒ’แƒ แƒแƒ› แƒกแƒแƒžแƒ˜แƒ แƒ˜แƒกแƒžแƒ˜แƒ แƒ แƒœแƒ˜แƒจแƒœแƒ˜แƒ—.

แƒแƒฅแƒ”แƒ“แƒแƒœ แƒ’แƒแƒ›แƒแƒ›แƒ“แƒ˜แƒœแƒแƒ แƒ”, แƒงแƒ•แƒ”แƒšแƒ แƒ›แƒ˜แƒ›แƒฆแƒ”แƒ‘แƒ˜ แƒจแƒ”แƒ›แƒชแƒ•แƒšแƒ”แƒšแƒ˜ แƒแƒ›แƒชแƒ˜แƒ แƒ”แƒ‘แƒก แƒ›แƒแƒ— แƒกแƒแƒคแƒฃแƒซแƒ•แƒ”แƒšแƒก, แƒงแƒ•แƒ”แƒšแƒ แƒ“แƒแƒœแƒแƒ แƒ˜ แƒจแƒ”แƒ›แƒชแƒ•แƒšแƒ”แƒšแƒ˜ แƒ–แƒ แƒ“แƒ˜แƒก แƒ›แƒแƒ— แƒคแƒฃแƒซแƒ”แƒฃแƒšแƒแƒ‘แƒแƒก.

แƒšแƒฃแƒ˜แƒกแƒ˜แƒก แƒ›แƒŸแƒแƒ•แƒ”แƒ‘แƒ˜ แƒ“แƒ แƒคแƒฃแƒซแƒ”แƒ”แƒ‘แƒ˜

แƒšแƒฃแƒ˜แƒกแƒ˜แƒก แƒคแƒฃแƒซแƒ”แƒ”แƒ‘แƒ˜ แƒแƒ แƒ˜แƒแƒœ แƒ”แƒšแƒ”แƒฅแƒขแƒ แƒแƒœแƒฃแƒšแƒ˜ แƒฌแƒงแƒ•แƒ˜แƒšแƒ˜แƒก แƒ“แƒแƒœแƒแƒ แƒ”แƒ‘แƒ˜, แƒ˜แƒกแƒ”แƒ•แƒ” แƒ แƒแƒ’แƒแƒ แƒช แƒ‘แƒ แƒแƒœแƒกแƒขแƒ”แƒ“แƒ˜.

แƒ›แƒŸแƒแƒ•แƒ”แƒ‘แƒ˜แƒก แƒšแƒฃแƒ˜แƒกแƒ˜แƒก แƒ’แƒแƒœแƒ›แƒแƒ แƒขแƒ”แƒ‘แƒ แƒ›แƒ™แƒ•แƒ”แƒ—แƒ แƒแƒ“ แƒ’แƒแƒœแƒกแƒฎแƒ•แƒแƒ•แƒ“แƒ”แƒ‘แƒ แƒฉแƒ•แƒ”แƒฃแƒšแƒ”แƒ‘แƒ แƒ˜แƒ•แƒ˜แƒกแƒ’แƒแƒœ (แƒ‘แƒ แƒแƒœแƒกแƒขแƒ”แƒ“แƒ˜แƒก แƒ›แƒ˜แƒฎแƒ”แƒ“แƒ•แƒ˜แƒ—). แƒšแƒฃแƒ˜แƒกแƒ˜แƒก แƒ›แƒ˜แƒฎแƒ”แƒ“แƒ•แƒ˜แƒ—, แƒšแƒฃแƒ˜แƒกแƒ˜แƒก แƒ›แƒŸแƒแƒ•แƒ แƒแƒ แƒ˜แƒก แƒœแƒ”แƒ‘แƒ˜แƒกแƒ›แƒ˜แƒ”แƒ แƒ˜ แƒ›แƒแƒšแƒ”แƒ™แƒฃแƒšแƒ แƒแƒœ แƒ˜แƒแƒœแƒ˜, แƒ แƒแƒ›แƒ”แƒšแƒกแƒแƒช แƒแƒฅแƒ•แƒก แƒ—แƒแƒ•แƒ˜แƒกแƒฃแƒคแƒแƒšแƒ˜ แƒแƒ แƒ‘แƒ˜แƒขแƒแƒšแƒ˜, แƒ แƒแƒ›แƒ”แƒšแƒ˜แƒช แƒจแƒ”แƒ˜แƒซแƒšแƒ”แƒ‘แƒ แƒจแƒ”แƒ˜แƒ•แƒกแƒแƒก แƒ”แƒšแƒ”แƒฅแƒขแƒ แƒแƒœแƒฃแƒšแƒ˜ แƒฌแƒงแƒ•แƒ˜แƒšแƒ˜แƒ— แƒฃแƒ แƒ—แƒ˜แƒ”แƒ แƒ—แƒฅแƒ›แƒ”แƒ“แƒ”แƒ‘แƒ˜แƒก แƒจแƒ”แƒ“แƒ”แƒ’แƒแƒ“. แƒ—แƒฃ แƒ‘แƒ แƒแƒœแƒกแƒขแƒ”แƒ“แƒ˜แƒก แƒแƒ–แƒ แƒ˜แƒ—, แƒ›แƒŸแƒแƒ•แƒ แƒแƒ แƒ˜แƒก แƒžแƒ แƒแƒขแƒแƒœแƒ˜แƒก แƒ“แƒแƒœแƒแƒ แƒ˜, แƒ›แƒแƒจแƒ˜แƒœ แƒšแƒฃแƒ˜แƒกแƒ˜แƒก แƒ›แƒ˜แƒฎแƒ”แƒ“แƒ•แƒ˜แƒ—, แƒ—แƒแƒ•แƒแƒ“ แƒžแƒ แƒแƒขแƒแƒœแƒ˜ (H +) แƒแƒ แƒ˜แƒก แƒ›แƒŸแƒแƒ•แƒ, แƒ แƒแƒ“แƒ’แƒแƒœ แƒ›แƒ˜แƒกแƒ˜ แƒแƒ แƒ‘แƒ˜แƒขแƒแƒšแƒ˜ แƒชแƒแƒ แƒ˜แƒ”แƒšแƒ˜แƒ. แƒšแƒฃแƒ˜แƒกแƒ˜แƒก แƒ›แƒŸแƒแƒ•แƒ”แƒ‘แƒ˜ แƒ‘แƒ”แƒ•แƒ แƒ˜แƒ: Na +, Mg 2+, SnCl 4, SbCl 5, AlCl 3, BF 3, FeBr 3 แƒ“แƒ แƒ.แƒจ. แƒšแƒฃแƒ˜แƒกแƒ˜แƒก แƒ—แƒ”แƒแƒ แƒ˜แƒ แƒกแƒแƒจแƒฃแƒแƒšแƒ”แƒ‘แƒแƒก แƒ˜แƒซแƒšแƒ”แƒ•แƒ แƒ›แƒ แƒแƒ•แƒแƒšแƒ˜ แƒ แƒ”แƒแƒฅแƒชแƒ˜แƒ แƒแƒฆแƒ˜แƒฌแƒ”แƒ แƒแƒก, แƒ แƒแƒ’แƒแƒ แƒช แƒ›แƒŸแƒแƒ•แƒ-แƒขแƒฃแƒขแƒแƒ•แƒแƒœแƒ˜ แƒฃแƒ แƒ—แƒ˜แƒ”แƒ แƒ—แƒฅแƒ›แƒ”แƒ“แƒ”แƒ‘แƒ. แฒ›แƒแƒ’แƒแƒšแƒ˜แƒ—แƒแƒ“:

แƒฎแƒจแƒ˜แƒ แƒแƒ“ แƒšแƒฃแƒ˜แƒกแƒ˜แƒก แƒ›แƒŸแƒแƒ•แƒ”แƒ‘แƒ—แƒแƒœ แƒ แƒ”แƒแƒฅแƒชแƒ˜แƒ”แƒ‘แƒจแƒ˜ แƒ›แƒแƒœแƒแƒฌแƒ˜แƒšแƒ”แƒแƒ‘แƒ”แƒœ แƒแƒ แƒ’แƒแƒœแƒฃแƒšแƒ˜ แƒœแƒแƒ”แƒ แƒ—แƒ”แƒ‘แƒ˜, แƒ แƒแƒ›แƒšแƒ”แƒ‘แƒ˜แƒช แƒฌแƒงแƒ•แƒ˜แƒšแƒ˜ p-แƒ”แƒšแƒ”แƒฅแƒขแƒ แƒแƒœแƒ”แƒ‘แƒ˜แƒก แƒ“แƒแƒœแƒแƒ แƒ˜แƒ:

แƒแƒ แƒ’แƒแƒœแƒฃแƒš แƒฅแƒ˜แƒ›แƒ˜แƒแƒจแƒ˜ แƒ›แƒ˜แƒฆแƒ”แƒ‘แƒฃแƒšแƒ˜แƒ แƒจแƒ”แƒ›แƒ“แƒ”แƒ’แƒ˜:

ยท แƒ—แƒฃ แƒ’แƒแƒ›แƒแƒ˜แƒงแƒ”แƒœแƒ”แƒ‘แƒ แƒขแƒ”แƒ แƒ›แƒ˜แƒœแƒ˜ โ€žแƒ›แƒŸแƒแƒ•แƒโ€œ, แƒ”แƒก แƒœแƒ˜แƒจแƒœแƒแƒ•แƒก แƒ‘แƒ แƒแƒœแƒกแƒขแƒ”แƒ“แƒ˜แƒก แƒ›แƒŸแƒแƒ•แƒแƒก;

แƒ—แƒฃ แƒขแƒ”แƒ แƒ›แƒ˜แƒœแƒ˜ "แƒ›แƒŸแƒแƒ•แƒ" แƒ’แƒแƒ›แƒแƒ˜แƒงแƒ”แƒœแƒ”แƒ‘แƒ แƒšแƒฃแƒ˜แƒกแƒ˜แƒก แƒ’แƒแƒ’แƒ”แƒ‘แƒ˜แƒ—, แƒ˜แƒกแƒ˜แƒœแƒ˜ แƒแƒ›แƒ‘แƒแƒ‘แƒ”แƒœ "แƒšแƒฃแƒ˜แƒก แƒ›แƒŸแƒแƒ•แƒ".


แƒšแƒ”แƒฅแƒชแƒ˜แƒ #5

แƒœแƒแƒฎแƒจแƒ˜แƒ แƒฌแƒงแƒแƒšแƒ‘แƒแƒ“แƒ”แƒ‘แƒ˜

แƒแƒšแƒ™แƒแƒœแƒ”แƒ‘แƒ˜

ยท แƒฐแƒแƒ›แƒแƒšแƒแƒ’แƒ˜แƒฃแƒ แƒ˜ แƒกแƒ”แƒ แƒ˜แƒ”แƒ‘แƒ˜, แƒœแƒแƒ›แƒ”แƒœแƒ™แƒšแƒแƒขแƒฃแƒ แƒ, แƒ˜แƒ–แƒแƒ›แƒ”แƒ แƒ˜แƒ–แƒ›แƒ˜, แƒแƒšแƒ™แƒ˜แƒšแƒ˜แƒก แƒ แƒแƒ“แƒ˜แƒ™แƒแƒšแƒ”แƒ‘แƒ˜. แƒแƒšแƒ™แƒแƒœแƒ˜แƒก แƒ›แƒแƒšแƒ”แƒ™แƒฃแƒšแƒ”แƒ‘แƒ˜แƒก แƒ”แƒšแƒ”แƒฅแƒขแƒ แƒแƒœแƒฃแƒšแƒ˜ แƒกแƒขแƒ แƒฃแƒฅแƒขแƒฃแƒ แƒ, sp 3 แƒฐแƒ˜แƒ‘แƒ แƒ˜แƒ“แƒ˜แƒ–แƒแƒชแƒ˜แƒ, s-แƒ‘แƒ›แƒ. C-C แƒ“แƒ C-H แƒ‘แƒ›แƒ˜แƒก แƒกแƒ˜แƒ’แƒ แƒซแƒ”, แƒ‘แƒ›แƒ˜แƒก แƒ™แƒฃแƒ—แƒฎแƒ”แƒ”แƒ‘แƒ˜, แƒ‘แƒ›แƒ˜แƒก แƒ”แƒœแƒ”แƒ แƒ’แƒ˜แƒ. แƒแƒ แƒ’แƒแƒœแƒฃแƒšแƒ˜ แƒœแƒ˜แƒ•แƒ—แƒ˜แƒ”แƒ แƒ”แƒ‘แƒ”แƒ‘แƒ˜แƒก แƒกแƒ˜แƒ•แƒ แƒชแƒ˜แƒ—แƒ˜ แƒ˜แƒ–แƒแƒ›แƒ”แƒ แƒ˜แƒ–แƒ›แƒ˜. sp 3 แƒฐแƒ˜แƒ‘แƒ แƒ˜แƒ“แƒ˜แƒ แƒ”แƒ‘แƒฃแƒšแƒ˜ แƒœแƒแƒฎแƒจแƒ˜แƒ แƒ‘แƒแƒ“แƒ˜แƒก แƒแƒขแƒแƒ›แƒ”แƒ‘แƒ˜แƒ— แƒ›แƒแƒšแƒ”แƒ™แƒฃแƒšแƒ”แƒ‘แƒ˜แƒก แƒกแƒ˜แƒ•แƒ แƒชแƒ˜แƒ—แƒ˜ แƒกแƒขแƒ แƒฃแƒฅแƒขแƒฃแƒ แƒ˜แƒก แƒ’แƒแƒ›แƒแƒกแƒแƒฎแƒ•แƒ˜แƒก แƒ›แƒ”แƒ—แƒแƒ“แƒ”แƒ‘แƒ˜. แƒแƒšแƒ™แƒแƒœแƒ”แƒ‘แƒ˜แƒก แƒกแƒžแƒ”แƒฅแƒขแƒ แƒฃแƒšแƒ˜ แƒ›แƒแƒฎแƒแƒกแƒ˜แƒแƒ—แƒ”แƒ‘แƒšแƒ”แƒ‘แƒ˜. แƒแƒšแƒ™แƒแƒœแƒ”แƒ‘แƒ˜แƒก แƒคแƒ˜แƒ–แƒ˜แƒ™แƒฃแƒ แƒ˜ แƒ—แƒ•แƒ˜แƒกแƒ”แƒ‘แƒ”แƒ‘แƒ˜ แƒ“แƒ แƒ›แƒแƒ—แƒ˜ แƒชแƒ•แƒšแƒ˜แƒšแƒ”แƒ‘แƒ”แƒ‘แƒ˜แƒก แƒœแƒ˜แƒ›แƒฃแƒจแƒ”แƒ‘แƒ˜ แƒฐแƒแƒ›แƒแƒšแƒแƒ’แƒ˜แƒฃแƒ  แƒกแƒ”แƒ แƒ˜แƒแƒจแƒ˜.

แƒแƒšแƒ™แƒแƒœแƒ”แƒ‘แƒ˜ (แƒ’แƒแƒฏแƒ”แƒ แƒ”แƒ‘แƒฃแƒšแƒ˜ แƒแƒชแƒ˜แƒ™แƒšแƒฃแƒ แƒ˜ แƒœแƒแƒ”แƒ แƒ—แƒ”แƒ‘แƒ˜, แƒžแƒแƒ แƒแƒคแƒ˜แƒœแƒ”แƒ‘แƒ˜)

แƒแƒšแƒ™แƒแƒœแƒ”แƒ‘แƒ˜ แƒแƒ แƒ˜แƒก แƒœแƒแƒฎแƒจแƒ˜แƒ แƒฌแƒงแƒแƒšแƒ‘แƒแƒ“แƒ”แƒ‘แƒ˜ แƒแƒขแƒแƒ›แƒ”แƒ‘แƒ˜แƒก แƒฆแƒ˜แƒ แƒฏแƒแƒญแƒ•แƒ˜แƒ—, แƒ แƒแƒ›แƒ”แƒšแƒ˜แƒช แƒจแƒ”แƒ”แƒกแƒแƒ‘แƒแƒ›แƒ”แƒ‘แƒ แƒคแƒแƒ แƒ›แƒฃแƒšแƒแƒก C n H 2 n + 2, แƒกแƒแƒ“แƒแƒช แƒœแƒแƒฎแƒจแƒ˜แƒ แƒ‘แƒแƒ“แƒ˜แƒก แƒแƒขแƒแƒ›แƒ”แƒ‘แƒ˜ แƒฃแƒ แƒ—แƒ˜แƒ”แƒ แƒ—แƒ“แƒแƒ™แƒแƒ•แƒจแƒ˜แƒ แƒ”แƒ‘แƒฃแƒšแƒ˜แƒ แƒ›แƒฎแƒแƒšแƒแƒ“ ฯƒ-แƒ‘แƒ›แƒ”แƒ‘แƒ˜แƒ—.

แƒขแƒ”แƒ แƒ›แƒ˜แƒœแƒ˜ โ€žแƒ’แƒแƒฏแƒ”แƒ แƒ”แƒ‘แƒฃแƒšแƒ˜โ€œ แƒœแƒ˜แƒจแƒœแƒแƒ•แƒก, แƒ แƒแƒ› แƒแƒกแƒ”แƒ—แƒ˜ แƒœแƒ˜แƒ•แƒ—แƒ˜แƒ”แƒ แƒ”แƒ‘แƒ˜แƒก แƒ›แƒแƒšแƒ”แƒ™แƒฃแƒšแƒแƒจแƒ˜ แƒ—แƒ˜แƒ—แƒแƒ”แƒฃแƒšแƒ˜ แƒœแƒแƒฎแƒจแƒ˜แƒ แƒ‘แƒแƒ“แƒ˜ แƒแƒกแƒแƒชแƒ˜แƒ แƒ“แƒ”แƒ‘แƒ แƒแƒขแƒแƒ›แƒ”แƒ‘แƒ˜แƒก แƒ›แƒแƒฅแƒกแƒ˜แƒ›แƒแƒšแƒฃแƒ  แƒจแƒ”แƒกแƒแƒซแƒšแƒ แƒ แƒแƒแƒ“แƒ”แƒœแƒแƒ‘แƒแƒกแƒ—แƒแƒœ (แƒแƒ—แƒฎแƒ˜ แƒแƒขแƒแƒ›แƒ˜แƒ—).

แƒ›แƒ”แƒ—แƒแƒœแƒ˜แƒก แƒแƒ’แƒ”แƒ‘แƒฃแƒšแƒ”แƒ‘แƒ แƒ“แƒ”แƒขแƒแƒšแƒฃแƒ แƒแƒ“ แƒแƒ แƒ˜แƒก แƒแƒฆแƒฌแƒ”แƒ แƒ˜แƒšแƒ˜ No2 แƒšแƒ”แƒฅแƒชแƒ˜แƒแƒจแƒ˜.

แƒ˜แƒ–แƒแƒ›แƒ”แƒ แƒ˜แƒ–แƒ›แƒ˜, แƒœแƒแƒ›แƒ”แƒœแƒ™แƒšแƒแƒขแƒฃแƒ แƒ

แƒฐแƒแƒ›แƒแƒšแƒแƒ’แƒ˜แƒฃแƒ แƒ˜ แƒกแƒ”แƒ แƒ˜แƒ˜แƒก แƒžแƒ˜แƒ แƒ•แƒ”แƒšแƒ˜ แƒกแƒแƒ›แƒ˜ แƒฌแƒ”แƒ•แƒ แƒ˜ (แƒ›แƒ”แƒ—แƒแƒœแƒ˜, แƒ”แƒ—แƒแƒœแƒ˜ แƒ“แƒ แƒžแƒ แƒแƒžแƒแƒœแƒ˜) แƒแƒ แƒกแƒ”แƒ‘แƒแƒ‘แƒก แƒ แƒแƒ’แƒแƒ แƒช แƒ”แƒ แƒ—แƒ˜ แƒกแƒขแƒ แƒฃแƒฅแƒขแƒฃแƒ แƒฃแƒšแƒ˜ แƒ˜แƒ–แƒแƒ›แƒ”แƒ แƒ˜. แƒ‘แƒฃแƒขแƒแƒœแƒ˜แƒ“แƒแƒœ แƒ“แƒแƒฌแƒงแƒ”แƒ‘แƒฃแƒšแƒ˜, แƒ˜แƒ–แƒแƒ›แƒ”แƒ แƒ”แƒ‘แƒ˜แƒก แƒ แƒแƒแƒ“แƒ”แƒœแƒแƒ‘แƒ แƒกแƒฌแƒ แƒแƒคแƒแƒ“ แƒ˜แƒ–แƒ แƒ“แƒ”แƒ‘แƒ: แƒžแƒ”แƒœแƒขแƒแƒœแƒก แƒแƒฅแƒ•แƒก แƒกแƒแƒ›แƒ˜ แƒ˜แƒ–แƒแƒ›แƒ”แƒ แƒ˜, แƒฎแƒแƒšแƒ แƒ“แƒ”แƒ™แƒแƒœแƒก (C 10 H 22) แƒฃแƒ™แƒ•แƒ” แƒแƒฅแƒ•แƒก 75.

แƒ—แƒแƒ•แƒ˜ 20

20.1. แƒ›แƒแƒฅแƒ›แƒ”แƒ“แƒ˜ แƒ›แƒแƒกแƒ”แƒ‘แƒ˜แƒก แƒ™แƒแƒœแƒแƒœแƒ˜

แƒ—แƒฅแƒ•แƒ”แƒœ แƒ’แƒแƒ”แƒชแƒแƒœแƒ˜แƒ— แƒ›แƒแƒกแƒ˜แƒก แƒ›แƒแƒฅแƒ›แƒ”แƒ“แƒ”แƒ‘แƒ˜แƒก แƒ™แƒแƒœแƒแƒœแƒก แƒจแƒ”แƒฅแƒชแƒ”แƒ•แƒแƒ“แƒ˜ แƒฅแƒ˜แƒ›แƒ˜แƒฃแƒ แƒ˜ แƒ แƒ”แƒแƒฅแƒชแƒ˜แƒ”แƒ‘แƒ˜แƒก แƒฌแƒแƒœแƒแƒกแƒฌแƒแƒ แƒแƒ‘แƒ˜แƒก แƒจแƒ”แƒกแƒฌแƒแƒ•แƒšแƒ˜แƒ— (แƒ—แƒแƒ•แƒ˜ 9 ยง 5). แƒจแƒ”แƒ’แƒแƒฎแƒกแƒ”แƒœแƒ”แƒ‘แƒ—, แƒ แƒแƒ› แƒ›แƒฃแƒ“แƒ›แƒ˜แƒ• แƒขแƒ”แƒ›แƒžแƒ”แƒ แƒแƒขแƒฃแƒ แƒแƒ–แƒ” แƒจแƒ”แƒฅแƒชแƒ”แƒ•แƒแƒ“แƒ˜ แƒ แƒ”แƒแƒฅแƒชแƒ˜แƒ˜แƒกแƒ—แƒ•แƒ˜แƒก

แƒ A+ แƒ‘แƒ‘ แƒ“ D+ แƒ•แƒค

แƒ›แƒแƒกแƒ˜แƒก แƒ›แƒแƒฅแƒ›แƒ”แƒ“แƒ”แƒ‘แƒ˜แƒก แƒ™แƒแƒœแƒแƒœแƒ˜ แƒ’แƒแƒ›แƒแƒ˜แƒฎแƒแƒขแƒ”แƒ‘แƒ แƒ’แƒแƒœแƒขแƒแƒšแƒ”แƒ‘แƒ˜แƒ—

แƒ›แƒแƒ’แƒ”แƒฎแƒกแƒ”แƒœแƒ”แƒ‘แƒแƒ—, แƒ แƒแƒ› แƒ›แƒแƒกแƒแƒ‘แƒ แƒ˜แƒ•แƒ˜ แƒ›แƒแƒฅแƒ›แƒ”แƒ“แƒ”แƒ‘แƒ˜แƒก แƒ™แƒแƒœแƒแƒœแƒ˜แƒก แƒ’แƒแƒ›แƒแƒงแƒ”แƒœแƒ”แƒ‘แƒ˜แƒกแƒแƒก แƒ›แƒœแƒ˜แƒจแƒ•แƒœแƒ”แƒšแƒแƒ•แƒแƒœแƒ˜แƒ แƒ˜แƒชแƒแƒ“แƒ”แƒ— แƒ แƒ แƒแƒ’แƒ แƒ”แƒ’แƒแƒชแƒ˜แƒ˜แƒก แƒ›แƒ“แƒ’แƒแƒ›แƒแƒ แƒ”แƒแƒ‘แƒแƒจแƒ˜ แƒแƒ แƒ˜แƒแƒœ แƒ แƒ”แƒแƒฅแƒชแƒ˜แƒแƒจแƒ˜ แƒ›แƒแƒœแƒแƒฌแƒ˜แƒšแƒ” แƒœแƒ˜แƒ•แƒ—แƒ˜แƒ”แƒ แƒ”แƒ‘แƒ”แƒ‘แƒ˜. แƒ›แƒแƒ’แƒ แƒแƒ› แƒแƒ แƒ แƒ›แƒฎแƒแƒšแƒแƒ“ แƒ”แƒก: แƒ›แƒœแƒ˜แƒจแƒ•แƒœแƒ”แƒšแƒแƒ•แƒแƒœแƒ˜แƒ แƒ›แƒแƒชแƒ”แƒ›แƒฃแƒš แƒฅแƒ˜แƒ›แƒ˜แƒฃแƒ  แƒกแƒ˜แƒกแƒขแƒ”แƒ›แƒแƒจแƒ˜ แƒคแƒแƒ–แƒ”แƒ‘แƒ˜แƒก แƒ แƒแƒแƒ“แƒ”แƒœแƒแƒ‘แƒ แƒ“แƒ แƒ—แƒแƒœแƒแƒคแƒแƒ แƒ“แƒแƒ‘แƒ. แƒคแƒแƒ–แƒ”แƒ‘แƒ˜แƒก แƒ แƒแƒแƒ“แƒ”แƒœแƒแƒ‘แƒ˜แƒก แƒ›แƒ˜แƒฎแƒ”แƒ“แƒ•แƒ˜แƒ—, แƒ แƒ”แƒแƒฅแƒชแƒ˜แƒ”แƒ‘แƒ˜ แƒ˜แƒงแƒแƒคแƒ แƒฐแƒแƒ›แƒแƒคแƒแƒ–แƒฃแƒ แƒ˜, แƒ“แƒ แƒฐแƒ”แƒขแƒ”แƒ แƒแƒคแƒแƒ–แƒ.แƒฐแƒ”แƒขแƒ”แƒ แƒแƒคแƒแƒ–แƒ˜แƒ”แƒ‘แƒก แƒจแƒแƒ แƒ˜แƒก, แƒ›แƒงแƒแƒ แƒ˜ แƒคแƒแƒ–แƒแƒ แƒ”แƒแƒฅแƒชแƒ˜แƒ”แƒ‘แƒ˜.

แƒฐแƒแƒ›แƒแƒคแƒแƒ–แƒฃแƒ แƒ˜ แƒ แƒ”แƒแƒฅแƒชแƒ˜แƒแƒฅแƒ˜แƒ›แƒ˜แƒฃแƒ แƒ˜ แƒ แƒ”แƒแƒฅแƒชแƒ˜แƒ, แƒ แƒแƒ›แƒ”แƒšแƒจแƒ˜แƒช แƒงแƒ•แƒ”แƒšแƒ แƒ›แƒแƒœแƒแƒฌแƒ˜แƒšแƒ” แƒ”แƒ แƒ—แƒกแƒ แƒ“แƒ แƒ˜แƒ›แƒแƒ•แƒ” แƒคแƒแƒ–แƒแƒจแƒ˜แƒ.

แƒแƒกแƒ”แƒ—แƒ˜ แƒคแƒแƒ–แƒ แƒจแƒ”แƒ˜แƒซแƒšแƒ”แƒ‘แƒ แƒ˜แƒงแƒแƒก แƒแƒ˜แƒ แƒ”แƒ‘แƒ˜แƒก แƒœแƒแƒ แƒ”แƒ•แƒ˜ (แƒ’แƒแƒ–แƒ˜แƒก แƒคแƒแƒ–แƒ), แƒแƒœ แƒ—แƒฎแƒ”แƒ•แƒแƒ“แƒ˜ แƒฎแƒกแƒœแƒแƒ แƒ˜ (แƒ—แƒฎแƒ”แƒ•แƒแƒ“แƒ˜ แƒคแƒแƒ–แƒ). แƒแƒ› แƒจแƒ”แƒ›แƒ—แƒฎแƒ•แƒ”แƒ•แƒแƒจแƒ˜, แƒ แƒ”แƒแƒฅแƒชแƒ˜แƒแƒจแƒ˜ แƒ›แƒแƒœแƒแƒฌแƒ˜แƒšแƒ” แƒงแƒ•แƒ”แƒšแƒ แƒœแƒแƒฌแƒ˜แƒšแƒแƒ™แƒก (A, B, D แƒ“แƒ F) แƒแƒฅแƒ•แƒก แƒฃแƒœแƒแƒ แƒ˜ แƒจแƒ”แƒแƒกแƒ แƒฃแƒšแƒแƒก แƒฅแƒแƒแƒขแƒฃแƒ แƒ˜ แƒ›แƒแƒซแƒ แƒแƒแƒ‘แƒ แƒ”แƒ แƒ—แƒ›แƒแƒœแƒ”แƒ—แƒ˜แƒกแƒ’แƒแƒœ แƒ“แƒแƒ›แƒแƒฃแƒ™แƒ˜แƒ“แƒ”แƒ‘แƒšแƒแƒ“ แƒ“แƒ แƒจแƒ”แƒฅแƒชแƒ”แƒ•แƒแƒ“แƒ˜ แƒ แƒ”แƒแƒฅแƒชแƒ˜แƒ แƒ›แƒ˜แƒ›แƒ“แƒ˜แƒœแƒแƒ แƒ”แƒแƒ‘แƒก แƒ แƒ”แƒแƒฅแƒชแƒ˜แƒ˜แƒก แƒกแƒ˜แƒกแƒขแƒ”แƒ›แƒ˜แƒก แƒ›แƒ—แƒ”แƒš แƒ›แƒแƒชแƒฃแƒšแƒแƒ‘แƒแƒจแƒ˜. แƒชแƒฎแƒแƒ“แƒ˜แƒ, แƒแƒกแƒ”แƒ—แƒ˜ แƒœแƒแƒฌแƒ˜แƒšแƒแƒ™แƒ”แƒ‘แƒ˜ แƒจแƒ”แƒ˜แƒซแƒšแƒ”แƒ‘แƒ แƒ˜แƒงแƒแƒก แƒแƒ˜แƒ แƒ˜แƒกแƒ”แƒ‘แƒ แƒ˜ แƒœแƒ˜แƒ•แƒ—แƒ˜แƒ”แƒ แƒ”แƒ‘แƒ”แƒ‘แƒ˜แƒก แƒ›แƒแƒšแƒ”แƒ™แƒฃแƒšแƒ”แƒ‘แƒ˜, แƒแƒœ แƒ›แƒแƒšแƒ”แƒ™แƒฃแƒšแƒ”แƒ‘แƒ˜ แƒแƒœ แƒ˜แƒแƒœแƒ”แƒ‘แƒ˜, แƒ แƒแƒ›แƒšแƒ”แƒ‘แƒ˜แƒช แƒฅแƒ›แƒœแƒ˜แƒแƒœ แƒกแƒ˜แƒ—แƒฎแƒ”แƒก. แƒจแƒ”แƒฅแƒชแƒ”แƒ•แƒแƒ“แƒ˜ แƒฐแƒแƒ›แƒแƒคแƒแƒ–แƒฃแƒ แƒ˜ แƒ แƒ”แƒแƒฅแƒชแƒ˜แƒ”แƒ‘แƒ˜แƒก แƒ›แƒแƒ’แƒแƒšแƒ˜แƒ—แƒ”แƒ‘แƒ˜แƒ แƒแƒ›แƒ˜แƒแƒ™แƒ˜แƒก แƒกแƒ˜แƒœแƒ—แƒ”แƒ–แƒ˜แƒก แƒ แƒ”แƒแƒฅแƒชแƒ˜แƒ”แƒ‘แƒ˜, แƒฌแƒงแƒแƒšแƒ‘แƒแƒ“แƒจแƒ˜ แƒฅแƒšแƒแƒ แƒ˜แƒก แƒฌแƒ•แƒ, แƒ แƒ”แƒแƒฅแƒชแƒ˜แƒ แƒแƒ›แƒ˜แƒแƒ™แƒ˜แƒก แƒ“แƒ แƒฌแƒงแƒแƒšแƒ‘แƒแƒ“แƒ˜แƒก แƒกแƒฃแƒšแƒคแƒ˜แƒ“แƒก แƒจแƒแƒ แƒ˜แƒก แƒฌแƒงแƒแƒšแƒฎแƒกแƒœแƒแƒ แƒจแƒ˜ แƒ“แƒ แƒ.แƒจ.

แƒ—แƒฃ แƒ แƒ”แƒแƒฅแƒชแƒ˜แƒแƒจแƒ˜ แƒ›แƒแƒœแƒแƒฌแƒ˜แƒšแƒ” แƒ”แƒ แƒ—แƒ˜ แƒœแƒ˜แƒ•แƒ—แƒ˜แƒ”แƒ แƒ”แƒ‘แƒ แƒ›แƒแƒ˜แƒœแƒช แƒ’แƒแƒœแƒกแƒฎแƒ•แƒแƒ•แƒ”แƒ‘แƒฃแƒš แƒคแƒแƒ–แƒแƒจแƒ˜แƒ, แƒ•แƒ˜แƒ“แƒ แƒ” แƒ“แƒแƒœแƒแƒ แƒฉแƒ”แƒœแƒ˜ แƒœแƒ˜แƒ•แƒ—แƒ˜แƒ”แƒ แƒ”แƒ‘แƒ”แƒ‘แƒ˜, แƒ›แƒแƒจแƒ˜แƒœ แƒจแƒ”แƒฅแƒชแƒ”แƒ•แƒแƒ“แƒ˜ แƒ แƒ”แƒแƒฅแƒชแƒ˜แƒ แƒ›แƒ˜แƒ›แƒ“แƒ˜แƒœแƒแƒ แƒ”แƒแƒ‘แƒก แƒ›แƒฎแƒแƒšแƒแƒ“ แƒ˜แƒœแƒขแƒ”แƒ แƒคแƒ”แƒ˜แƒกแƒ–แƒ” แƒ“แƒ แƒ”แƒฌแƒแƒ“แƒ”แƒ‘แƒ แƒฐแƒ”แƒขแƒ”แƒ แƒแƒคแƒแƒ–แƒแƒก แƒ แƒ”แƒแƒฅแƒชแƒ˜แƒแƒก.

แƒฐแƒ”แƒขแƒ”แƒ แƒแƒคแƒแƒ–แƒ˜แƒฃแƒ แƒ˜ แƒ แƒ”แƒแƒฅแƒชแƒ˜แƒ- แƒฅแƒ˜แƒ›แƒ˜แƒฃแƒ แƒ˜ แƒ แƒ”แƒแƒฅแƒชแƒ˜แƒ, แƒ แƒแƒ›แƒšแƒ˜แƒก แƒ›แƒแƒœแƒแƒฌแƒ˜แƒšแƒ”แƒ”แƒ‘แƒ˜ แƒกแƒฎแƒ•แƒแƒ“แƒแƒกแƒฎแƒ•แƒ แƒคแƒแƒ–แƒแƒจแƒ˜ แƒแƒ แƒ˜แƒแƒœ.

แƒจแƒ”แƒฅแƒชแƒ”แƒ•แƒแƒ“แƒ˜ แƒฐแƒ”แƒขแƒ”แƒ แƒแƒคแƒแƒ–แƒ˜แƒฃแƒ แƒ˜ แƒ แƒ”แƒแƒฅแƒชแƒ˜แƒ”แƒ‘แƒ˜ แƒ›แƒแƒ˜แƒชแƒแƒ•แƒก แƒ แƒ”แƒแƒฅแƒชแƒ˜แƒ”แƒ‘แƒก, แƒ แƒแƒ›แƒ”แƒšแƒจแƒ˜แƒช แƒจแƒ”แƒ“แƒ˜แƒก แƒแƒ˜แƒ แƒ˜แƒกแƒ”แƒ‘แƒ แƒ˜ แƒ“แƒ แƒ›แƒงแƒแƒ แƒ˜ แƒœแƒ˜แƒ•แƒ—แƒ˜แƒ”แƒ แƒ”แƒ‘แƒ”แƒ‘แƒ˜ (แƒ›แƒแƒ’แƒแƒšแƒ˜แƒ—แƒแƒ“, แƒ™แƒแƒšแƒชแƒ˜แƒฃแƒ›แƒ˜แƒก แƒ™แƒแƒ แƒ‘แƒแƒœแƒแƒขแƒ˜แƒก แƒ“แƒแƒจแƒšแƒ), แƒ—แƒฎแƒ”แƒ•แƒแƒ“แƒ˜ แƒ“แƒ แƒ›แƒงแƒแƒ แƒ˜ แƒœแƒ˜แƒ•แƒ—แƒ˜แƒ”แƒ แƒ”แƒ‘แƒ”แƒ‘แƒ˜ (แƒ›แƒแƒ’แƒแƒšแƒ˜แƒ—แƒแƒ“, แƒœแƒแƒšแƒ”แƒฅแƒ˜ แƒ‘แƒแƒ แƒ˜แƒฃแƒ›แƒ˜แƒก แƒกแƒฃแƒšแƒคแƒแƒขแƒ˜แƒก แƒฎแƒกแƒœแƒแƒ แƒ˜แƒ“แƒแƒœ แƒแƒœ แƒ—แƒฃแƒ—แƒ˜แƒ˜แƒก แƒ แƒ”แƒแƒฅแƒชแƒ˜แƒ แƒ›แƒแƒ แƒ˜แƒšแƒ›แƒŸแƒแƒ•แƒแƒกแƒ—แƒแƒœ), แƒแƒ’แƒ แƒ”แƒ—แƒ•แƒ” แƒแƒ˜แƒ แƒ˜แƒกแƒ”แƒ‘แƒ แƒ˜ แƒ“แƒ แƒ—แƒฎแƒ”แƒ•แƒแƒ“แƒ˜ แƒœแƒ˜แƒ•แƒ—แƒ˜แƒ”แƒ แƒ”แƒ‘แƒ”แƒ‘แƒ˜.

แƒฐแƒ”แƒขแƒ”แƒ แƒแƒคแƒแƒ–แƒ˜แƒก แƒ แƒ”แƒแƒฅแƒชแƒ˜แƒ”แƒ‘แƒ˜แƒก แƒ’แƒแƒœแƒกแƒแƒ™แƒฃแƒ—แƒ แƒ”แƒ‘แƒฃแƒšแƒ˜ แƒจแƒ”แƒ›แƒ—แƒฎแƒ•แƒ”แƒ•แƒแƒ แƒ›แƒงแƒแƒ แƒ˜ แƒคแƒแƒ–แƒ˜แƒก แƒ แƒ”แƒแƒฅแƒชแƒ˜แƒ”แƒ‘แƒ˜, แƒแƒœแƒฃ แƒ แƒ”แƒแƒฅแƒชแƒ˜แƒ”แƒ‘แƒ˜, แƒ แƒแƒ›แƒ”แƒšแƒจแƒ˜แƒช แƒงแƒ•แƒ”แƒšแƒ แƒ›แƒแƒœแƒแƒฌแƒ˜แƒšแƒ” แƒ›แƒงแƒแƒ แƒ˜แƒ.

แƒกแƒ˜แƒœแƒแƒ›แƒ“แƒ•แƒ˜แƒšแƒ”แƒจแƒ˜, แƒ’แƒแƒœแƒขแƒแƒšแƒ”แƒ‘แƒ (1) แƒ›แƒแƒฅแƒ›แƒ”แƒ“แƒ”แƒ‘แƒก แƒœแƒ”แƒ‘แƒ˜แƒกแƒ›แƒ˜แƒ”แƒ แƒ˜ แƒจแƒ”แƒฅแƒชแƒ”แƒ•แƒแƒ“แƒ˜ แƒ แƒ”แƒแƒฅแƒชแƒ˜แƒ˜แƒกแƒ—แƒ•แƒ˜แƒก, แƒ›แƒ˜แƒฃแƒฎแƒ”แƒ“แƒแƒ•แƒแƒ“ แƒ˜แƒ›แƒ˜แƒกแƒ, แƒ แƒแƒ›แƒ”แƒš แƒฉแƒแƒ›แƒแƒ—แƒ•แƒšแƒ˜แƒš แƒฏแƒ’แƒฃแƒคแƒก แƒ›แƒ˜แƒ”แƒ™แƒฃแƒ—แƒ•แƒœแƒ”แƒ‘แƒ แƒ˜แƒ’แƒ˜. แƒ›แƒแƒ’แƒ แƒแƒ› แƒฐแƒ”แƒขแƒ”แƒ แƒแƒคแƒแƒ–แƒ˜แƒก แƒ แƒ”แƒแƒฅแƒชแƒ˜แƒแƒจแƒ˜, แƒœแƒ˜แƒ•แƒ—แƒ˜แƒ”แƒ แƒ”แƒ‘แƒ”แƒ‘แƒ˜แƒก แƒฌแƒแƒœแƒแƒกแƒฌแƒแƒ แƒฃแƒšแƒ˜ แƒ™แƒแƒœแƒชแƒ”แƒœแƒขแƒ แƒแƒชแƒ˜แƒ”แƒ‘แƒ˜ แƒฃแƒคแƒ แƒ แƒ›แƒแƒฌแƒ”แƒกแƒ แƒ˜แƒ’แƒ”แƒ‘แƒฃแƒš แƒคแƒแƒ–แƒแƒจแƒ˜ แƒแƒ แƒ˜แƒก แƒ›แƒฃแƒ“แƒ›แƒ˜แƒ•แƒ˜ แƒ“แƒ แƒจแƒ”แƒ˜แƒซแƒšแƒ”แƒ‘แƒ แƒ’แƒแƒ”แƒ แƒ—แƒ˜แƒแƒœแƒ“แƒ”แƒก แƒฌแƒแƒœแƒแƒกแƒฌแƒแƒ แƒแƒ‘แƒ˜แƒก แƒ›แƒฃแƒ“แƒ›แƒ˜แƒ•แƒจแƒ˜ (แƒ˜แƒฎ. แƒ—แƒแƒ•แƒ˜ 9 ยง 5).

แƒแƒกแƒ” แƒ แƒแƒ›, แƒฐแƒ”แƒขแƒ”แƒ แƒแƒคแƒแƒ–แƒ˜แƒก แƒ แƒ”แƒแƒฅแƒชแƒ˜แƒ˜แƒกแƒ—แƒ•แƒ˜แƒก

แƒ A g+ แƒ‘ B แƒ™แƒ  แƒ“ D r+ แƒ• F แƒ™แƒ 

แƒ›แƒ˜แƒ›แƒแƒ แƒ—แƒ”แƒ‘แƒ˜แƒ— แƒ’แƒแƒ›แƒแƒ˜แƒกแƒแƒฎแƒ”แƒ‘แƒ แƒ›แƒแƒกแƒแƒ‘แƒ แƒ˜แƒ•แƒ˜ แƒ›แƒแƒฅแƒ›แƒ”แƒ“แƒ”แƒ‘แƒ˜แƒก แƒ™แƒแƒœแƒแƒœแƒ˜

แƒแƒ› แƒ—แƒแƒœแƒแƒคแƒแƒ แƒ“แƒแƒ‘แƒ˜แƒก แƒขแƒ˜แƒžแƒ˜ แƒ“แƒแƒ›แƒแƒ™แƒ˜แƒ“แƒ”แƒ‘แƒฃแƒšแƒ˜แƒ แƒ˜แƒ›แƒแƒ–แƒ”, แƒ—แƒฃ แƒ แƒแƒ›แƒ”แƒšแƒ˜ แƒœแƒ˜แƒ•แƒ—แƒ˜แƒ”แƒ แƒ”แƒ‘แƒ”แƒ‘แƒ˜ แƒ›แƒแƒœแƒแƒฌแƒ˜แƒšแƒ”แƒแƒ‘แƒ”แƒœ แƒ แƒ”แƒแƒฅแƒชแƒ˜แƒแƒจแƒ˜ แƒ›แƒงแƒแƒ  แƒแƒœ แƒ—แƒฎแƒ”แƒ•แƒแƒ“ แƒ›แƒ“แƒ’แƒแƒ›แƒแƒ แƒ”แƒแƒ‘แƒแƒจแƒ˜ (แƒ—แƒฎแƒ”แƒ•แƒแƒ“แƒ˜, แƒ—แƒฃ แƒ“แƒแƒœแƒแƒ แƒฉแƒ”แƒœแƒ˜ แƒœแƒ˜แƒ•แƒ—แƒ˜แƒ”แƒ แƒ”แƒ‘แƒ”แƒ‘แƒ˜ แƒแƒ˜แƒ แƒ”แƒ‘แƒ˜แƒ).

แƒ›แƒแƒกแƒ˜แƒก แƒ›แƒแƒฅแƒ›แƒ”แƒ“แƒ”แƒ‘แƒ˜แƒก แƒ™แƒแƒœแƒแƒœแƒ˜แƒก (1) แƒ“แƒ (2) แƒ’แƒแƒ›แƒแƒœแƒแƒ—แƒฅแƒ•แƒแƒ›แƒ”แƒ‘แƒจแƒ˜ แƒ›แƒแƒšแƒ”แƒ™แƒฃแƒšแƒ”แƒ‘แƒ˜แƒก แƒแƒœ แƒ˜แƒแƒœแƒ”แƒ‘แƒ˜แƒก แƒคแƒแƒ แƒ›แƒฃแƒšแƒ”แƒ‘แƒ˜ แƒ™แƒ•แƒแƒ“แƒ แƒแƒขแƒฃแƒš แƒคแƒ แƒฉแƒฎแƒ˜แƒšแƒ”แƒ‘แƒจแƒ˜ แƒœแƒ˜แƒจแƒœแƒแƒ•แƒก แƒแƒ› แƒœแƒแƒฌแƒ˜แƒšแƒแƒ™แƒ”แƒ‘แƒ˜แƒก แƒฌแƒแƒœแƒแƒกแƒฌแƒแƒ แƒฃแƒš แƒ™แƒแƒœแƒชแƒ”แƒœแƒขแƒ แƒแƒชแƒ˜แƒแƒก แƒ’แƒแƒ–แƒจแƒ˜ แƒแƒœ แƒฎแƒกแƒœแƒแƒ แƒจแƒ˜. แƒแƒ› แƒจแƒ”แƒ›แƒ—แƒฎแƒ•แƒ”แƒ•แƒแƒจแƒ˜, แƒ™แƒแƒœแƒชแƒ”แƒœแƒขแƒ แƒแƒชแƒ˜แƒ”แƒ‘แƒ˜ แƒแƒ  แƒฃแƒœแƒ“แƒ แƒ˜แƒงแƒแƒก แƒ“แƒ˜แƒ“แƒ˜ (แƒแƒ แƒแƒฃแƒ›แƒ”แƒขแƒ”แƒก 0,1 แƒ›แƒแƒš/แƒš), แƒ•แƒ˜แƒœแƒแƒ˜แƒ“แƒแƒœ แƒ”แƒก แƒ™แƒแƒ”แƒคแƒ˜แƒชแƒ˜แƒ”แƒœแƒขแƒ”แƒ‘แƒ˜ แƒ›แƒแƒฅแƒ›แƒ”แƒ“แƒ”แƒ‘แƒก แƒ›แƒฎแƒแƒšแƒแƒ“ แƒ˜แƒ“แƒ”แƒแƒšแƒฃแƒ แƒ˜ แƒ’แƒแƒ–แƒ”แƒ‘แƒ˜แƒกแƒ แƒ“แƒ แƒ˜แƒ“แƒ”แƒแƒšแƒฃแƒ แƒ˜ แƒฎแƒกแƒœแƒแƒ แƒ”แƒ‘แƒ˜แƒกแƒ—แƒ•แƒ˜แƒก. (แƒ›แƒแƒฆแƒแƒšแƒ˜ แƒ™แƒแƒœแƒชแƒ”แƒœแƒขแƒ แƒแƒชแƒ˜แƒ˜แƒก แƒ“แƒ แƒแƒก แƒ›แƒแƒกแƒ˜แƒก แƒ›แƒแƒฅแƒ›แƒ”แƒ“แƒ”แƒ‘แƒ˜แƒก แƒ™แƒแƒœแƒแƒœแƒ˜ แƒซแƒแƒšแƒแƒจแƒ˜ แƒ แƒฉแƒ”แƒ‘แƒ, แƒ›แƒแƒ’แƒ แƒแƒ› แƒ™แƒแƒœแƒชแƒ”แƒœแƒขแƒ แƒแƒชแƒ˜แƒ˜แƒก แƒœแƒแƒชแƒ•แƒšแƒแƒ“ แƒฃแƒœแƒ“แƒ แƒ’แƒแƒ›แƒแƒ•แƒ˜แƒงแƒ”แƒœแƒแƒ— แƒกแƒฎแƒ•แƒ แƒคแƒ˜แƒ–แƒ˜แƒ™แƒฃแƒ แƒ˜ แƒ แƒแƒแƒ“แƒ”แƒœแƒแƒ‘แƒ (แƒ”.แƒฌ. แƒแƒฅแƒขแƒ˜แƒ•แƒแƒ‘แƒ), แƒ แƒแƒ›แƒ”แƒšแƒ˜แƒช แƒ˜แƒ—แƒ•แƒแƒšแƒ˜แƒกแƒฌแƒ˜แƒœแƒ”แƒ‘แƒก แƒ’แƒแƒ–แƒ˜แƒก แƒœแƒแƒฌแƒ˜แƒšแƒแƒ™แƒ”แƒ‘แƒก แƒแƒœ แƒฎแƒกแƒœแƒแƒ แƒ”แƒ‘แƒก แƒจแƒแƒ แƒ˜แƒก แƒฃแƒ แƒ—แƒ˜แƒ”แƒ แƒ—แƒฅแƒ›แƒ”แƒ“แƒ”แƒ‘แƒแƒก. แƒแƒฅแƒขแƒ˜แƒ•แƒแƒ‘แƒ แƒแƒ  แƒแƒ แƒ˜แƒก แƒ™แƒแƒœแƒชแƒ”แƒœแƒขแƒ แƒแƒชแƒ˜แƒ˜แƒก แƒžแƒ แƒแƒžแƒแƒ แƒชแƒ˜แƒฃแƒšแƒ˜. ).

แƒ›แƒแƒกแƒ˜แƒก แƒ›แƒแƒฅแƒ›แƒ”แƒ“แƒ”แƒ‘แƒ˜แƒก แƒ™แƒแƒœแƒแƒœแƒ˜ แƒ’แƒแƒ›แƒแƒ˜แƒงแƒ”แƒœแƒ”แƒ‘แƒ แƒแƒ แƒ แƒ›แƒฎแƒแƒšแƒแƒ“ แƒจแƒ”แƒฅแƒชแƒ”แƒ•แƒแƒ“แƒ˜ แƒฅแƒ˜แƒ›แƒ˜แƒฃแƒ แƒ˜ แƒ แƒ”แƒแƒฅแƒชแƒ˜แƒ”แƒ‘แƒ˜แƒกแƒ—แƒ•แƒ˜แƒก, แƒแƒ แƒแƒ›แƒ”แƒ“ แƒ›แƒ แƒแƒ•แƒแƒšแƒ˜ แƒจแƒ”แƒฅแƒชแƒ”แƒ•แƒแƒ“แƒ˜ แƒคแƒ˜แƒ–แƒ˜แƒ™แƒฃแƒ แƒ˜ แƒžแƒ แƒแƒชแƒ”แƒกแƒ˜แƒช แƒ”แƒ›แƒแƒ แƒฉแƒ˜แƒšแƒ”แƒ‘แƒ แƒ›แƒแƒก, แƒ›แƒแƒ’แƒแƒšแƒ˜แƒ—แƒแƒ“, แƒชแƒแƒšแƒ™แƒ”แƒฃแƒšแƒ˜ แƒœแƒ˜แƒ•แƒ—แƒ˜แƒ”แƒ แƒ”แƒ‘แƒ”แƒ‘แƒ˜แƒก แƒจแƒฃแƒแƒšแƒ”แƒ“แƒฃแƒ แƒ˜ แƒฌแƒแƒœแƒแƒกแƒฌแƒแƒ แƒแƒ‘แƒ แƒแƒ’แƒ แƒ”แƒ’แƒแƒชแƒ˜แƒ˜แƒก แƒ”แƒ แƒ—แƒ˜ แƒ›แƒ“แƒ’แƒแƒ›แƒแƒ แƒ”แƒแƒ‘แƒ˜แƒ“แƒแƒœ แƒ›แƒ”แƒแƒ แƒ”แƒจแƒ˜ แƒ’แƒแƒ“แƒแƒกแƒ•แƒšแƒ˜แƒกแƒแƒก. แƒแƒกแƒ” แƒ แƒแƒ›, แƒแƒแƒ แƒ—แƒฅแƒšแƒ”แƒ‘แƒ˜แƒก แƒจแƒ”แƒฅแƒชแƒ”แƒ•แƒแƒ“แƒ˜ แƒžแƒ แƒแƒชแƒ”แƒกแƒ˜ - แƒฌแƒงแƒšแƒ˜แƒก แƒ™แƒแƒœแƒ“แƒ”แƒœแƒกแƒแƒชแƒ˜แƒ แƒจแƒ”แƒ˜แƒซแƒšแƒ”แƒ‘แƒ แƒ’แƒแƒ›แƒแƒ˜แƒกแƒแƒฎแƒแƒก แƒ’แƒแƒœแƒขแƒแƒšแƒ”แƒ‘แƒ˜แƒ—

H 2 O f H 2 O แƒ’

แƒแƒ› แƒžแƒ แƒแƒชแƒ”แƒกแƒ˜แƒกแƒ—แƒ•แƒ˜แƒก แƒฉแƒ•แƒ”แƒœ แƒจแƒ”แƒ’แƒ•แƒ˜แƒซแƒšแƒ˜แƒ แƒ“แƒแƒ•แƒฌแƒ”แƒ แƒแƒ— แƒฌแƒแƒœแƒแƒกแƒฌแƒแƒ แƒแƒ‘แƒ˜แƒก แƒ›แƒฃแƒ“แƒ›แƒ˜แƒ•แƒ˜แƒก แƒ’แƒแƒœแƒขแƒแƒšแƒ”แƒ‘แƒ:

แƒจแƒ”แƒ“แƒ”แƒ’แƒแƒ“ แƒ›แƒ˜แƒฆแƒ”แƒ‘แƒฃแƒšแƒ˜ แƒ—แƒแƒœแƒแƒคแƒแƒ แƒ“แƒแƒ‘แƒ แƒแƒ“แƒแƒกแƒขแƒฃแƒ แƒ”แƒ‘แƒก, แƒ™แƒ”แƒ แƒซแƒแƒ“, แƒคแƒ˜แƒ–แƒ˜แƒ™แƒ˜แƒ“แƒแƒœ แƒ—แƒฅแƒ•แƒ”แƒœแƒ—แƒ•แƒ˜แƒก แƒชแƒœแƒแƒ‘แƒ˜แƒšแƒ˜ แƒ›แƒขแƒ™แƒ˜แƒชแƒ”แƒ‘แƒแƒก, แƒ แƒแƒ› แƒฐแƒแƒ”แƒ แƒ˜แƒก แƒขแƒ”แƒœแƒ˜แƒแƒœแƒแƒ‘แƒ แƒ“แƒแƒ›แƒแƒ™แƒ˜แƒ“แƒ”แƒ‘แƒฃแƒšแƒ˜แƒ แƒขแƒ”แƒ›แƒžแƒ”แƒ แƒแƒขแƒฃแƒ แƒแƒกแƒ แƒ“แƒ แƒฌแƒœแƒ”แƒ•แƒแƒ–แƒ”.

20.2. แƒแƒ•แƒขแƒแƒžแƒ แƒแƒขแƒแƒšแƒ˜แƒ–แƒ˜แƒก แƒ›แƒฃแƒ“แƒ›แƒ˜แƒ•แƒ˜ (แƒ˜แƒแƒœแƒฃแƒ แƒ˜ แƒžแƒ แƒแƒ“แƒฃแƒฅแƒขแƒ˜)

แƒ—แƒฅแƒ•แƒ”แƒœแƒ—แƒ•แƒ˜แƒก แƒชแƒœแƒแƒ‘แƒ˜แƒšแƒ˜ แƒ›แƒแƒกแƒแƒ‘แƒ แƒ˜แƒ•แƒ˜ แƒ›แƒแƒฅแƒ›แƒ”แƒ“แƒ”แƒ‘แƒ˜แƒก แƒ™แƒแƒœแƒแƒœแƒ˜แƒก แƒ™แƒ˜แƒ“แƒ”แƒ• แƒ”แƒ แƒ—แƒ˜ แƒ’แƒแƒ›แƒแƒงแƒ”แƒœแƒ”แƒ‘แƒ แƒแƒ แƒ˜แƒก แƒแƒ•แƒขแƒแƒžแƒ แƒแƒขแƒแƒšแƒ˜แƒ–แƒ˜แƒก แƒ แƒแƒแƒ“แƒ”แƒœแƒแƒ‘แƒ แƒ˜แƒ•แƒ˜ แƒแƒฆแƒฌแƒ”แƒ แƒ (แƒ—แƒแƒ•แƒ˜ X ยง 5). แƒ˜แƒชแƒ˜แƒ— แƒ—แƒฃ แƒแƒ แƒ, แƒ แƒแƒ› แƒกแƒฃแƒคแƒ—แƒ แƒฌแƒงแƒแƒšแƒ˜ แƒฐแƒแƒ›แƒแƒคแƒแƒ–แƒ˜แƒก แƒฌแƒแƒœแƒแƒกแƒฌแƒแƒ แƒแƒ‘แƒแƒจแƒ˜แƒ?

2H 2 OH 3 O + + OH -

แƒ แƒแƒ›แƒšแƒ˜แƒก แƒ แƒแƒแƒ“แƒ”แƒœแƒแƒ‘แƒ แƒ˜แƒ•แƒ˜ แƒแƒฆแƒฌแƒ”แƒ แƒ˜แƒกแƒ—แƒ•แƒ˜แƒก แƒจแƒ”แƒ’แƒ˜แƒซแƒšแƒ˜แƒแƒ— แƒ’แƒแƒ›แƒแƒ˜แƒงแƒ”แƒœแƒแƒ— แƒ›แƒแƒกแƒแƒ‘แƒ แƒ˜แƒ•แƒ˜ แƒ›แƒแƒฅแƒ›แƒ”แƒ“แƒ”แƒ‘แƒ˜แƒก แƒ™แƒแƒœแƒแƒœแƒ˜, แƒ แƒแƒ›แƒšแƒ˜แƒก แƒ›แƒแƒ—แƒ”แƒ›แƒแƒขแƒ˜แƒ™แƒฃแƒ แƒ˜ แƒ’แƒแƒ›แƒแƒฎแƒแƒขแƒฃแƒšแƒ”แƒ‘แƒแƒ แƒแƒ•แƒขแƒแƒžแƒ แƒแƒขแƒแƒšแƒ˜แƒ–แƒ˜แƒก แƒ›แƒฃแƒ“แƒ›แƒ˜แƒ•แƒ˜แƒฌแƒงแƒšแƒ˜แƒก (แƒ˜แƒแƒœแƒฃแƒ แƒ˜ แƒžแƒ แƒแƒ“แƒฃแƒฅแƒขแƒ˜).

แƒแƒ•แƒขแƒแƒžแƒ แƒแƒขแƒแƒšแƒ˜แƒ–แƒ˜ แƒ“แƒแƒ›แƒแƒฎแƒแƒกแƒ˜แƒแƒ—แƒ”แƒ‘แƒ”แƒšแƒ˜แƒ แƒแƒ แƒ แƒ›แƒฎแƒแƒšแƒแƒ“ แƒฌแƒงแƒšแƒ˜แƒกแƒ—แƒ•แƒ˜แƒก, แƒแƒ แƒแƒ›แƒ”แƒ“ แƒ›แƒ แƒแƒ•แƒแƒšแƒ˜ แƒกแƒฎแƒ•แƒ แƒกแƒ˜แƒ—แƒฎแƒ”แƒ”แƒ‘แƒ˜แƒกแƒ—แƒ•แƒ˜แƒก, แƒ แƒแƒ›แƒ”แƒšแƒ—แƒ แƒ›แƒแƒšแƒ”แƒ™แƒฃแƒšแƒ”แƒ‘แƒ˜ แƒ”แƒ แƒ—แƒ›แƒแƒœแƒ”แƒ—แƒ—แƒแƒœ แƒแƒ แƒ˜แƒก แƒ“แƒแƒ™แƒแƒ•แƒจแƒ˜แƒ แƒ”แƒ‘แƒฃแƒšแƒ˜ แƒฌแƒงแƒแƒšแƒ‘แƒแƒ“แƒ˜แƒก แƒ‘แƒ›แƒ”แƒ‘แƒ˜แƒ—, แƒ›แƒแƒ’แƒแƒšแƒ˜แƒ—แƒแƒ“, แƒแƒ›แƒ˜แƒแƒ™แƒ˜แƒก, แƒ›แƒ”แƒ—แƒแƒœแƒแƒšแƒ˜แƒกแƒ แƒ“แƒ แƒฌแƒงแƒแƒšแƒ‘แƒแƒ“แƒ˜แƒก แƒคแƒขแƒแƒ แƒ˜แƒ“แƒ˜แƒกแƒ—แƒ•แƒ˜แƒก:

2NH 3 NH 4 + + NH 2 - แƒ™(NH 3) = 1.91. 10 โ€“33 (-50 o ะก-แƒ–แƒ”);
2CH 3 OH CH 3 OH 2 + + CH 3 O - แƒ™(CH 3 OH) = 4.90. 10โ€“18 (25 o C-แƒ–แƒ”);
2HF H 2 F + + F - แƒ™(HF) = 2.00 . 10โ€“12 (0 o C-แƒ–แƒ”).

แƒแƒ› แƒ“แƒ แƒ›แƒ แƒแƒ•แƒแƒšแƒ˜ แƒกแƒฎแƒ•แƒ แƒœแƒ˜แƒ•แƒ—แƒ˜แƒ”แƒ แƒ”แƒ‘แƒ˜แƒกแƒ—แƒ•แƒ˜แƒก แƒชแƒœแƒแƒ‘แƒ˜แƒšแƒ˜แƒ แƒแƒ•แƒขแƒแƒžแƒ แƒแƒขแƒแƒšแƒ˜แƒ–แƒ˜แƒก แƒ›แƒฃแƒ“แƒ›แƒ˜แƒ•แƒ”แƒ‘แƒ˜, แƒ แƒแƒ›แƒšแƒ”แƒ‘แƒ˜แƒช แƒ›แƒฎแƒ”แƒ“แƒ•แƒ”แƒšแƒแƒ‘แƒแƒจแƒ˜ แƒ›แƒ˜แƒ˜แƒฆแƒ”แƒ‘แƒ แƒกแƒฎแƒ•แƒแƒ“แƒแƒกแƒฎแƒ•แƒ แƒฅแƒ˜แƒ›แƒ˜แƒฃแƒ แƒ˜ แƒ แƒ”แƒแƒฅแƒชแƒ˜แƒ˜แƒกแƒ—แƒ•แƒ˜แƒก แƒ’แƒแƒ›แƒฎแƒกแƒœแƒ”แƒšแƒ˜แƒก แƒแƒ แƒฉแƒ”แƒ•แƒ˜แƒกแƒแƒก.

แƒกแƒ˜แƒ›แƒ‘แƒแƒšแƒ แƒฎแƒจแƒ˜แƒ แƒแƒ“ แƒ’แƒแƒ›แƒแƒ˜แƒงแƒ”แƒœแƒ”แƒ‘แƒ แƒแƒ•แƒขแƒแƒžแƒ แƒแƒขแƒแƒšแƒ˜แƒ–แƒ˜แƒก แƒ›แƒฃแƒ“แƒ›แƒ˜แƒ•แƒ˜แƒก แƒแƒฆแƒกแƒแƒœแƒ˜แƒจแƒœแƒแƒ•แƒแƒ“ แƒ™ แƒก.

แƒแƒ•แƒขแƒแƒžแƒ แƒแƒขแƒแƒšแƒ˜แƒ–แƒ˜แƒก แƒ›แƒฃแƒ“แƒ›แƒ˜แƒ•แƒ˜ แƒแƒ  แƒแƒ แƒ˜แƒก แƒ“แƒแƒ›แƒแƒ™แƒ˜แƒ“แƒ”แƒ‘แƒฃแƒšแƒ˜ แƒ—แƒ”แƒแƒ แƒ˜แƒแƒ–แƒ”, แƒ แƒแƒ›แƒ”แƒšแƒจแƒ˜แƒช แƒ’แƒแƒœแƒ˜แƒฎแƒ˜แƒšแƒ”แƒ‘แƒ แƒแƒ•แƒขแƒแƒžแƒ แƒแƒขแƒแƒšแƒ˜แƒ–แƒ˜. แƒฌแƒแƒœแƒแƒกแƒฌแƒแƒ แƒแƒ‘แƒ˜แƒก แƒ›แƒฃแƒ“แƒ›แƒ˜แƒ•แƒ”แƒ‘แƒ˜แƒก แƒ›แƒœแƒ˜แƒจแƒ•แƒœแƒ”แƒšแƒแƒ‘แƒ”แƒ‘แƒ˜, แƒžแƒ˜แƒ แƒ˜แƒฅแƒ˜แƒ—, แƒ“แƒแƒ›แƒแƒ™แƒ˜แƒ“แƒ”แƒ‘แƒฃแƒšแƒ˜แƒ แƒ›แƒ˜แƒฆแƒ”แƒ‘แƒฃแƒš แƒ›แƒแƒ“แƒ”แƒšแƒ–แƒ”. แƒฉแƒ•แƒ”แƒœ แƒแƒ›แƒแƒก แƒ“แƒแƒ•แƒแƒ›แƒแƒฌแƒ›แƒ”แƒ‘แƒ— แƒฌแƒงแƒšแƒ˜แƒก แƒแƒ•แƒขแƒแƒžแƒ แƒแƒขแƒแƒšแƒ˜แƒ–แƒ˜แƒก แƒแƒฆแƒฌแƒ”แƒ แƒ˜แƒšแƒแƒ‘แƒ˜แƒก แƒจแƒ”แƒ“แƒแƒ แƒ”แƒ‘แƒ˜แƒ— แƒžแƒ แƒแƒขแƒแƒšแƒ˜แƒ–แƒฃแƒ แƒ˜ แƒ—แƒ”แƒแƒ แƒ˜แƒ˜แƒก (แƒ›แƒแƒ แƒชแƒฎแƒ”แƒœแƒ แƒกแƒ•แƒ”แƒขแƒ˜) แƒ“แƒ แƒ”แƒšแƒ”แƒฅแƒขแƒ แƒแƒšแƒ˜แƒขแƒฃแƒ แƒ˜ แƒ“แƒ˜แƒกแƒแƒชแƒ˜แƒแƒชแƒ˜แƒ˜แƒก แƒ›แƒแƒซแƒ•แƒ”แƒšแƒ”แƒ‘แƒฃแƒšแƒ˜, แƒ›แƒแƒ’แƒ แƒแƒ› แƒฏแƒ”แƒ  แƒ™แƒ˜แƒ“แƒ”แƒ• แƒคแƒแƒ แƒ—แƒแƒ“ แƒ’แƒแƒ›แƒแƒงแƒ”แƒœแƒ”แƒ‘แƒฃแƒšแƒ˜ แƒ—แƒ”แƒแƒ แƒ˜แƒ˜แƒก แƒ›แƒ˜แƒฎแƒ”แƒ“แƒ•แƒ˜แƒ— (แƒ›แƒแƒ แƒฏแƒ•แƒ”แƒœแƒ แƒกแƒ•แƒ”แƒขแƒ˜):

แƒ”แƒšแƒ”แƒฅแƒขแƒ แƒแƒšแƒ˜แƒขแƒฃแƒ แƒ˜ แƒ“แƒ˜แƒกแƒแƒชแƒ˜แƒแƒชแƒ˜แƒ˜แƒก แƒ—แƒ”แƒแƒ แƒ˜แƒ˜แƒก แƒ›แƒ˜แƒฎแƒ”แƒ“แƒ•แƒ˜แƒ—, แƒ•แƒแƒ แƒแƒฃแƒ“แƒแƒ‘แƒ“แƒœแƒ”แƒœ, แƒ แƒแƒ› แƒฌแƒงแƒšแƒ˜แƒก แƒ›แƒแƒšแƒ”แƒ™แƒฃแƒšแƒ”แƒ‘แƒ˜ แƒœแƒแƒฌแƒ˜แƒšแƒแƒ‘แƒ แƒ˜แƒ• แƒ˜แƒจแƒšแƒ”แƒ‘แƒ (แƒ˜แƒฎแƒ แƒฌแƒœแƒ”แƒ‘แƒ) แƒฌแƒงแƒแƒšแƒ‘แƒแƒ“แƒ˜แƒก แƒ˜แƒแƒœแƒ”แƒ‘แƒแƒ“ แƒ“แƒ แƒฐแƒ˜แƒ“แƒ แƒแƒฅแƒกแƒ˜แƒ“แƒ˜แƒก แƒ˜แƒแƒœแƒ”แƒ‘แƒแƒ“. แƒ—แƒ”แƒแƒ แƒ˜แƒ แƒแƒ  แƒฎแƒกแƒœแƒ˜แƒ“แƒ แƒแƒ› โ€žแƒ“แƒแƒจแƒšแƒ˜แƒกโ€œ แƒแƒ แƒช แƒ›แƒ˜แƒ–แƒ”แƒ–แƒ”แƒ‘แƒก แƒ“แƒ แƒแƒ แƒช แƒ›แƒ”แƒฅแƒแƒœแƒ˜แƒ–แƒ›แƒก. แƒกแƒแƒฎแƒ”แƒšแƒฌแƒแƒ“แƒ”แƒ‘แƒ "แƒแƒ•แƒขแƒแƒžแƒ แƒแƒขแƒแƒšแƒ˜แƒ–แƒ˜แƒก แƒ›แƒฃแƒ“แƒ›แƒ˜แƒ•แƒ˜" แƒฉแƒ•แƒ”แƒฃแƒšแƒ”แƒ‘แƒ แƒ˜แƒ• แƒ’แƒแƒ›แƒแƒ˜แƒงแƒ”แƒœแƒ”แƒ‘แƒ แƒžแƒ แƒแƒขแƒแƒšแƒ˜แƒ–แƒฃแƒ  แƒ—แƒ”แƒแƒ แƒ˜แƒแƒจแƒ˜, แƒฎแƒแƒšแƒ "แƒ˜แƒแƒœแƒฃแƒ แƒ˜ แƒžแƒ แƒแƒ“แƒฃแƒฅแƒขแƒ˜" แƒ”แƒšแƒ”แƒฅแƒขแƒ แƒแƒšแƒ˜แƒขแƒฃแƒ แƒ˜ แƒ“แƒ˜แƒกแƒแƒชแƒ˜แƒแƒชแƒ˜แƒ˜แƒก แƒ—แƒ”แƒแƒ แƒ˜แƒแƒจแƒ˜.

20.3. แƒ›แƒŸแƒแƒ•แƒ˜แƒแƒœแƒแƒ‘แƒ˜แƒก แƒ“แƒ แƒ‘แƒแƒ–แƒ˜แƒกแƒฃแƒ แƒแƒ‘แƒ˜แƒก แƒ›แƒฃแƒ“แƒ›แƒ˜แƒ•แƒ”แƒ‘แƒ˜. แƒฌแƒงแƒแƒšแƒ‘แƒแƒ“แƒ˜แƒก แƒ›แƒแƒฉแƒ•แƒ”แƒœแƒ”แƒ‘แƒ”แƒšแƒ˜

แƒ›แƒแƒกแƒ˜แƒก แƒ›แƒแƒฅแƒ›แƒ”แƒ“แƒ”แƒ‘แƒ˜แƒก แƒ™แƒแƒœแƒแƒœแƒ˜ แƒแƒกแƒ”แƒ•แƒ” แƒ’แƒแƒ›แƒแƒ˜แƒงแƒ”แƒœแƒ”แƒ‘แƒ แƒกแƒฎแƒ•แƒแƒ“แƒแƒกแƒฎแƒ•แƒ แƒœแƒ˜แƒ•แƒ—แƒ˜แƒ”แƒ แƒ”แƒ‘แƒ˜แƒก แƒ›แƒŸแƒแƒ•แƒ-แƒขแƒฃแƒขแƒแƒ•แƒแƒœแƒ˜ แƒ—แƒ•แƒ˜แƒกแƒ”แƒ‘แƒ”แƒ‘แƒ˜แƒก แƒ’แƒแƒกแƒแƒ–แƒแƒ›แƒแƒ“. แƒžแƒ แƒแƒขแƒแƒšแƒ˜แƒ–แƒฃแƒ  แƒ—แƒ”แƒแƒ แƒ˜แƒแƒจแƒ˜ แƒแƒ›แƒ˜แƒกแƒแƒ—แƒ•แƒ˜แƒก แƒ’แƒแƒ›แƒแƒ˜แƒงแƒ”แƒœแƒ”แƒ‘แƒ แƒ›แƒŸแƒแƒ•แƒ˜แƒแƒœแƒแƒ‘แƒ˜แƒก แƒ“แƒ แƒคแƒฃแƒซแƒ”แƒแƒ‘แƒ˜แƒก แƒ›แƒฃแƒ“แƒ›แƒ˜แƒ•แƒ”แƒ‘แƒ˜, แƒฎแƒแƒšแƒ แƒ”แƒšแƒ”แƒฅแƒขแƒ แƒแƒšแƒ˜แƒขแƒฃแƒ แƒ˜ แƒ“แƒ˜แƒกแƒแƒชแƒ˜แƒแƒชแƒ˜แƒ˜แƒก แƒ—แƒ”แƒแƒ แƒ˜แƒแƒจแƒ˜ - แƒ“แƒ˜แƒกแƒแƒชแƒ˜แƒแƒชแƒ˜แƒ˜แƒก แƒ›แƒฃแƒ“แƒ›แƒ˜แƒ•แƒ”แƒ‘แƒ˜.

แƒ แƒแƒ’แƒแƒ  แƒฎแƒกแƒœแƒ˜แƒก แƒžแƒ แƒแƒขแƒแƒšแƒ˜แƒ–แƒฃแƒ แƒ˜ แƒ—แƒ”แƒแƒ แƒ˜แƒ แƒฅแƒ˜แƒ›แƒ˜แƒ™แƒแƒขแƒ”แƒ‘แƒ˜แƒก แƒ›แƒŸแƒแƒ•แƒ-แƒขแƒฃแƒขแƒแƒ•แƒแƒœ แƒ—แƒ•แƒ˜แƒกแƒ”แƒ‘แƒ”แƒ‘แƒก, แƒ—แƒฅแƒ•แƒ”แƒœ แƒฃแƒ™แƒ•แƒ” แƒ˜แƒชแƒ˜แƒ— (แƒ—. XII ยง 4). แƒ›แƒแƒ“แƒ˜แƒ— แƒจแƒ”แƒ•แƒแƒ“แƒแƒ แƒแƒ— แƒ”แƒก แƒ›แƒ˜แƒ“แƒ’แƒแƒ›แƒ แƒ”แƒšแƒ”แƒฅแƒขแƒ แƒแƒšแƒ˜แƒขแƒฃแƒ แƒ˜ แƒ“แƒ˜แƒกแƒแƒชแƒ˜แƒแƒชแƒ˜แƒ˜แƒก แƒ—แƒ”แƒแƒ แƒ˜แƒ˜แƒก แƒ›แƒ˜แƒ“แƒ’แƒแƒ›แƒแƒก แƒจแƒ”แƒฅแƒชแƒ”แƒ•แƒแƒ“แƒ˜ แƒฐแƒแƒ›แƒแƒคแƒแƒ–แƒ˜แƒก แƒ แƒ”แƒแƒฅแƒชแƒ˜แƒ˜แƒก แƒ›แƒแƒ’แƒแƒšแƒ˜แƒ—แƒ˜แƒก แƒ’แƒแƒ›แƒแƒงแƒ”แƒœแƒ”แƒ‘แƒ˜แƒ— แƒฐแƒ˜แƒ“แƒ แƒแƒชแƒ˜แƒแƒœแƒ›แƒŸแƒแƒ•แƒแƒก HCN แƒฌแƒงแƒแƒšแƒ—แƒแƒœ, แƒกแƒฃแƒกแƒขแƒ˜ แƒ›แƒŸแƒแƒ•แƒ˜แƒ— (แƒ›แƒแƒ แƒชแƒฎแƒœแƒ˜แƒ• - แƒžแƒ แƒแƒขแƒแƒšแƒ˜แƒ–แƒฃแƒ แƒ˜ แƒ—แƒ”แƒแƒ แƒ˜แƒ˜แƒก แƒ›แƒ˜แƒฎแƒ”แƒ“แƒ•แƒ˜แƒ—, แƒ›แƒแƒ แƒฏแƒ•แƒœแƒ˜แƒ• - แƒ—แƒ”แƒแƒ แƒ˜แƒ˜แƒก แƒ›แƒ˜แƒฎแƒ”แƒ“แƒ•แƒ˜แƒ— แƒ”แƒšแƒ”แƒฅแƒขแƒ แƒแƒšแƒ˜แƒขแƒฃแƒ แƒ˜ แƒ“แƒ˜แƒกแƒแƒชแƒ˜แƒแƒชแƒ˜แƒ˜แƒก):

HCN + H 2 O H 3 O + + CN -

แƒ™แƒ™(HCN) = K C. == 4.93. 10โ€“10 แƒ›แƒแƒš/แƒš

HCN H + + CN โ€“

แƒฌแƒแƒœแƒแƒกแƒฌแƒแƒ แƒแƒ‘แƒ˜แƒก แƒ›แƒฃแƒ“แƒ›แƒ˜แƒ•แƒ˜ K Cแƒแƒ› แƒจแƒ”แƒ›แƒ—แƒฎแƒ•แƒ”แƒ•แƒแƒจแƒ˜ แƒ”แƒฌแƒแƒ“แƒ”แƒ‘แƒ แƒ“แƒ˜แƒกแƒแƒชแƒ˜แƒแƒชแƒ˜แƒ˜แƒก แƒ›แƒฃแƒ“แƒ›แƒ˜แƒ•แƒ˜(แƒแƒœ แƒ˜แƒแƒœแƒ˜แƒ–แƒแƒชแƒ˜แƒ˜แƒก แƒ›แƒฃแƒ“แƒ›แƒ˜แƒ•แƒ˜), แƒแƒฆแƒ˜แƒœแƒ˜แƒจแƒœแƒ”แƒ‘แƒ TOแƒ“แƒ แƒฃแƒ“แƒ แƒ˜แƒก แƒžแƒ แƒแƒขแƒแƒšแƒ˜แƒ–แƒฃแƒ  แƒ—แƒ”แƒแƒ แƒ˜แƒแƒจแƒ˜ แƒ›แƒŸแƒแƒ•แƒ˜แƒแƒœแƒแƒ‘แƒ˜แƒก แƒ›แƒฃแƒ“แƒ›แƒ˜แƒ•แƒแƒก.

K = 4.93. 10โ€“10 แƒ›แƒแƒš/แƒš

แƒ”แƒšแƒ”แƒฅแƒขแƒ แƒแƒšแƒ˜แƒขแƒฃแƒ แƒ˜ แƒ“แƒ˜แƒกแƒแƒชแƒ˜แƒแƒชแƒ˜แƒ˜แƒก แƒ—แƒ”แƒแƒ แƒ˜แƒแƒจแƒ˜ แƒกแƒฃแƒกแƒขแƒ˜ แƒ›แƒŸแƒแƒ•แƒ˜แƒก () แƒžแƒ แƒแƒขแƒแƒšแƒ˜แƒ–แƒ˜แƒก แƒฎแƒแƒ แƒ˜แƒกแƒฎแƒก แƒฃแƒฌแƒแƒ“แƒ”แƒ‘แƒ”แƒœ แƒ“แƒ˜แƒกแƒแƒชแƒ˜แƒแƒชแƒ˜แƒ˜แƒก แƒฎแƒแƒ แƒ˜แƒกแƒฎแƒ˜(แƒ—แƒฃ แƒ›แƒฎแƒแƒšแƒแƒ“ แƒ”แƒก แƒ—แƒ”แƒแƒ แƒ˜แƒ แƒ›แƒแƒชแƒ”แƒ›แƒฃแƒš แƒœแƒ˜แƒ•แƒ—แƒ˜แƒ”แƒ แƒ”แƒ‘แƒแƒก แƒ›แƒŸแƒแƒ•แƒแƒ“ แƒ›แƒ˜แƒ˜แƒฉแƒœแƒ”แƒ•แƒก).

แƒžแƒ แƒแƒขแƒแƒšแƒ˜แƒ–แƒฃแƒ  แƒ—แƒ”แƒแƒ แƒ˜แƒแƒจแƒ˜, แƒคแƒฃแƒซแƒ˜แƒก แƒ“แƒแƒกแƒแƒฎแƒแƒกแƒ˜แƒแƒ—แƒ”แƒ‘แƒšแƒแƒ“, แƒจแƒ”แƒ’แƒ˜แƒซแƒšแƒ˜แƒแƒ— แƒ’แƒแƒ›แƒแƒ˜แƒงแƒ”แƒœแƒแƒ— แƒ›แƒ˜แƒกแƒ˜ แƒ‘แƒแƒ–แƒ˜แƒกแƒฃแƒ แƒแƒ‘แƒ˜แƒก แƒ›แƒฃแƒ“แƒ›แƒ˜แƒ•แƒ˜, แƒแƒœ แƒจแƒ”แƒ’แƒ˜แƒซแƒšแƒ˜แƒแƒ— แƒ’แƒแƒฃแƒ›แƒ™แƒšแƒแƒ•แƒ“แƒ”แƒ— แƒ™แƒแƒœแƒ˜แƒฃแƒ’แƒ˜แƒ แƒ”แƒ‘แƒฃแƒšแƒ˜ แƒ›แƒŸแƒแƒ•แƒแƒก แƒ›แƒŸแƒแƒ•แƒ˜แƒแƒœแƒแƒ‘แƒ˜แƒก แƒ›แƒฃแƒ“แƒ›แƒ˜แƒ•แƒแƒก. แƒ”แƒšแƒ”แƒฅแƒขแƒ แƒแƒšแƒ˜แƒขแƒฃแƒ แƒ˜ แƒ“แƒ˜แƒกแƒแƒชแƒ˜แƒแƒชแƒ˜แƒ˜แƒก แƒ—แƒ”แƒแƒ แƒ˜แƒแƒจแƒ˜ แƒ‘แƒแƒ–แƒ”แƒ‘แƒแƒ“ แƒ˜แƒ—แƒ•แƒšแƒ”แƒ‘แƒแƒ“แƒ แƒ›แƒฎแƒแƒšแƒแƒ“ แƒœแƒ˜แƒ•แƒ—แƒ˜แƒ”แƒ แƒ”แƒ‘แƒ”แƒ‘แƒ˜, แƒ แƒแƒ›แƒšแƒ”แƒ‘แƒ˜แƒช แƒฎแƒกแƒœแƒแƒ แƒจแƒ˜ แƒ˜แƒจแƒšแƒ”แƒ‘แƒ แƒ™แƒแƒขแƒ˜แƒแƒœแƒจแƒ˜ แƒ“แƒ แƒฐแƒ˜แƒ“แƒ แƒแƒฅแƒกแƒ˜แƒ“แƒ˜แƒก แƒ˜แƒแƒœแƒ”แƒ‘แƒจแƒ˜, แƒแƒ›แƒ˜แƒขแƒแƒ›, แƒ›แƒแƒ’แƒแƒšแƒ˜แƒ—แƒแƒ“, แƒ•แƒแƒ แƒแƒฃแƒ“แƒแƒ‘แƒ“แƒœแƒ”แƒœ, แƒ แƒแƒ› แƒแƒ›แƒ˜แƒแƒ™แƒ˜แƒก แƒฎแƒกแƒœแƒแƒ แƒ˜ แƒจแƒ”แƒ˜แƒชแƒแƒ•แƒก "แƒแƒ›แƒแƒœแƒ˜แƒฃแƒ›แƒ˜แƒก แƒฐแƒ˜แƒ“แƒ แƒแƒฅแƒกแƒ˜แƒ“แƒก", แƒฎแƒแƒšแƒ แƒ›แƒแƒ’แƒ•แƒ˜แƒแƒœแƒ”แƒ‘แƒ˜แƒ— - แƒแƒ›แƒ˜แƒแƒ™แƒ˜แƒก แƒฐแƒ˜แƒ“แƒ แƒแƒขแƒก.

NH 3 + H 2 O NH 4 + + OH -

K O (NH 3) \u003d K C . =
1.74. 10โ€“5 แƒ›แƒแƒš/แƒš

NH3. H 2 O NH 4 + + OH -

แƒฌแƒแƒœแƒแƒกแƒฌแƒแƒ แƒแƒ‘แƒ˜แƒก แƒ›แƒฃแƒ“แƒ›แƒ˜แƒ•แƒ˜ K Cแƒ“แƒ แƒแƒ› แƒจแƒ”แƒ›แƒ—แƒฎแƒ•แƒ”แƒ•แƒแƒจแƒ˜ แƒ”แƒฌแƒแƒ“แƒ”แƒ‘แƒ แƒ“แƒ˜แƒกแƒแƒชแƒ˜แƒแƒชแƒ˜แƒ˜แƒก แƒ›แƒฃแƒ“แƒ›แƒ˜แƒ•แƒ˜, แƒแƒฆแƒ˜แƒœแƒ˜แƒจแƒœแƒ”แƒ‘แƒ TOแƒ“แƒ แƒฃแƒ“แƒ แƒ˜แƒก แƒกแƒแƒคแƒฃแƒซแƒ•แƒšแƒ˜แƒแƒœแƒแƒ‘แƒ˜แƒก แƒ›แƒฃแƒ“แƒ›แƒ˜แƒ•แƒแƒก.

K = 1.74. 10โ€“5 แƒ›แƒแƒš/แƒš

แƒแƒ› แƒ—แƒ”แƒแƒ แƒ˜แƒแƒจแƒ˜ แƒแƒ  แƒแƒ แƒกแƒ”แƒ‘แƒแƒ‘แƒก แƒ™แƒแƒœแƒ˜แƒฃแƒ’แƒ˜แƒ แƒ”แƒ‘แƒฃแƒšแƒ˜ แƒ›แƒŸแƒแƒ•แƒ˜แƒก แƒ™แƒแƒœแƒชแƒ”แƒคแƒชแƒ˜แƒ. แƒแƒ›แƒแƒœแƒ˜แƒฃแƒ›แƒ˜แƒก แƒ˜แƒแƒœแƒ˜ แƒแƒ  แƒ˜แƒ—แƒ•แƒšแƒ”แƒ‘แƒ แƒ›แƒŸแƒแƒ•แƒแƒ“. แƒแƒ›แƒแƒœแƒ˜แƒฃแƒ›แƒ˜แƒก แƒ›แƒแƒ แƒ˜แƒšแƒ”แƒ‘แƒ˜แƒก แƒฎแƒกแƒœแƒแƒ แƒ”แƒ‘แƒจแƒ˜ แƒ›แƒŸแƒแƒ•แƒ” แƒ’แƒแƒ แƒ”แƒ›แƒ แƒแƒ˜แƒฎแƒกแƒœแƒ”แƒ‘แƒ แƒฐแƒ˜แƒ“แƒ แƒแƒšแƒ˜แƒ–แƒ˜แƒ—.

แƒ”แƒšแƒ”แƒฅแƒขแƒ แƒแƒšแƒ˜แƒขแƒฃแƒ แƒ˜ แƒ“แƒ˜แƒกแƒแƒชแƒ˜แƒแƒชแƒ˜แƒ˜แƒก แƒ—แƒ”แƒแƒ แƒ˜แƒแƒจแƒ˜ แƒ™แƒ˜แƒ“แƒ”แƒ• แƒฃแƒคแƒ แƒ แƒ แƒ—แƒฃแƒšแƒ˜แƒ แƒกแƒฎแƒ•แƒ แƒœแƒ˜แƒ•แƒ—แƒ˜แƒ”แƒ แƒ”แƒ‘แƒ”แƒ‘แƒ˜แƒก แƒซแƒ˜แƒ แƒ˜แƒ—แƒแƒ“แƒ˜ แƒ—แƒ•แƒ˜แƒกแƒ”แƒ‘แƒ”แƒ‘แƒ˜แƒก แƒแƒฆแƒฌแƒ”แƒ แƒ, แƒ แƒแƒ›แƒšแƒ”แƒ‘แƒ˜แƒช แƒแƒ  แƒจแƒ”แƒ˜แƒชแƒแƒ•แƒก แƒฐแƒ˜แƒ“แƒ แƒแƒฅแƒกแƒ˜แƒšแƒ”แƒ‘แƒก, แƒ›แƒแƒ’แƒแƒšแƒ˜แƒ—แƒแƒ“, แƒแƒ›แƒ˜แƒœแƒ”แƒ‘แƒก (แƒ›แƒ”แƒ—แƒ˜แƒšแƒแƒ›แƒ˜แƒœแƒ˜ CH 3 NH 2, แƒแƒœแƒ˜แƒšแƒ˜แƒœแƒ˜ C 6 H 5 NH 2 แƒ“แƒ แƒ.แƒจ.).

แƒฎแƒกแƒœแƒแƒ แƒ”แƒ‘แƒ˜แƒก แƒ›แƒŸแƒแƒ•แƒ” แƒ“แƒ แƒซแƒ˜แƒ แƒ˜แƒ—แƒแƒ“แƒ˜ แƒ—แƒ•แƒ˜แƒกแƒ”แƒ‘แƒ”แƒ‘แƒ˜แƒก แƒ“แƒแƒกแƒแƒฎแƒแƒกแƒ˜แƒแƒ—แƒ”แƒ‘แƒšแƒแƒ“ แƒ’แƒแƒ›แƒแƒ˜แƒงแƒ”แƒœแƒ”แƒ‘แƒ แƒกแƒฎแƒ•แƒ แƒคแƒ˜แƒ–แƒ˜แƒ™แƒฃแƒ แƒ˜ แƒ แƒแƒแƒ“แƒ”แƒœแƒแƒ‘แƒ - pH แƒ›แƒœแƒ˜แƒจแƒ•แƒœแƒ”แƒšแƒแƒ‘แƒ(แƒแƒฆแƒœแƒ˜แƒจแƒœแƒฃแƒšแƒ˜แƒ pH-แƒ˜แƒ—, แƒฌแƒแƒ˜แƒ™แƒ˜แƒ—แƒฎแƒ”แƒ— "ph"). แƒ”แƒšแƒ”แƒฅแƒขแƒ แƒแƒšแƒ˜แƒขแƒฃแƒ แƒ˜ แƒ“แƒ˜แƒกแƒแƒชแƒ˜แƒแƒชแƒ˜แƒ˜แƒก แƒ—แƒ”แƒแƒ แƒ˜แƒ˜แƒก แƒคแƒแƒ แƒ’แƒšแƒ”แƒ‘แƒจแƒ˜ แƒฌแƒงแƒแƒšแƒ‘แƒแƒ“แƒ˜แƒก แƒ˜แƒœแƒ“แƒ”แƒฅแƒกแƒ˜ แƒ’แƒแƒœแƒ˜แƒกแƒแƒ–แƒฆแƒ•แƒ แƒ แƒจแƒ”แƒ›แƒ“แƒ”แƒ’แƒœแƒแƒ˜แƒ แƒแƒ“:

pH = -แƒšแƒ’

แƒฃแƒคแƒ แƒ แƒ–แƒฃแƒกแƒขแƒ˜ แƒ’แƒแƒœแƒ›แƒแƒ แƒขแƒ”แƒ‘แƒ, แƒฎแƒกแƒœแƒแƒ แƒจแƒ˜ แƒฌแƒงแƒแƒšแƒ‘แƒแƒ“แƒ˜แƒก แƒ˜แƒแƒœแƒ”แƒ‘แƒ˜แƒก แƒแƒ แƒแƒ แƒกแƒ”แƒ‘แƒแƒ‘แƒ˜แƒก แƒ“แƒ แƒกแƒแƒ–แƒแƒ›แƒ˜ แƒ”แƒ แƒ—แƒ”แƒฃแƒšแƒ”แƒ‘แƒ˜แƒก แƒšแƒแƒ’แƒแƒ แƒ˜แƒ—แƒ›แƒ”แƒ‘แƒ˜แƒก แƒแƒฆแƒ”แƒ‘แƒ˜แƒก แƒจแƒ”แƒฃแƒซแƒšแƒ”แƒ‘แƒšแƒแƒ‘แƒ˜แƒก แƒ’แƒแƒ—แƒ•แƒแƒšแƒ˜แƒกแƒฌแƒ˜แƒœแƒ”แƒ‘แƒ˜แƒ—:

pH = โ€“lg ()

แƒฃแƒคแƒ แƒ แƒกแƒฌแƒแƒ แƒ˜ แƒ˜แƒฅแƒœแƒ”แƒ‘แƒ แƒแƒ› แƒ›แƒœแƒ˜แƒจแƒ•แƒœแƒ”แƒšแƒแƒ‘แƒแƒก แƒ“แƒแƒ•แƒแƒ แƒฅแƒ•แƒแƒ— "แƒแƒฅแƒกแƒแƒœแƒ˜แƒฃแƒ›แƒ˜" แƒ“แƒ แƒแƒ แƒ pH แƒ›แƒœแƒ˜แƒจแƒ•แƒœแƒ”แƒšแƒแƒ‘แƒ, แƒ›แƒแƒ’แƒ แƒแƒ› แƒ”แƒก แƒกแƒแƒฎแƒ”แƒšแƒ˜ แƒแƒ  แƒ’แƒแƒ›แƒแƒ˜แƒงแƒ”แƒœแƒ”แƒ‘แƒ.

แƒ˜แƒ’แƒ˜ แƒ’แƒแƒœแƒ˜แƒกแƒแƒ–แƒฆแƒ•แƒ แƒ”แƒ‘แƒ แƒฌแƒงแƒแƒšแƒ‘แƒแƒ“แƒ˜แƒก แƒ›แƒกแƒ’แƒแƒ•แƒกแƒแƒ“ แƒฐแƒ˜แƒ“แƒ แƒแƒฅแƒกแƒ˜แƒ“แƒ˜แƒก แƒ˜แƒœแƒ“แƒ”แƒฅแƒกแƒ˜(แƒ›แƒ˜แƒœแƒ˜แƒจแƒœแƒ”แƒ‘แƒ pOH, แƒฌแƒแƒ˜แƒ™แƒ˜แƒ—แƒฎแƒ”แƒ— "แƒžแƒ” แƒแƒแƒจ").

pOH = -lg ()

แƒฌแƒงแƒแƒšแƒ‘แƒแƒ“แƒ˜แƒกแƒ แƒ“แƒ แƒฐแƒ˜แƒ“แƒ แƒแƒฅแƒกแƒ˜แƒ“แƒ˜แƒก แƒ˜แƒœแƒ“แƒ”แƒฅแƒกแƒ”แƒ‘แƒ˜แƒกแƒ—แƒ•แƒ˜แƒก แƒ’แƒแƒ›แƒแƒกแƒแƒฎแƒฃแƒšแƒ”แƒ‘แƒ”แƒ‘แƒจแƒ˜ แƒ แƒแƒแƒ“แƒ”แƒœแƒแƒ‘แƒ˜แƒก แƒ แƒ˜แƒชแƒฎแƒแƒ‘แƒ แƒ˜แƒ•แƒ˜ แƒ›แƒœแƒ˜แƒจแƒ•แƒœแƒ”แƒšแƒแƒ‘แƒ˜แƒก แƒแƒฆแƒ›แƒœแƒ˜แƒจแƒ•แƒœแƒ”แƒšแƒ˜ แƒฎแƒ•แƒ”แƒฃแƒšแƒ˜ แƒคแƒ แƒฉแƒฎแƒ˜แƒšแƒ”แƒ‘แƒ˜ แƒซแƒแƒšแƒ˜แƒแƒœ แƒฎแƒจแƒ˜แƒ แƒแƒ“ แƒแƒ  แƒแƒ แƒ˜แƒก แƒ’แƒแƒœแƒ—แƒแƒ•แƒกแƒ”แƒ‘แƒฃแƒšแƒ˜, แƒ“แƒแƒ’แƒ•แƒแƒ•แƒ˜แƒฌแƒงแƒ“แƒ”แƒก, แƒ แƒแƒ› แƒจแƒ”แƒฃแƒซแƒšแƒ”แƒ‘แƒ”แƒšแƒ˜แƒ แƒคแƒ˜แƒ–แƒ˜แƒ™แƒฃแƒ แƒ˜ แƒกแƒ˜แƒ“แƒ˜แƒ“แƒ”แƒ”แƒ‘แƒ˜แƒก แƒšแƒแƒ’แƒแƒ แƒ˜แƒ—แƒ›แƒ˜แƒก แƒแƒฆแƒ”แƒ‘แƒ.

แƒ•แƒ˜แƒœแƒแƒ˜แƒ“แƒแƒœ แƒฌแƒงแƒšแƒ˜แƒก แƒ˜แƒแƒœแƒฃแƒ แƒ˜ แƒžแƒ แƒแƒ“แƒฃแƒฅแƒขแƒ˜ แƒ›แƒฃแƒ“แƒ›แƒ˜แƒ•แƒ˜ แƒ›แƒœแƒ˜แƒจแƒ•แƒœแƒ”แƒšแƒแƒ‘แƒแƒ แƒแƒ แƒ แƒ›แƒฎแƒแƒšแƒแƒ“ แƒกแƒฃแƒคแƒ—แƒ แƒฌแƒงแƒแƒšแƒจแƒ˜, แƒแƒ แƒแƒ›แƒ”แƒ“ แƒ›แƒŸแƒแƒ•แƒ”แƒ‘แƒ˜แƒกแƒ แƒ“แƒ แƒคแƒฃแƒซแƒ”แƒ”แƒ‘แƒ˜แƒก แƒ’แƒแƒœแƒ–แƒแƒ•แƒ”แƒ‘แƒฃแƒš แƒฎแƒกแƒœแƒแƒ แƒ”แƒ‘แƒจแƒ˜, แƒฌแƒงแƒแƒšแƒ‘แƒแƒ“แƒ˜แƒก แƒ“แƒ แƒฐแƒ˜แƒ“แƒ แƒแƒฅแƒกแƒ˜แƒ“แƒ˜แƒก แƒ˜แƒœแƒ“แƒ”แƒฅแƒกแƒ”แƒ‘แƒ˜ แƒฃแƒ แƒ—แƒ˜แƒ”แƒ แƒ—แƒ“แƒแƒ™แƒแƒ•แƒจแƒ˜แƒ แƒ”แƒ‘แƒฃแƒšแƒ˜แƒ:

K (H 2 O) \u003d \u003d 10 -14 แƒ›แƒแƒšแƒ˜ 2 / แƒš 2
lg () = lg () + lg () = -14
pH + pOH = 14

แƒกแƒฃแƒคแƒ—แƒ แƒฌแƒงแƒแƒšแƒจแƒ˜ = = 10-7 แƒ›แƒแƒš/แƒš, แƒจแƒ”แƒกแƒแƒ‘แƒแƒ›แƒ˜แƒกแƒแƒ“, pH = pOH = 7.

แƒ›แƒŸแƒแƒ•แƒ แƒฎแƒกแƒœแƒแƒ แƒจแƒ˜ (แƒ›แƒŸแƒแƒ•แƒ” แƒฎแƒกแƒœแƒแƒ แƒจแƒ˜) แƒแƒ แƒ˜แƒก แƒแƒฅแƒกแƒแƒœแƒ˜แƒฃแƒ›แƒ˜แƒก แƒ˜แƒแƒœแƒ”แƒ‘แƒ˜แƒก แƒญแƒแƒ แƒ‘แƒ˜ แƒ แƒแƒแƒ“แƒ”แƒœแƒแƒ‘แƒ, แƒ›แƒแƒ—แƒ˜ แƒ™แƒแƒœแƒชแƒ”แƒœแƒขแƒ แƒแƒชแƒ˜แƒ 10-7 แƒ›แƒแƒš/แƒš-แƒ–แƒ” แƒ›แƒ”แƒขแƒ˜แƒ แƒ“แƒ, แƒจแƒ”แƒกแƒแƒ‘แƒแƒ›แƒ˜แƒกแƒแƒ“, pH.< 7.

แƒคแƒฃแƒซแƒ” แƒฎแƒกแƒœแƒแƒ แƒจแƒ˜ (แƒขแƒฃแƒขแƒ” แƒฎแƒกแƒœแƒแƒ แƒจแƒ˜), แƒžแƒ˜แƒ แƒ˜แƒฅแƒ˜แƒ—, แƒญแƒแƒ แƒ‘แƒแƒ‘แƒก แƒฐแƒ˜แƒ“แƒ แƒแƒฅแƒกแƒ˜แƒ“แƒ˜แƒก แƒ˜แƒแƒœแƒ”แƒ‘แƒ˜ แƒ“แƒ, แƒจแƒ”แƒกแƒแƒ‘แƒแƒ›แƒ˜แƒกแƒแƒ“, แƒแƒฅแƒกแƒแƒœแƒ˜แƒฃแƒ›แƒ˜แƒก แƒ˜แƒแƒœแƒ”แƒ‘แƒ˜แƒก แƒ™แƒแƒœแƒชแƒ”แƒœแƒขแƒ แƒแƒชแƒ˜แƒ 10-7 แƒ›แƒแƒš/แƒš-แƒ–แƒ” แƒœแƒแƒ™แƒšแƒ”แƒ‘แƒ˜แƒ; แƒแƒ› แƒจแƒ”แƒ›แƒ—แƒฎแƒ•แƒ”แƒ•แƒแƒจแƒ˜ pH > 7.

20.4. แƒฐแƒ˜แƒ“แƒ แƒแƒšแƒ˜แƒ–แƒ˜แƒก แƒ›แƒฃแƒ“แƒ›แƒ˜แƒ•แƒ˜

แƒ”แƒšแƒ”แƒฅแƒขแƒ แƒแƒšแƒ˜แƒขแƒฃแƒ แƒ˜ แƒ“แƒ˜แƒกแƒแƒชแƒ˜แƒแƒชแƒ˜แƒ˜แƒก แƒ—แƒ”แƒแƒ แƒ˜แƒ˜แƒก แƒคแƒแƒ แƒ’แƒšแƒ”แƒ‘แƒจแƒ˜ แƒชแƒแƒšแƒ™แƒ” แƒžแƒ แƒแƒชแƒ”แƒกแƒแƒ“ แƒ’แƒแƒœแƒ˜แƒฎแƒ˜แƒšแƒ”แƒ‘แƒ แƒจแƒ”แƒฅแƒชแƒ”แƒ•แƒแƒ“แƒ˜ แƒฐแƒ˜แƒ“แƒ แƒแƒšแƒ˜แƒ–แƒ˜ (แƒ›แƒแƒ แƒ˜แƒšแƒ”แƒ‘แƒ˜แƒก แƒฐแƒ˜แƒ“แƒ แƒแƒšแƒ˜แƒ–แƒ˜), แƒฎแƒแƒšแƒ แƒ’แƒแƒ›แƒแƒงแƒแƒคแƒ”แƒœ แƒฐแƒ˜แƒ“แƒ แƒแƒšแƒ˜แƒ–แƒ˜แƒก แƒจแƒ”แƒ›แƒ—แƒฎแƒ•แƒ”แƒ•แƒ”แƒ‘แƒก.

  • แƒซแƒšแƒ˜แƒ”แƒ แƒ˜ แƒคแƒฃแƒซแƒ˜แƒกแƒ แƒ“แƒ แƒกแƒฃแƒกแƒขแƒ˜ แƒ›แƒŸแƒแƒ•แƒ˜แƒก แƒ›แƒแƒ แƒ˜แƒšแƒ”แƒ‘แƒ˜
  • แƒกแƒฃแƒกแƒขแƒ˜ แƒคแƒฃแƒซแƒ˜แƒกแƒ แƒ“แƒ แƒซแƒšแƒ˜แƒ”แƒ แƒ˜ แƒ›แƒŸแƒแƒ•แƒ˜แƒก แƒ›แƒแƒ แƒ˜แƒšแƒ”แƒ‘แƒ˜ แƒ“แƒ
  • แƒกแƒฃแƒกแƒขแƒ˜ แƒคแƒฃแƒซแƒ˜แƒกแƒ แƒ“แƒ แƒกแƒฃแƒกแƒขแƒ˜ แƒ›แƒŸแƒแƒ•แƒ˜แƒก แƒ›แƒแƒ แƒ˜แƒšแƒ”แƒ‘แƒ˜.

แƒ’แƒแƒœแƒ•แƒ˜แƒฎแƒ˜แƒšแƒแƒ— แƒ”แƒก แƒจแƒ”แƒ›แƒ—แƒฎแƒ•แƒ”แƒ•แƒ”แƒ‘แƒ˜ แƒžแƒแƒ แƒแƒšแƒ”แƒšแƒฃแƒ แƒแƒ“ แƒžแƒ แƒแƒขแƒแƒšแƒ˜แƒ–แƒฃแƒ แƒ˜ แƒ—แƒ”แƒแƒ แƒ˜แƒ˜แƒก แƒคแƒแƒ แƒ’แƒšแƒ”แƒ‘แƒจแƒ˜ แƒ“แƒ แƒ”แƒšแƒ”แƒฅแƒขแƒ แƒแƒšแƒ˜แƒขแƒฃแƒ แƒ˜ แƒ“แƒ˜แƒกแƒแƒชแƒ˜แƒแƒชแƒ˜แƒ˜แƒก แƒ—แƒ”แƒแƒ แƒ˜แƒ˜แƒก แƒคแƒแƒ แƒ’แƒšแƒ”แƒ‘แƒจแƒ˜.

แƒซแƒšแƒ˜แƒ”แƒ แƒ˜ แƒคแƒฃแƒซแƒ˜แƒก แƒ›แƒแƒ แƒ˜แƒšแƒ˜ แƒ“แƒ แƒกแƒฃแƒกแƒขแƒ˜ แƒ›แƒŸแƒแƒ•แƒ

แƒ แƒแƒ’แƒแƒ แƒช แƒžแƒ˜แƒ แƒ•แƒ”แƒšแƒ˜ แƒ›แƒแƒ’แƒแƒšแƒ˜แƒ—แƒ˜, แƒ’แƒแƒœแƒ•แƒ˜แƒฎแƒ˜แƒšแƒแƒ— KNO 2-แƒ˜แƒก แƒฐแƒ˜แƒ“แƒ แƒแƒšแƒ˜แƒ–แƒ˜, แƒซแƒšแƒ˜แƒ”แƒ แƒ˜ แƒคแƒฃแƒซแƒ˜แƒก แƒ›แƒแƒ แƒ˜แƒšแƒ˜ แƒ“แƒ แƒกแƒฃแƒกแƒขแƒ˜ แƒ›แƒแƒœแƒแƒ‘แƒแƒ–แƒฃแƒ แƒ˜ แƒ›แƒŸแƒแƒ•แƒ.

K +, NO 2 - แƒ“แƒ H 2 O.

NO 2 - แƒแƒ แƒ˜แƒก แƒกแƒฃแƒกแƒขแƒ˜ แƒคแƒฃแƒซแƒ”, แƒฎแƒแƒšแƒ H 2 O แƒแƒ แƒ˜แƒก แƒแƒ›แƒคแƒแƒšแƒ˜แƒขแƒ˜, แƒจแƒ”แƒกแƒแƒ‘แƒแƒ›แƒ˜แƒกแƒแƒ“, แƒจแƒ”แƒกแƒแƒซแƒšแƒ”แƒ‘แƒ”แƒšแƒ˜แƒ แƒจแƒ”แƒฅแƒชแƒ”แƒ•แƒแƒ“แƒ˜ แƒ แƒ”แƒแƒฅแƒชแƒ˜แƒ

NO 2 - + H 2 O HNO 2 + OH -,

แƒ แƒแƒ›แƒšแƒ˜แƒก แƒฌแƒแƒœแƒแƒกแƒฌแƒแƒ แƒแƒ‘แƒ แƒแƒฆแƒฌแƒ”แƒ แƒ˜แƒšแƒ˜แƒ แƒœแƒ˜แƒขแƒ แƒ˜แƒขแƒ˜แƒก แƒ˜แƒแƒœแƒ˜แƒก แƒ‘แƒแƒ–แƒ˜แƒกแƒฃแƒ แƒแƒ‘แƒ˜แƒก แƒ›แƒฃแƒ“แƒ›แƒ˜แƒ•แƒ˜แƒ— แƒ“แƒ แƒจแƒ”แƒ˜แƒซแƒšแƒ”แƒ‘แƒ แƒ’แƒแƒ›แƒแƒ˜แƒฎแƒแƒขแƒแƒก แƒแƒ–แƒแƒขแƒ˜แƒก แƒ›แƒŸแƒแƒ•แƒแƒก แƒ›แƒŸแƒแƒ•แƒ˜แƒแƒœแƒแƒ‘แƒ˜แƒก แƒ›แƒฃแƒ“แƒ›แƒ˜แƒ•แƒแƒ‘แƒ˜แƒ—:

K o (NO 2 -) \u003d

แƒ แƒแƒ“แƒ”แƒกแƒแƒช แƒ”แƒก แƒœแƒ˜แƒ•แƒ—แƒ˜แƒ”แƒ แƒ”แƒ‘แƒ แƒ˜แƒฎแƒกแƒœแƒ”แƒ‘แƒ, แƒ˜แƒก แƒจแƒ”แƒฃแƒฅแƒชแƒ”แƒ•แƒแƒ“ แƒ˜แƒจแƒšแƒ”แƒ‘แƒ K + แƒ“แƒ NO 2 - แƒ˜แƒแƒœแƒ”แƒ‘แƒแƒ“:

KNO 2 = K + + NO 2 -

H 2 O H + + OH -

แƒฎแƒกแƒœแƒแƒ แƒจแƒ˜ H + แƒ“แƒ NO 2 - แƒ˜แƒแƒœแƒ”แƒ‘แƒ˜แƒก แƒ”แƒ แƒ—แƒ“แƒ แƒแƒฃแƒšแƒ˜ แƒแƒ แƒกแƒ”แƒ‘แƒแƒ‘แƒ˜แƒ—, แƒฎแƒ“แƒ”แƒ‘แƒ แƒจแƒ”แƒฅแƒชแƒ”แƒ•แƒแƒ“แƒ˜ แƒ แƒ”แƒแƒฅแƒชแƒ˜แƒ.

H + + NO 2 - HNO 2

NO 2 - + H 2 O HNO 2 + OH -

แƒฐแƒ˜แƒ“แƒ แƒแƒšแƒ˜แƒ–แƒ˜แƒก แƒ แƒ”แƒแƒฅแƒชแƒ˜แƒ˜แƒก แƒฌแƒแƒœแƒแƒกแƒฌแƒแƒ แƒแƒ‘แƒ แƒแƒฆแƒฌแƒ”แƒ แƒ˜แƒšแƒ˜แƒ แƒฐแƒ˜แƒ“แƒ แƒแƒšแƒ˜แƒ–แƒ˜แƒก แƒ›แƒฃแƒ“แƒ›แƒ˜แƒ•แƒ˜แƒ— ( แƒ™ แƒ—) แƒ“แƒ แƒจแƒ”แƒ˜แƒซแƒšแƒ”แƒ‘แƒ แƒ’แƒแƒ›แƒแƒ˜แƒฎแƒแƒขแƒแƒก แƒ“แƒ˜แƒกแƒแƒชแƒ˜แƒแƒชแƒ˜แƒ˜แƒก แƒ›แƒฃแƒ“แƒ›แƒ˜แƒ•แƒ˜แƒก ( TOแƒ”) แƒแƒ–แƒแƒขแƒ˜แƒก แƒ›แƒŸแƒแƒ•แƒ:

K h = แƒ™แƒ™ . =

แƒ แƒแƒ’แƒแƒ แƒช แƒฎแƒ”แƒ“แƒแƒ•แƒ—, แƒแƒ› แƒจแƒ”แƒ›แƒ—แƒฎแƒ•แƒ”แƒ•แƒแƒจแƒ˜ แƒฐแƒ˜แƒ“แƒ แƒแƒšแƒ˜แƒ–แƒ˜แƒก แƒ›แƒฃแƒ“แƒ›แƒ˜แƒ•แƒ˜ แƒฃแƒ“แƒ แƒ˜แƒก แƒกแƒแƒ‘แƒแƒ–แƒ˜แƒกแƒ แƒœแƒแƒฌแƒ˜แƒšแƒแƒ™แƒ”แƒ‘แƒ˜แƒก แƒ‘แƒแƒ–แƒ˜แƒกแƒฃแƒ แƒแƒ‘แƒ˜แƒก แƒ›แƒฃแƒ“แƒ›แƒ˜แƒ•แƒแƒก.

แƒ˜แƒ›แƒ˜แƒกแƒ“แƒ แƒ›แƒ˜แƒฃแƒฎแƒ”แƒ“แƒแƒ•แƒแƒ“, แƒ แƒแƒ› แƒจแƒ”แƒฅแƒชแƒ”แƒ•แƒแƒ“แƒ˜ แƒฐแƒ˜แƒ“แƒ แƒแƒšแƒ˜แƒ–แƒ˜ แƒฎแƒ“แƒ”แƒ‘แƒ แƒ›แƒฎแƒแƒšแƒแƒ“ แƒฎแƒกแƒœแƒแƒ แƒจแƒ˜, แƒ˜แƒก แƒ›แƒ—แƒšแƒ˜แƒแƒœแƒแƒ“ "แƒฉแƒแƒฎแƒจแƒแƒ‘แƒ˜แƒšแƒ˜แƒ" แƒฌแƒงแƒšแƒ˜แƒก แƒแƒ›แƒแƒฆแƒ”แƒ‘แƒ˜แƒกแƒแƒก แƒ“แƒ, แƒจแƒ”แƒกแƒแƒ‘แƒแƒ›แƒ˜แƒกแƒแƒ“, แƒแƒ› แƒ แƒ”แƒแƒฅแƒชแƒ˜แƒ˜แƒก แƒžแƒ แƒแƒ“แƒฃแƒฅแƒขแƒ”แƒ‘แƒ˜แƒก แƒ›แƒ˜แƒฆแƒ”แƒ‘แƒ แƒจแƒ”แƒฃแƒซแƒšแƒ”แƒ‘แƒ”แƒšแƒ˜แƒ, แƒ”แƒšแƒ”แƒฅแƒขแƒ แƒแƒšแƒ˜แƒขแƒฃแƒ แƒ˜ แƒ“แƒ˜แƒกแƒแƒชแƒ˜แƒแƒชแƒ˜แƒ˜แƒก แƒ—แƒ”แƒแƒ แƒ˜แƒ˜แƒก แƒคแƒแƒ แƒ’แƒšแƒ”แƒ‘แƒจแƒ˜, แƒ›แƒแƒšแƒ”แƒ™แƒฃแƒšแƒฃแƒ แƒ˜ แƒฐแƒ˜แƒ“แƒ แƒแƒšแƒ˜แƒ–แƒ˜แƒก แƒ’แƒแƒœแƒขแƒแƒšแƒ”แƒ‘แƒแƒ. แƒแƒกแƒ”แƒ•แƒ” แƒ“แƒแƒฌแƒ”แƒ แƒ˜แƒšแƒ˜:

KNO 2 + H 2 O KOH + HNO 2

แƒ แƒแƒ’แƒแƒ แƒช แƒกแƒฎแƒ•แƒ แƒ›แƒแƒ’แƒแƒšแƒ˜แƒ—แƒ˜, แƒ’แƒแƒœแƒ•แƒ˜แƒฎแƒ˜แƒšแƒแƒ— Na 2 CO 3 แƒฐแƒ˜แƒ“แƒ แƒแƒšแƒ˜แƒ–แƒ˜, แƒซแƒšแƒ˜แƒ”แƒ แƒ˜ แƒคแƒฃแƒซแƒ˜แƒก แƒ›แƒแƒ แƒ˜แƒšแƒ˜ แƒ“แƒ แƒกแƒฃแƒกแƒขแƒ˜ แƒแƒ แƒคแƒฃแƒซแƒ˜แƒแƒœแƒ˜ แƒ›แƒŸแƒแƒ•แƒ. แƒ›แƒกแƒฏแƒ”แƒšแƒแƒ‘แƒ˜แƒก แƒฎแƒแƒ–แƒ˜ แƒแƒฅ แƒ–แƒฃแƒกแƒขแƒแƒ“ แƒ˜แƒ’แƒ˜แƒ•แƒ”แƒ. แƒแƒ แƒ˜แƒ•แƒ” แƒ—แƒ”แƒแƒ แƒ˜แƒ˜แƒก แƒคแƒแƒ แƒ’แƒšแƒ”แƒ‘แƒจแƒ˜ แƒ›แƒ˜แƒ˜แƒฆแƒ”แƒ‘แƒ แƒ˜แƒแƒœแƒฃแƒ แƒ˜ แƒ’แƒแƒœแƒขแƒแƒšแƒ”แƒ‘แƒ:

CO 3 2- + H 2 O HCO 3 - + OH -

แƒžแƒ แƒแƒขแƒแƒšแƒ˜แƒ–แƒฃแƒ แƒ˜ แƒ—แƒ”แƒแƒ แƒ˜แƒ˜แƒก แƒคแƒแƒ แƒ’แƒšแƒ”แƒ‘แƒจแƒ˜ แƒ›แƒแƒก แƒ”แƒฌแƒแƒ“แƒ”แƒ‘แƒ แƒ™แƒแƒ แƒ‘แƒแƒœแƒแƒขแƒฃแƒšแƒ˜ แƒ˜แƒแƒœแƒฃแƒ แƒ˜ แƒžแƒ แƒแƒขแƒแƒšแƒ˜แƒ–แƒ˜แƒก แƒ’แƒแƒœแƒขแƒแƒšแƒ”แƒ‘แƒ, แƒฎแƒแƒšแƒ แƒ”แƒšแƒ”แƒฅแƒขแƒ แƒแƒšแƒ˜แƒขแƒฃแƒ แƒ˜ แƒ“แƒ˜แƒกแƒแƒชแƒ˜แƒแƒชแƒ˜แƒ˜แƒก แƒ—แƒ”แƒแƒ แƒ˜แƒ˜แƒก แƒคแƒแƒ แƒ’แƒšแƒ”แƒ‘แƒจแƒ˜ - แƒœแƒแƒขแƒ แƒ˜แƒฃแƒ›แƒ˜แƒก แƒ™แƒแƒ แƒ‘แƒแƒœแƒแƒขแƒ˜แƒก แƒฐแƒ˜แƒ“แƒ แƒแƒšแƒ˜แƒ–แƒ˜แƒก แƒ˜แƒแƒœแƒฃแƒ แƒ˜ แƒ’แƒแƒœแƒขแƒแƒšแƒ”แƒ‘แƒ.

Na 2 CO 3 + H 2 O NaHCO 3 + NaOH

แƒ™แƒแƒ แƒ‘แƒแƒœแƒแƒขแƒฃแƒšแƒ˜ แƒ˜แƒแƒœแƒ˜แƒก แƒ‘แƒแƒ–แƒ˜แƒกแƒฃแƒ แƒแƒ‘แƒ˜แƒก แƒ›แƒฃแƒ“แƒ›แƒ˜แƒ•แƒแƒก TED-แƒ˜แƒก แƒฉแƒแƒ แƒฉแƒแƒจแƒ˜ แƒ”แƒฌแƒแƒ“แƒ”แƒ‘แƒ แƒฐแƒ˜แƒ“แƒ แƒแƒšแƒ˜แƒ–แƒ˜แƒก แƒ›แƒฃแƒ“แƒ›แƒ˜แƒ•แƒ˜ แƒ“แƒ แƒ’แƒแƒ›แƒแƒ˜แƒฎแƒแƒขแƒ”แƒ‘แƒ โ€žแƒ›แƒ”แƒแƒ แƒ” แƒ”แƒขแƒแƒžแƒ–แƒ” แƒœแƒแƒฎแƒจแƒ˜แƒ แƒ›แƒŸแƒแƒ•แƒแƒก แƒ“แƒ˜แƒกแƒแƒชแƒ˜แƒแƒชแƒ˜แƒ˜แƒก แƒ›แƒฃแƒ“แƒ›แƒ˜แƒ•แƒ˜แƒ—โ€œ, แƒแƒœแƒฃ แƒฐแƒ˜แƒ“แƒ แƒแƒ™แƒแƒ แƒ‘แƒแƒœแƒแƒขแƒฃแƒšแƒ˜ แƒ˜แƒแƒœแƒ˜แƒก แƒ›แƒŸแƒแƒ•แƒ˜แƒแƒœแƒแƒ‘แƒ˜แƒก แƒ›แƒฃแƒ“แƒ›แƒ˜แƒ•แƒ˜แƒ—.

แƒฃแƒœแƒ“แƒ แƒแƒฆแƒ˜แƒœแƒ˜แƒจแƒœแƒแƒก, แƒ แƒแƒ› แƒแƒ› แƒžแƒ˜แƒ แƒแƒ‘แƒ”แƒ‘แƒจแƒ˜, HCO 3 -, แƒ แƒแƒ’แƒแƒ แƒช แƒซแƒแƒšแƒ˜แƒแƒœ แƒกแƒฃแƒกแƒขแƒ˜ แƒ‘แƒแƒ–แƒ, แƒžแƒ แƒแƒฅแƒขแƒ˜แƒ™แƒฃแƒšแƒแƒ“ แƒแƒ  แƒ แƒ”แƒแƒ’แƒ˜แƒ แƒ”แƒ‘แƒก แƒฌแƒงแƒแƒšแƒ—แƒแƒœ, แƒ แƒแƒ“แƒ’แƒแƒœ แƒจแƒ”แƒกแƒแƒซแƒšแƒ แƒžแƒ แƒแƒขแƒแƒšแƒ˜แƒ–แƒ˜ แƒ—แƒ แƒ’แƒฃแƒœแƒแƒ•แƒก แƒฎแƒกแƒœแƒแƒ แƒจแƒ˜ แƒซแƒแƒšแƒ˜แƒแƒœ แƒซแƒšแƒ˜แƒ”แƒ แƒ˜ แƒคแƒฃแƒซแƒ˜แƒก แƒœแƒแƒฌแƒ˜แƒšแƒแƒ™แƒ”แƒ‘แƒ˜แƒก, แƒฐแƒ˜แƒ“แƒ แƒแƒฅแƒกแƒ˜แƒ“แƒ˜แƒก แƒ˜แƒแƒœแƒ”แƒ‘แƒ˜แƒก แƒแƒ แƒกแƒ”แƒ‘แƒแƒ‘แƒ˜แƒ—.

แƒกแƒฃแƒกแƒขแƒ˜ แƒคแƒฃแƒซแƒ˜แƒก แƒ›แƒแƒ แƒ˜แƒšแƒ˜ แƒ“แƒ แƒซแƒšแƒ˜แƒ”แƒ แƒ˜ แƒ›แƒŸแƒแƒ•แƒ

แƒ’แƒแƒœแƒ•แƒ˜แƒฎแƒ˜แƒšแƒแƒ— NH 4 Cl-แƒ˜แƒก แƒฐแƒ˜แƒ“แƒ แƒแƒšแƒ˜แƒ–แƒ˜. TED-แƒ˜แƒก แƒคแƒแƒ แƒ’แƒšแƒ”แƒ‘แƒจแƒ˜ แƒ”แƒก แƒแƒ แƒ˜แƒก แƒกแƒฃแƒกแƒขแƒ˜ แƒ›แƒแƒœแƒแƒ›แƒŸแƒแƒ•แƒฃแƒ แƒ˜ แƒคแƒฃแƒซแƒ˜แƒก แƒ›แƒแƒ แƒ˜แƒšแƒ˜ แƒ“แƒ แƒซแƒšแƒ˜แƒ”แƒ แƒ˜ แƒ›แƒŸแƒแƒ•แƒ.

แƒแƒ› แƒœแƒ˜แƒ•แƒ—แƒ˜แƒ”แƒ แƒ”แƒ‘แƒ˜แƒก แƒฎแƒกแƒœแƒแƒ แƒจแƒ˜ แƒแƒ แƒ˜แƒก แƒœแƒแƒฌแƒ˜แƒšแƒแƒ™แƒ”แƒ‘แƒ˜:

NH 4 +, Cl - แƒ“แƒ H 2 O.

NH 4 + แƒแƒ แƒ˜แƒก แƒกแƒฃแƒกแƒขแƒ˜ แƒ›แƒŸแƒแƒ•แƒ, แƒฎแƒแƒšแƒ H 2 O แƒแƒ แƒ˜แƒก แƒแƒ›แƒคแƒแƒšแƒ˜แƒขแƒ˜, แƒแƒ›แƒ˜แƒขแƒแƒ› แƒจแƒ”แƒกแƒแƒซแƒšแƒ”แƒ‘แƒ”แƒšแƒ˜แƒ แƒจแƒ”แƒฅแƒชแƒ”แƒ•แƒแƒ“แƒ˜ แƒ แƒ”แƒแƒฅแƒชแƒ˜แƒ

NH 4 + + H 2 O NH 3 + H 3 O +,

แƒ แƒแƒ›แƒšแƒ˜แƒก แƒฌแƒแƒœแƒแƒกแƒฌแƒแƒ แƒแƒ‘แƒ แƒแƒฆแƒฌแƒ”แƒ แƒ˜แƒšแƒ˜แƒ แƒแƒ›แƒแƒœแƒ˜แƒฃแƒ›แƒ˜แƒก แƒ˜แƒแƒœแƒ˜แƒก แƒ›แƒŸแƒแƒ•แƒ˜แƒแƒœแƒแƒ‘แƒ˜แƒก แƒ›แƒฃแƒ“แƒ›แƒ˜แƒ•แƒแƒ‘แƒ˜แƒ— แƒ“แƒ แƒจแƒ”แƒ˜แƒซแƒšแƒ”แƒ‘แƒ แƒ’แƒแƒ›แƒแƒ˜แƒฎแƒแƒขแƒแƒก แƒแƒ›แƒ˜แƒแƒ™แƒ˜แƒก แƒคแƒฃแƒซแƒ”แƒแƒ‘แƒ˜แƒก แƒ›แƒฃแƒ“แƒ›แƒ˜แƒ•แƒแƒ‘แƒ˜แƒ—:

K K (NH 4 +) \u003d

แƒ แƒแƒ“แƒ”แƒกแƒแƒช แƒ”แƒก แƒœแƒ˜แƒ•แƒ—แƒ˜แƒ”แƒ แƒ”แƒ‘แƒ แƒ˜แƒฎแƒกแƒœแƒ”แƒ‘แƒ, แƒ˜แƒก แƒจแƒ”แƒฃแƒฅแƒชแƒ”แƒ•แƒแƒ“ แƒ˜แƒจแƒšแƒ”แƒ‘แƒ NH 4 + แƒ“แƒ Cl - แƒ˜แƒแƒœแƒ”แƒ‘แƒแƒ“:

NH 4 Cl \u003d NH 4 + + Cl -

แƒฌแƒงแƒแƒšแƒ˜ แƒกแƒฃแƒกแƒขแƒ˜ แƒ”แƒšแƒ”แƒฅแƒขแƒ แƒแƒšแƒ˜แƒขแƒ˜แƒ แƒ“แƒ แƒจแƒ”แƒฅแƒชแƒ”แƒ•แƒแƒ“แƒแƒ“ แƒ˜แƒจแƒšแƒ”แƒ‘แƒ:

H 2 O H + + OH -

NH 4 + + OH - NH 3. H2O

แƒแƒ› แƒแƒ แƒ˜ แƒจแƒ”แƒฅแƒชแƒ”แƒ•แƒแƒ“แƒ˜ แƒ แƒ”แƒแƒฅแƒชแƒ˜แƒ˜แƒก แƒ’แƒแƒœแƒขแƒแƒšแƒ”แƒ‘แƒ”แƒ‘แƒ˜แƒก แƒ“แƒแƒ›แƒแƒขแƒ”แƒ‘แƒ แƒ“แƒ แƒ›แƒกแƒ’แƒแƒ•แƒกแƒ˜ แƒขแƒ”แƒ แƒ›แƒ˜แƒœแƒ”แƒ‘แƒ˜แƒก แƒ›แƒแƒงแƒ•แƒแƒœแƒ, แƒฉแƒ•แƒ”แƒœ แƒ•แƒ˜แƒฆแƒ”แƒ‘แƒ— แƒ˜แƒแƒœแƒฃแƒ แƒ˜ แƒฐแƒ˜แƒ“แƒ แƒแƒšแƒ˜แƒ–แƒ˜แƒก แƒ’แƒแƒœแƒขแƒแƒšแƒ”แƒ‘แƒแƒก.

NH 4 + + H 2 O NH 3. H2O+H+

แƒฐแƒ˜แƒ“แƒ แƒแƒšแƒ˜แƒ–แƒ˜แƒก แƒ แƒ”แƒแƒฅแƒชแƒ˜แƒ˜แƒก แƒฌแƒแƒœแƒแƒกแƒฌแƒแƒ แƒแƒ‘แƒ แƒแƒฆแƒฌแƒ”แƒ แƒ˜แƒšแƒ˜แƒ แƒฐแƒ˜แƒ“แƒ แƒแƒšแƒ˜แƒ–แƒ˜แƒก แƒ›แƒฃแƒ“แƒ›แƒ˜แƒ•แƒ˜แƒ— แƒ“แƒ แƒจแƒ”แƒ˜แƒซแƒšแƒ”แƒ‘แƒ แƒ’แƒแƒ›แƒแƒ˜แƒฎแƒแƒขแƒแƒก แƒแƒ›แƒ˜แƒแƒ™แƒ˜แƒก แƒฐแƒ˜แƒ“แƒ แƒแƒขแƒ˜แƒก แƒ“แƒ˜แƒกแƒแƒชแƒ˜แƒแƒชแƒ˜แƒ˜แƒก แƒ›แƒฃแƒ“แƒ›แƒ˜แƒ•แƒแƒ‘แƒ˜แƒ—:

K h =

แƒแƒ› แƒจแƒ”แƒ›แƒ—แƒฎแƒ•แƒ”แƒ•แƒแƒจแƒ˜, แƒฐแƒ˜แƒ“แƒ แƒแƒšแƒ˜แƒ–แƒ˜แƒก แƒ›แƒฃแƒ“แƒ›แƒ˜แƒ•แƒ˜ แƒฃแƒ“แƒ แƒ˜แƒก แƒแƒ›แƒแƒœแƒ˜แƒฃแƒ›แƒ˜แƒก แƒ˜แƒแƒœแƒ˜แƒก แƒ›แƒŸแƒแƒ•แƒ˜แƒแƒœแƒแƒ‘แƒ˜แƒก แƒ›แƒฃแƒ“แƒ›แƒ˜แƒ•แƒแƒก. แƒแƒ›แƒ˜แƒแƒ™แƒ˜แƒก แƒฐแƒ˜แƒ“แƒ แƒแƒขแƒ˜แƒก แƒ“แƒ˜แƒกแƒแƒชแƒ˜แƒแƒชแƒ˜แƒ˜แƒก แƒ›แƒฃแƒ“แƒ›แƒ˜แƒ•แƒ˜ แƒขแƒแƒšแƒ˜แƒ แƒแƒ›แƒ˜แƒแƒ™แƒ˜แƒก แƒ‘แƒแƒ–แƒ˜แƒกแƒฃแƒ แƒแƒ‘แƒ˜แƒก แƒ›แƒฃแƒ“แƒ›แƒ˜แƒ•แƒ—แƒแƒœ.

แƒฐแƒ˜แƒ“แƒ แƒแƒšแƒ˜แƒ–แƒ˜แƒก แƒ›แƒแƒšแƒ”แƒ™แƒฃแƒšแƒฃแƒ แƒ˜ แƒ’แƒแƒœแƒขแƒแƒšแƒ”แƒ‘แƒ (TED-แƒ˜แƒก แƒคแƒแƒ แƒ’แƒšแƒ”แƒ‘แƒจแƒ˜): NH 4 Cl + H 2 O NH 3. H2O + HCl

แƒแƒ› แƒขแƒ˜แƒžแƒ˜แƒก แƒ›แƒแƒ แƒ˜แƒšแƒ”แƒ‘แƒ˜แƒก แƒฐแƒ˜แƒ“แƒ แƒแƒšแƒ˜แƒ–แƒ˜แƒก แƒ แƒ”แƒแƒฅแƒชแƒ˜แƒ˜แƒก แƒ™แƒ˜แƒ“แƒ”แƒ• แƒ”แƒ แƒ—แƒ˜ แƒ›แƒแƒ’แƒแƒšแƒ˜แƒ—แƒ˜แƒ ZnCl 2-แƒ˜แƒก แƒฐแƒ˜แƒ“แƒ แƒแƒšแƒ˜แƒ–แƒ˜.

แƒแƒ› แƒœแƒ˜แƒ•แƒ—แƒ˜แƒ”แƒ แƒ”แƒ‘แƒ˜แƒก แƒฎแƒกแƒœแƒแƒ แƒจแƒ˜ แƒแƒ แƒ˜แƒก แƒœแƒแƒฌแƒ˜แƒšแƒแƒ™แƒ”แƒ‘แƒ˜:

Zn2+ แƒแƒ™, Cl - แƒ“แƒ H 2 O.

แƒ—แƒฃแƒ—แƒ˜แƒ˜แƒก แƒ˜แƒแƒœแƒ”แƒ‘แƒ˜ แƒแƒ แƒ˜แƒก แƒแƒ™แƒ•แƒแƒ™แƒแƒชแƒ˜แƒ”แƒ‘แƒ˜ 2+ แƒ“แƒ แƒแƒ แƒ˜แƒก แƒกแƒฃแƒกแƒขแƒ˜ แƒ™แƒแƒ—แƒ˜แƒแƒœแƒฃแƒ แƒ˜ แƒ›แƒŸแƒแƒ•แƒ”แƒ‘แƒ˜, แƒฎแƒแƒšแƒ H 2 O แƒแƒ แƒ˜แƒก แƒแƒ›แƒคแƒแƒšแƒ˜แƒขแƒ˜, แƒจแƒ”แƒกแƒแƒ‘แƒแƒ›แƒ˜แƒกแƒแƒ“, แƒจแƒ”แƒกแƒแƒซแƒšแƒ”แƒ‘แƒ”แƒšแƒ˜แƒ แƒจแƒ”แƒฅแƒชแƒ”แƒ•แƒแƒ“แƒ˜ แƒ แƒ”แƒแƒฅแƒชแƒ˜แƒ.

2= โ€‹โ€‹+ H 2 O + + H 3 O + ,

แƒ แƒแƒ›แƒšแƒ˜แƒก แƒฌแƒแƒœแƒแƒกแƒฌแƒแƒ แƒแƒ‘แƒ แƒแƒฆแƒฌแƒ”แƒ แƒ˜แƒšแƒ˜แƒ แƒ—แƒฃแƒ—แƒ˜แƒ˜แƒก แƒแƒ™แƒ•แƒแƒ™แƒแƒชแƒ˜แƒ˜แƒก แƒ›แƒŸแƒแƒ•แƒ˜แƒแƒœแƒแƒ‘แƒ˜แƒก แƒ›แƒฃแƒ“แƒ›แƒ˜แƒ•แƒแƒ‘แƒ˜แƒ— แƒ“แƒ แƒจแƒ”แƒ˜แƒซแƒšแƒ”แƒ‘แƒ แƒ’แƒแƒ›แƒแƒ˜แƒฎแƒแƒขแƒแƒก แƒขแƒ แƒ˜แƒแƒ™แƒ•แƒแƒฐแƒ˜แƒ“แƒ แƒแƒฅแƒกแƒ˜แƒœแƒ™แƒ˜แƒก แƒ˜แƒแƒœแƒ˜แƒก แƒคแƒฃแƒซแƒ”แƒ›แƒ“แƒ”แƒ‘แƒšแƒแƒ‘แƒ˜แƒก แƒ›แƒฃแƒ“แƒ›แƒ˜แƒ•แƒแƒ‘แƒ˜แƒ—:

K K (2+) =

=

แƒ แƒแƒ“แƒ”แƒกแƒแƒช แƒ”แƒก แƒœแƒ˜แƒ•แƒ—แƒ˜แƒ”แƒ แƒ”แƒ‘แƒ แƒ˜แƒฎแƒกแƒœแƒ”แƒ‘แƒ, แƒ˜แƒก แƒจแƒ”แƒฃแƒฅแƒชแƒ”แƒ•แƒแƒ“ แƒ˜แƒจแƒšแƒ”แƒ‘แƒ Zn 2+ แƒ“แƒ Cl- แƒ˜แƒแƒœแƒ”แƒ‘แƒแƒ“:

ZnCl 2 \u003d Zn 2+ + 2Cl -

แƒฌแƒงแƒแƒšแƒ˜ แƒกแƒฃแƒกแƒขแƒ˜ แƒ”แƒšแƒ”แƒฅแƒขแƒ แƒแƒšแƒ˜แƒขแƒ˜แƒ แƒ“แƒ แƒจแƒ”แƒฅแƒชแƒ”แƒ•แƒแƒ“แƒแƒ“ แƒ˜แƒจแƒšแƒ”แƒ‘แƒ:

H 2 O H + + OH -

แƒฎแƒกแƒœแƒแƒ แƒจแƒ˜ OH - แƒ“แƒ Zn 2+ แƒ˜แƒแƒœแƒ”แƒ‘แƒ˜แƒก แƒ”แƒ แƒ—แƒ“แƒ แƒแƒฃแƒšแƒ˜ แƒแƒ แƒกแƒ”แƒ‘แƒแƒ‘แƒ˜แƒ—, แƒฎแƒ“แƒ”แƒ‘แƒ แƒจแƒ”แƒฅแƒชแƒ”แƒ•แƒแƒ“แƒ˜ แƒ แƒ”แƒแƒฅแƒชแƒ˜แƒ.

Zn 2+ + OH - ZnOH +

แƒแƒ› แƒแƒ แƒ˜ แƒจแƒ”แƒฅแƒชแƒ”แƒ•แƒแƒ“แƒ˜ แƒ แƒ”แƒแƒฅแƒชแƒ˜แƒ˜แƒก แƒ’แƒแƒœแƒขแƒแƒšแƒ”แƒ‘แƒ”แƒ‘แƒ˜แƒก แƒ“แƒแƒ›แƒแƒขแƒ”แƒ‘แƒ แƒ“แƒ แƒ›แƒกแƒ’แƒแƒ•แƒกแƒ˜ แƒขแƒ”แƒ แƒ›แƒ˜แƒœแƒ”แƒ‘แƒ˜แƒก แƒ›แƒแƒงแƒ•แƒแƒœแƒ, แƒฉแƒ•แƒ”แƒœ แƒ•แƒ˜แƒฆแƒ”แƒ‘แƒ— แƒ˜แƒแƒœแƒฃแƒ แƒ˜ แƒฐแƒ˜แƒ“แƒ แƒแƒšแƒ˜แƒ–แƒ˜แƒก แƒ’แƒแƒœแƒขแƒแƒšแƒ”แƒ‘แƒแƒก.

Zn 2+ + H 2 O ZnOH + + H +

แƒฐแƒ˜แƒ“แƒ แƒแƒšแƒ˜แƒ–แƒ˜แƒก แƒ แƒ”แƒแƒฅแƒชแƒ˜แƒ˜แƒก แƒฌแƒแƒœแƒแƒกแƒฌแƒแƒ แƒแƒ‘แƒ แƒแƒฆแƒฌแƒ”แƒ แƒ˜แƒšแƒ˜แƒ แƒฐแƒ˜แƒ“แƒ แƒแƒšแƒ˜แƒ–แƒ˜แƒก แƒ›แƒฃแƒ“แƒ›แƒ˜แƒ•แƒ˜แƒ— แƒ“แƒ แƒจแƒ”แƒ˜แƒซแƒšแƒ”แƒ‘แƒ แƒ’แƒแƒ›แƒแƒ˜แƒฎแƒแƒขแƒแƒก "แƒ—แƒฃแƒ—แƒ˜แƒ˜แƒก แƒฐแƒ˜แƒ“แƒ แƒแƒฅแƒกแƒ˜แƒ“แƒ˜แƒก แƒ“แƒ˜แƒกแƒแƒชแƒ˜แƒแƒชแƒ˜แƒ˜แƒก แƒ›แƒฃแƒ“แƒ›แƒ˜แƒ•แƒ˜ แƒ›แƒ”แƒแƒ แƒ” แƒ”แƒขแƒแƒžแƒ–แƒ”":

K h =

แƒแƒ› แƒ›แƒแƒ แƒ˜แƒšแƒ˜แƒก แƒฐแƒ˜แƒ“แƒ แƒแƒšแƒ˜แƒ–แƒ˜แƒก แƒ›แƒฃแƒ“แƒ›แƒ˜แƒ•แƒ˜ แƒฃแƒ“แƒ แƒ˜แƒก แƒ—แƒฃแƒ—แƒ˜แƒ˜แƒก แƒแƒ™แƒ•แƒแƒ™แƒแƒชแƒ˜แƒ˜แƒก แƒ›แƒŸแƒแƒ•แƒ˜แƒแƒœแƒแƒ‘แƒ˜แƒก แƒ›แƒฃแƒ“แƒ›แƒ˜แƒ•แƒแƒก, แƒฎแƒแƒšแƒ แƒ—แƒฃแƒ—แƒ˜แƒ˜แƒก แƒฐแƒ˜แƒ“แƒ แƒแƒฅแƒกแƒ˜แƒ“แƒ˜แƒก แƒ“แƒ˜แƒกแƒแƒชแƒ˜แƒแƒชแƒ˜แƒ˜แƒก แƒ›แƒฃแƒ“แƒ›แƒ˜แƒ•แƒ แƒ›แƒ”แƒแƒ แƒ” แƒกแƒแƒคแƒ”แƒฎแƒฃแƒ แƒ–แƒ” แƒฃแƒ“แƒ แƒ˜แƒก + แƒ˜แƒแƒœแƒ˜แƒก แƒ‘แƒแƒ–แƒ˜แƒกแƒฃแƒ แƒแƒ‘แƒ˜แƒก แƒ›แƒฃแƒ“แƒ›แƒ˜แƒ•แƒแƒก.

.+ แƒ˜แƒแƒœแƒ˜ แƒฃแƒคแƒ แƒ แƒกแƒฃแƒกแƒขแƒ˜ แƒ›แƒŸแƒแƒ•แƒแƒ แƒ•แƒ˜แƒ“แƒ แƒ” 2+ แƒ˜แƒแƒœแƒ˜, แƒแƒ›แƒ˜แƒขแƒแƒ› แƒ˜แƒ’แƒ˜ แƒžแƒ แƒแƒฅแƒขแƒ˜แƒ™แƒฃแƒšแƒแƒ“ แƒแƒ  แƒ แƒ”แƒแƒ’แƒ˜แƒ แƒ”แƒ‘แƒก แƒฌแƒงแƒแƒšแƒ—แƒแƒœ, แƒ•แƒ˜แƒœแƒแƒ˜แƒ“แƒแƒœ แƒ”แƒก แƒ แƒ”แƒแƒฅแƒชแƒ˜แƒ แƒ—แƒ แƒ’แƒฃแƒœแƒแƒ•แƒก แƒฎแƒกแƒœแƒแƒ แƒจแƒ˜ แƒแƒฅแƒกแƒแƒœแƒ˜แƒฃแƒ›แƒ˜แƒก แƒ˜แƒแƒœแƒ”แƒ‘แƒ˜แƒก แƒแƒ แƒกแƒ”แƒ‘แƒแƒ‘แƒ˜แƒก แƒ’แƒแƒ›แƒ. TED-แƒ˜แƒก แƒคแƒแƒ แƒ’แƒšแƒ”แƒ‘แƒจแƒ˜ แƒ”แƒก แƒ’แƒแƒœแƒชแƒฎแƒแƒ“แƒ”แƒ‘แƒ แƒแƒกแƒ” แƒŸแƒฆแƒ”แƒ แƒก: "แƒ—แƒฃแƒ—แƒ˜แƒ˜แƒก แƒฅแƒšแƒแƒ แƒ˜แƒ“แƒ˜แƒก แƒฐแƒ˜แƒ“แƒ แƒแƒšแƒ˜แƒ–แƒ˜ แƒ›แƒ”แƒแƒ แƒ” แƒ”แƒขแƒแƒžแƒ–แƒ” แƒžแƒ แƒแƒฅแƒขแƒ˜แƒ™แƒฃแƒšแƒแƒ“ แƒแƒ  แƒ›แƒ˜แƒ“แƒ˜แƒก".

แƒฐแƒ˜แƒ“แƒ แƒแƒšแƒ˜แƒ–แƒ˜แƒก แƒ›แƒแƒšแƒ”แƒ™แƒฃแƒšแƒฃแƒ แƒ˜ แƒ’แƒแƒœแƒขแƒแƒšแƒ”แƒ‘แƒ (TED-แƒ˜แƒก แƒคแƒแƒ แƒ’แƒšแƒ”แƒ‘แƒจแƒ˜):

ZnCl 2 + H 2 O Zn(OH)Cl + HCl.

แƒกแƒฃแƒกแƒขแƒ˜ แƒคแƒฃแƒซแƒ˜แƒกแƒ แƒ“แƒ แƒกแƒฃแƒกแƒขแƒ˜ แƒ›แƒŸแƒแƒ•แƒ˜แƒก แƒ›แƒแƒ แƒ˜แƒšแƒ˜

แƒแƒ›แƒแƒœแƒ˜แƒฃแƒ›แƒ˜แƒก แƒ›แƒแƒ แƒ˜แƒšแƒ”แƒ‘แƒ˜แƒก แƒ’แƒแƒ แƒ“แƒ, แƒแƒกแƒ”แƒ—แƒ˜ แƒ›แƒแƒ แƒ˜แƒšแƒ”แƒ‘แƒ˜ แƒซแƒ˜แƒ แƒ˜แƒ—แƒแƒ“แƒแƒ“ แƒฌแƒงแƒแƒšแƒจแƒ˜ แƒฃแƒฎแƒกแƒœแƒแƒ“แƒ˜แƒ. แƒ›แƒแƒจแƒแƒกแƒแƒ“แƒแƒ›แƒ”, แƒ›แƒแƒ“แƒ˜แƒ— แƒ’แƒแƒœแƒ•แƒ˜แƒฎแƒ˜แƒšแƒแƒ— แƒแƒ› แƒขแƒ˜แƒžแƒ˜แƒก แƒ แƒ”แƒแƒฅแƒชแƒ˜แƒ”แƒ‘แƒ˜ แƒแƒ›แƒแƒœแƒ˜แƒฃแƒ›แƒ˜แƒก แƒชแƒ˜แƒแƒœแƒ˜แƒ“แƒ˜แƒก NH 4 CN แƒ›แƒแƒ’แƒแƒšแƒ˜แƒ—แƒ˜แƒก แƒ’แƒแƒ›แƒแƒงแƒ”แƒœแƒ”แƒ‘แƒ˜แƒ—.

แƒแƒ› แƒœแƒ˜แƒ•แƒ—แƒ˜แƒ”แƒ แƒ”แƒ‘แƒ˜แƒก แƒฎแƒกแƒœแƒแƒ แƒจแƒ˜ แƒแƒ แƒ˜แƒก แƒœแƒแƒฌแƒ˜แƒšแƒแƒ™แƒ”แƒ‘แƒ˜:

NH 4 +, CN - แƒ“แƒ H 2 O.

NH 4 + แƒแƒ แƒ˜แƒก แƒกแƒฃแƒกแƒขแƒ˜ แƒ›แƒŸแƒแƒ•แƒ, CN - แƒกแƒฃแƒกแƒขแƒ˜ แƒคแƒฃแƒซแƒ”, แƒฎแƒแƒšแƒ H 2 O แƒแƒ แƒ˜แƒก แƒแƒ›แƒคแƒแƒšแƒ˜แƒขแƒ˜, แƒแƒ›แƒ˜แƒขแƒแƒ› แƒจแƒ”แƒกแƒแƒซแƒšแƒ”แƒ‘แƒ”แƒšแƒ˜แƒ แƒแƒกแƒ”แƒ—แƒ˜ แƒจแƒ”แƒฅแƒชแƒ”แƒ•แƒแƒ“แƒ˜ แƒ แƒ”แƒแƒฅแƒชแƒ˜แƒ”แƒ‘แƒ˜:

NH 4 + + H 2 O NH 3 + H 3 O + , (1)

CN - + H 2 O HCN + OH - , (2)

NH 4 + + CN - NH 3 + HCN. (3)

แƒกแƒแƒกแƒฃแƒ แƒ•แƒ”แƒšแƒ˜แƒ แƒ‘แƒแƒšแƒ แƒ แƒ”แƒแƒฅแƒชแƒ˜แƒ, แƒ แƒแƒ“แƒ’แƒแƒœ แƒ›แƒแƒกแƒจแƒ˜, แƒžแƒ˜แƒ แƒ•แƒ”แƒšแƒ˜ แƒแƒ แƒ˜แƒกแƒ’แƒแƒœ แƒ’แƒแƒœแƒกแƒฎแƒ•แƒแƒ•แƒ”แƒ‘แƒ˜แƒ—, แƒฌแƒแƒ แƒ›แƒแƒ˜แƒฅแƒ›แƒœแƒ”แƒ‘แƒ แƒกแƒฃแƒกแƒขแƒ˜ แƒ›แƒŸแƒแƒ•แƒ แƒ“แƒ แƒกแƒฃแƒกแƒขแƒ˜ แƒคแƒฃแƒซแƒ”. แƒกแƒฌแƒแƒ แƒ”แƒ“ แƒ”แƒก แƒ แƒ”แƒแƒฅแƒชแƒ˜แƒ แƒ›แƒ˜แƒ›แƒ“แƒ˜แƒœแƒแƒ แƒ”แƒแƒ‘แƒก แƒฃแƒžแƒ˜แƒ แƒแƒขแƒ”แƒกแƒแƒ“ แƒฌแƒงแƒแƒšแƒจแƒ˜ แƒแƒ›แƒแƒœแƒ˜แƒฃแƒ›แƒ˜แƒก แƒชแƒ˜แƒแƒœแƒ˜แƒ“แƒ˜แƒก แƒ’แƒแƒฎแƒกแƒœแƒ˜แƒกแƒแƒก, แƒ›แƒแƒ’แƒ แƒแƒ› แƒแƒ›แƒ˜แƒก แƒ“แƒแƒ“แƒ’แƒ”แƒœแƒ แƒจแƒ”แƒฃแƒซแƒšแƒ”แƒ‘แƒ”แƒšแƒ˜แƒ แƒฎแƒกแƒœแƒแƒ แƒ˜แƒก แƒ›แƒŸแƒแƒ•แƒ˜แƒแƒœแƒแƒ‘แƒ˜แƒก แƒจแƒ”แƒชแƒ•แƒšแƒ˜แƒ—. แƒฎแƒกแƒœแƒแƒ แƒ˜แƒก แƒฃแƒ›แƒœแƒ˜แƒจแƒ•แƒœแƒ”แƒšแƒ แƒแƒšแƒ™แƒแƒšแƒ˜แƒ–แƒแƒชแƒ˜แƒ แƒ’แƒแƒœแƒžแƒ˜แƒ แƒแƒ‘แƒ”แƒ‘แƒฃแƒšแƒ˜แƒ แƒ˜แƒ›แƒ˜แƒ—, แƒ แƒแƒ› แƒ›แƒ”แƒแƒ แƒ” แƒ แƒ”แƒแƒฅแƒชแƒ˜แƒ แƒฏแƒ”แƒ  แƒ™แƒ˜แƒ“แƒ”แƒ• แƒ’แƒแƒ แƒ™แƒ•แƒ”แƒฃแƒšแƒฌแƒ˜แƒšแƒแƒ“ แƒฃแƒคแƒ แƒ แƒกแƒแƒกแƒฃแƒ แƒ•แƒ”แƒšแƒ˜แƒ, แƒ•แƒ˜แƒ“แƒ แƒ” แƒžแƒ˜แƒ แƒ•แƒ”แƒšแƒ˜, แƒ แƒแƒ“แƒ’แƒแƒœ แƒฐแƒ˜แƒ“แƒ แƒแƒชแƒ˜แƒแƒœแƒ›แƒŸแƒแƒ•แƒแƒก (HCN) แƒ›แƒŸแƒแƒ•แƒ˜แƒแƒœแƒแƒ‘แƒ˜แƒก แƒ›แƒฃแƒ“แƒ›แƒ˜แƒ•แƒ˜ แƒ‘แƒ”แƒ•แƒ แƒแƒ“ แƒœแƒแƒ™แƒšแƒ”แƒ‘แƒ˜แƒ แƒแƒ›แƒ˜แƒแƒ™แƒ˜แƒก แƒคแƒฃแƒซแƒ”แƒแƒ‘แƒ˜แƒก แƒ›แƒฃแƒ“แƒ›แƒ˜แƒ•แƒ–แƒ”.

แƒแƒ› แƒกแƒ˜แƒกแƒขแƒ”แƒ›แƒแƒจแƒ˜ แƒฌแƒแƒœแƒแƒกแƒฌแƒแƒ แƒแƒ‘แƒ แƒฎแƒแƒกแƒ˜แƒแƒ—แƒ“แƒ”แƒ‘แƒ แƒฐแƒ˜แƒ“แƒ แƒแƒชแƒ˜แƒแƒœแƒ›แƒŸแƒแƒ•แƒแƒก แƒ›แƒŸแƒแƒ•แƒ˜แƒแƒœแƒแƒ‘แƒ˜แƒก แƒ›แƒฃแƒ“แƒ›แƒ˜แƒ•แƒ˜แƒ—, แƒแƒ›แƒ˜แƒแƒ™แƒ˜แƒก แƒคแƒฃแƒซแƒ”แƒ›แƒ“แƒ”แƒ‘แƒšแƒแƒ‘แƒ˜แƒก แƒ›แƒฃแƒ“แƒ›แƒ˜แƒ•แƒ˜แƒ— แƒ“แƒ แƒ›แƒ”แƒกแƒแƒ›แƒ” แƒ แƒ”แƒแƒฅแƒชแƒ˜แƒ˜แƒก แƒฌแƒแƒœแƒแƒกแƒฌแƒแƒ แƒแƒ‘แƒ˜แƒก แƒ›แƒฃแƒ“แƒ›แƒ˜แƒ•แƒ˜แƒ—:

แƒžแƒ˜แƒ แƒ•แƒ”แƒšแƒ˜ แƒ’แƒแƒœแƒขแƒแƒšแƒ”แƒ‘แƒ˜แƒ“แƒแƒœ แƒ’แƒแƒ›แƒแƒ•แƒฎแƒแƒขแƒแƒ•แƒ— แƒฐแƒ˜แƒ“แƒ แƒแƒชแƒ˜แƒแƒœแƒ›แƒŸแƒแƒ•แƒแƒก แƒฌแƒแƒœแƒแƒกแƒฌแƒแƒ แƒฃแƒš แƒ™แƒแƒœแƒชแƒ”แƒœแƒขแƒ แƒแƒชแƒ˜แƒแƒก, แƒฎแƒแƒšแƒ แƒ›แƒ”แƒแƒ แƒ” แƒ’แƒแƒœแƒขแƒแƒšแƒ”แƒ‘แƒ˜แƒ“แƒแƒœ - แƒแƒ›แƒ˜แƒแƒ™แƒ˜แƒก แƒฌแƒแƒœแƒแƒกแƒฌแƒแƒ แƒฃแƒš แƒ™แƒแƒœแƒชแƒ”แƒœแƒขแƒ แƒแƒชแƒ˜แƒแƒก แƒ“แƒ แƒแƒ› แƒ แƒแƒแƒ“แƒ”แƒœแƒแƒ‘แƒ”แƒ‘แƒก แƒ•แƒชแƒ•แƒšแƒ˜แƒ— แƒ›แƒ”แƒกแƒแƒ›แƒ” แƒ’แƒแƒœแƒขแƒแƒšแƒ”แƒ‘แƒแƒจแƒ˜. แƒจแƒ”แƒ“แƒ”แƒ’แƒแƒ“, แƒฉแƒ•แƒ”แƒœ แƒ•แƒ˜แƒฆแƒ”แƒ‘แƒ—

แƒ แƒแƒ“แƒ”แƒกแƒแƒช แƒ”แƒก แƒœแƒ˜แƒ•แƒ—แƒ˜แƒ”แƒ แƒ”แƒ‘แƒ แƒ˜แƒฎแƒกแƒœแƒ”แƒ‘แƒ, แƒ˜แƒก แƒจแƒ”แƒฃแƒฅแƒชแƒ”แƒ•แƒแƒ“ แƒ˜แƒจแƒšแƒ”แƒ‘แƒ NH 4 + แƒ“แƒ CN - แƒ˜แƒแƒœแƒ”แƒ‘แƒแƒ“:

NH 4 CN \u003d NH 4 + + CN -

แƒฌแƒงแƒแƒšแƒ˜ แƒกแƒฃแƒกแƒขแƒ˜ แƒ”แƒšแƒ”แƒฅแƒขแƒ แƒแƒšแƒ˜แƒขแƒ˜แƒ แƒ“แƒ แƒจแƒ”แƒฅแƒชแƒ”แƒ•แƒแƒ“แƒแƒ“ แƒ˜แƒจแƒšแƒ”แƒ‘แƒ:

H 2 O H + + OH -

แƒฎแƒกแƒœแƒแƒ แƒจแƒ˜ OH - แƒ“แƒ NH 4 + แƒ˜แƒแƒœแƒ”แƒ‘แƒ˜แƒก แƒ”แƒ แƒ—แƒ“แƒ แƒแƒฃแƒšแƒ˜ แƒแƒ แƒกแƒ”แƒ‘แƒแƒ‘แƒ˜แƒ—, แƒฎแƒ“แƒ”แƒ‘แƒ แƒจแƒ”แƒฅแƒชแƒ”แƒ•แƒแƒ“แƒ˜ แƒ แƒ”แƒแƒฅแƒชแƒ˜แƒ.

NH 4 + + OH - NH 3. H2O

แƒ“แƒ H + แƒ“แƒ CN - แƒ˜แƒแƒœแƒ”แƒ‘แƒ˜แƒก แƒ”แƒ แƒ—แƒ“แƒ แƒแƒฃแƒšแƒ˜ แƒแƒ แƒกแƒ”แƒ‘แƒแƒ‘แƒ˜แƒ—, แƒ™แƒ˜แƒ“แƒ”แƒ• แƒ”แƒ แƒ—แƒ˜ แƒจแƒ”แƒฅแƒชแƒ”แƒ•แƒแƒ“แƒ˜ แƒ แƒ”แƒแƒฅแƒชแƒ˜แƒ แƒ›แƒ˜แƒ›แƒ“แƒ˜แƒœแƒแƒ แƒ”แƒแƒ‘แƒก

แƒแƒ› แƒกแƒแƒ›แƒ˜ แƒจแƒ”แƒฅแƒชแƒ”แƒ•แƒแƒ“แƒ˜ แƒ แƒ”แƒแƒฅแƒชแƒ˜แƒ˜แƒก แƒ’แƒแƒœแƒขแƒแƒšแƒ”แƒ‘แƒ”แƒ‘แƒ˜แƒก แƒ›แƒ˜แƒ›แƒแƒขแƒ”แƒ‘แƒ˜แƒ— แƒ“แƒ แƒ›แƒกแƒ’แƒแƒ•แƒกแƒ˜ แƒขแƒ”แƒ แƒ›แƒ˜แƒœแƒ”แƒ‘แƒ˜แƒก แƒ›แƒแƒงแƒ•แƒแƒœแƒ˜แƒ—, แƒ›แƒ˜แƒ•แƒ˜แƒฆแƒ”แƒ‘แƒ— แƒ˜แƒแƒœแƒฃแƒ แƒ˜ แƒฐแƒ˜แƒ“แƒ แƒแƒšแƒ˜แƒ–แƒ˜แƒก แƒ’แƒแƒœแƒขแƒแƒšแƒ”แƒ‘แƒแƒก.

NH 4 + + CN - + H 2 O NH 3. H2O + HCN

แƒฐแƒ˜แƒ“แƒ แƒแƒšแƒ˜แƒ–แƒ˜แƒก แƒ›แƒฃแƒ“แƒ›แƒ˜แƒ•แƒ˜แƒก แƒคแƒแƒ แƒ›แƒ แƒแƒ› แƒจแƒ”แƒ›แƒ—แƒฎแƒ•แƒ”แƒ•แƒแƒจแƒ˜ แƒจแƒ”แƒ›แƒ“แƒ”แƒ’แƒ˜แƒ:

K h =

แƒ“แƒ แƒ”แƒก แƒจแƒ”แƒ˜แƒซแƒšแƒ”แƒ‘แƒ แƒ’แƒแƒ›แƒแƒ˜แƒฎแƒแƒขแƒแƒก แƒแƒ›แƒ˜แƒแƒ™แƒ˜แƒก แƒฐแƒ˜แƒ“แƒ แƒแƒขแƒ˜แƒก แƒ“แƒ˜แƒกแƒแƒชแƒ˜แƒแƒชแƒ˜แƒ˜แƒก แƒ›แƒฃแƒ“แƒ›แƒ˜แƒ•แƒแƒ‘แƒ˜แƒ— แƒ“แƒ แƒฐแƒ˜แƒ“แƒ แƒแƒชแƒ˜แƒแƒœแƒ›แƒŸแƒแƒ•แƒแƒก แƒ“แƒ˜แƒกแƒแƒชแƒ˜แƒแƒชแƒ˜แƒ˜แƒก แƒ›แƒฃแƒ“แƒ›แƒ˜แƒ•แƒ˜แƒ—:

K h =

แƒฐแƒ˜แƒ“แƒ แƒแƒšแƒ˜แƒ–แƒ˜แƒก แƒ›แƒแƒšแƒ”แƒ™แƒฃแƒšแƒฃแƒ แƒ˜ แƒ’แƒแƒœแƒขแƒแƒšแƒ”แƒ‘แƒ (TED-แƒ˜แƒก แƒคแƒแƒ แƒ’แƒšแƒ”แƒ‘แƒจแƒ˜):

NH 4 CN + H 2 O NH 3. H2O + HCN

20.5. แƒฎแƒกแƒœแƒแƒ“แƒแƒ‘แƒ˜แƒก แƒ›แƒฃแƒ“แƒ›แƒ˜แƒ•แƒ˜ (แƒฎแƒกแƒœแƒแƒ“แƒแƒ‘แƒ˜แƒก แƒžแƒ แƒแƒ“แƒฃแƒฅแƒขแƒ˜)

แƒฌแƒงแƒแƒšแƒจแƒ˜ แƒ›แƒงแƒแƒ แƒ˜ แƒœแƒ˜แƒ•แƒ—แƒ˜แƒ”แƒ แƒ”แƒ‘แƒ˜แƒก แƒฅแƒ˜แƒ›แƒ˜แƒฃแƒ แƒ˜ แƒ“แƒแƒจแƒšแƒ˜แƒก แƒžแƒ แƒแƒชแƒ”แƒกแƒ˜ (แƒ“แƒ แƒแƒ แƒ แƒ›แƒฎแƒแƒšแƒแƒ“ แƒฌแƒงแƒแƒšแƒจแƒ˜) แƒจแƒ”แƒ˜แƒซแƒšแƒ”แƒ‘แƒ แƒ’แƒแƒ›แƒแƒ˜แƒกแƒแƒฎแƒแƒก แƒ’แƒแƒœแƒขแƒแƒšแƒ”แƒ‘แƒ˜แƒ—. แƒ›แƒแƒ’แƒแƒšแƒ˜แƒ—แƒแƒ“, แƒœแƒแƒขแƒ แƒ˜แƒฃแƒ›แƒ˜แƒก แƒฅแƒšแƒแƒ แƒ˜แƒ“แƒ˜แƒก แƒ“แƒแƒจแƒšแƒ˜แƒก แƒจแƒ”แƒ›แƒ—แƒฎแƒ•แƒ”แƒ•แƒแƒจแƒ˜:

NaCl cr + ( แƒœ+แƒ›)H 2 O = + + -

แƒ”แƒก แƒ’แƒแƒœแƒขแƒแƒšแƒ”แƒ‘แƒ แƒชแƒแƒšแƒกแƒแƒฎแƒแƒ“ แƒแƒฉแƒ•แƒ”แƒœแƒ”แƒ‘แƒก, แƒ แƒแƒ› แƒœแƒแƒขแƒ แƒ˜แƒฃแƒ›แƒ˜แƒก แƒฅแƒšแƒแƒ แƒ˜แƒ“แƒ˜แƒก แƒ“แƒแƒจแƒšแƒ˜แƒก แƒงแƒ•แƒ”แƒšแƒแƒ–แƒ” แƒ›แƒœแƒ˜แƒจแƒ•แƒœแƒ”แƒšแƒแƒ•แƒแƒœแƒ˜ แƒ›แƒ˜แƒ–แƒ”แƒ–แƒ˜ แƒแƒ แƒ˜แƒก Na + แƒ“แƒ Cl - แƒ˜แƒแƒœแƒ”แƒ‘แƒ˜แƒก แƒฐแƒ˜แƒ“แƒ แƒแƒขแƒแƒชแƒ˜แƒ.

แƒ’แƒแƒฏแƒ”แƒ แƒ”แƒ‘แƒฃแƒš แƒฎแƒกแƒœแƒแƒ แƒจแƒ˜ แƒ“แƒแƒ›แƒงแƒแƒ แƒ”แƒ‘แƒฃแƒšแƒ˜แƒ แƒฐแƒ”แƒขแƒ”แƒ แƒแƒคแƒแƒ–แƒ˜แƒก แƒฌแƒแƒœแƒแƒกแƒฌแƒแƒ แƒแƒ‘แƒ:

NaCl cr + ( แƒœ+แƒ›)H 2 O + + -,

แƒ แƒแƒ›แƒ”แƒšแƒ˜แƒช แƒ”แƒ›แƒแƒ แƒฉแƒ˜แƒšแƒ”แƒ‘แƒ แƒ›แƒแƒกแƒแƒ‘แƒ แƒ˜แƒ•แƒ˜ แƒ›แƒแƒฅแƒ›แƒ”แƒ“แƒ”แƒ‘แƒ˜แƒก แƒ™แƒแƒœแƒแƒœแƒก. แƒ›แƒแƒ’แƒ แƒแƒ›, แƒ แƒแƒ“แƒ’แƒแƒœ แƒœแƒแƒขแƒ แƒ˜แƒฃแƒ›แƒ˜แƒก แƒฅแƒšแƒแƒ แƒ˜แƒ“แƒ˜แƒก แƒฎแƒกแƒœแƒแƒ“แƒแƒ‘แƒ แƒกแƒแƒ™แƒ›แƒแƒแƒ“ แƒ›แƒœแƒ˜แƒจแƒ•แƒœแƒ”แƒšแƒแƒ•แƒแƒœแƒ˜แƒ, แƒฌแƒแƒœแƒแƒกแƒฌแƒแƒ แƒแƒ‘แƒ˜แƒก แƒ›แƒฃแƒ“แƒ›แƒ˜แƒ•แƒ˜แƒก แƒ’แƒแƒ›แƒแƒฎแƒแƒขแƒฃแƒšแƒ”แƒ‘แƒ แƒแƒ› แƒจแƒ”แƒ›แƒ—แƒฎแƒ•แƒ”แƒ•แƒแƒจแƒ˜ แƒจแƒ”แƒ˜แƒซแƒšแƒ”แƒ‘แƒ แƒ“แƒแƒ˜แƒฌแƒ”แƒ แƒแƒก แƒ›แƒฎแƒแƒšแƒแƒ“ แƒ˜แƒแƒœแƒ”แƒ‘แƒ˜แƒก แƒแƒฅแƒขแƒ˜แƒ•แƒแƒ‘แƒ”แƒ‘แƒ˜แƒก แƒ’แƒแƒ›แƒแƒงแƒ”แƒœแƒ”แƒ‘แƒ˜แƒ—, แƒ แƒแƒ›แƒšแƒ”แƒ‘แƒ˜แƒช แƒจแƒแƒ แƒก แƒแƒ แƒ˜แƒก แƒงแƒแƒ•แƒ”แƒšแƒ—แƒ•แƒ˜แƒก แƒชแƒœแƒแƒ‘แƒ˜แƒšแƒ˜.

แƒชแƒฃแƒ“แƒแƒ“ แƒฎแƒกแƒœแƒแƒ“แƒ˜ (แƒแƒœ แƒžแƒ แƒแƒฅแƒขแƒ˜แƒ™แƒฃแƒšแƒแƒ“ แƒฃแƒฎแƒกแƒœแƒแƒ“แƒ˜ แƒœแƒ˜แƒ•แƒ—แƒ˜แƒ”แƒ แƒ”แƒ‘แƒ˜แƒก) แƒฎแƒกแƒœแƒแƒ แƒจแƒ˜ แƒฌแƒแƒœแƒแƒกแƒฌแƒแƒ แƒแƒ‘แƒ˜แƒก แƒจแƒ”แƒ›แƒ—แƒฎแƒ•แƒ”แƒ•แƒแƒจแƒ˜, แƒ’แƒแƒฏแƒ”แƒ แƒ”แƒ‘แƒฃแƒš แƒฎแƒกแƒœแƒแƒ แƒจแƒ˜ แƒฌแƒแƒœแƒแƒกแƒฌแƒแƒ แƒแƒ‘แƒ˜แƒก แƒ›แƒฃแƒ“แƒ›แƒ˜แƒ•แƒ˜แƒก แƒ’แƒแƒ›แƒแƒฎแƒแƒขแƒฃแƒšแƒ”แƒ‘แƒ แƒจแƒ”แƒ˜แƒซแƒšแƒ”แƒ‘แƒ แƒ“แƒแƒ˜แƒฌแƒ”แƒ แƒแƒก แƒฌแƒแƒœแƒแƒกแƒฌแƒแƒ แƒฃแƒšแƒ˜ แƒ™แƒแƒœแƒชแƒ”แƒœแƒขแƒ แƒแƒชแƒ˜แƒ”แƒ‘แƒ˜แƒก แƒ’แƒแƒ›แƒแƒงแƒ”แƒœแƒ”แƒ‘แƒ˜แƒ—. แƒ›แƒแƒ’แƒแƒšแƒ˜แƒ—แƒแƒ“, แƒฌแƒแƒœแƒแƒกแƒฌแƒแƒ แƒแƒ‘แƒ˜แƒกแƒ—แƒ•แƒ˜แƒก แƒ•แƒ”แƒ แƒชแƒฎแƒšแƒ˜แƒก แƒฅแƒšแƒแƒ แƒ˜แƒ“แƒ˜แƒก แƒ’แƒแƒฏแƒ”แƒ แƒ”แƒ‘แƒฃแƒš แƒฎแƒกแƒœแƒแƒ แƒจแƒ˜

AgCl cr + ( แƒœ+แƒ›)H 2 O + + -

แƒ•แƒ˜แƒœแƒแƒ˜แƒ“แƒแƒœ แƒฌแƒงแƒšแƒ˜แƒก แƒฌแƒแƒœแƒแƒกแƒฌแƒแƒ แƒฃแƒšแƒ˜ แƒ™แƒแƒœแƒชแƒ”แƒœแƒขแƒ แƒแƒชแƒ˜แƒ แƒ’แƒแƒœแƒ–แƒแƒ•แƒ”แƒ‘แƒฃแƒš แƒฎแƒกแƒœแƒแƒ แƒจแƒ˜ แƒ—แƒ˜แƒ—แƒฅแƒ›แƒ˜แƒก แƒ›แƒฃแƒ“แƒ›แƒ˜แƒ•แƒ˜แƒ, แƒจแƒ”แƒ’แƒ•แƒ˜แƒซแƒšแƒ˜แƒ แƒ“แƒแƒ•แƒฌแƒ”แƒ แƒแƒ—

K G (AgCl) = K C . แƒœ+แƒ› = .

แƒ˜แƒ’แƒ˜แƒ•แƒ” แƒ’แƒแƒ›แƒแƒ แƒขแƒ˜แƒ•แƒ“แƒ

K G (AgCl) = แƒแƒœ แƒ™ G(AgCl) =

แƒจแƒ”แƒ“แƒ”แƒ’แƒแƒ“ แƒ›แƒ˜แƒฆแƒ”แƒ‘แƒฃแƒšแƒ˜ แƒ›แƒœแƒ˜แƒจแƒ•แƒœแƒ”แƒšแƒแƒ‘แƒ ( แƒ™แƒ“) แƒ“แƒแƒกแƒแƒฎแƒ”แƒšแƒ”แƒ‘แƒฃแƒšแƒ˜แƒ แƒฐแƒ˜แƒ“แƒ แƒแƒขแƒแƒชแƒ˜แƒ˜แƒก แƒ›แƒฃแƒ“แƒ›แƒ˜แƒ•แƒ”แƒ‘แƒ˜(แƒœแƒ”แƒ‘แƒ˜แƒกแƒ›แƒ˜แƒ”แƒ แƒ˜ แƒ“แƒ แƒแƒ แƒ แƒ›แƒฎแƒแƒšแƒแƒ“ แƒฌแƒงแƒแƒšแƒฎแƒกแƒœแƒแƒ แƒ˜แƒก แƒจแƒ”แƒ›แƒ—แƒฎแƒ•แƒ”แƒ•แƒแƒจแƒ˜ - แƒแƒ›แƒแƒฎแƒกแƒœแƒ˜แƒก แƒ›แƒฃแƒ“แƒ›แƒ˜แƒ•แƒ”แƒ‘แƒ˜).

แƒ”แƒšแƒ”แƒฅแƒขแƒ แƒแƒšแƒ˜แƒขแƒฃแƒ แƒ˜ แƒ“แƒ˜แƒกแƒแƒชแƒ˜แƒแƒชแƒ˜แƒ˜แƒก แƒ—แƒ”แƒแƒ แƒ˜แƒ˜แƒก แƒคแƒแƒ แƒ’แƒšแƒ”แƒ‘แƒจแƒ˜, แƒฌแƒแƒœแƒแƒกแƒฌแƒแƒ แƒแƒ‘แƒ AgCl แƒฎแƒกแƒœแƒแƒ แƒจแƒ˜ แƒ˜แƒฌแƒ”แƒ แƒ”แƒ‘แƒ แƒจแƒ”แƒ›แƒ“แƒ”แƒ’แƒœแƒแƒ˜แƒ แƒแƒ“:

AgCl cr Ag + + Cl โ€“

แƒจแƒ”แƒกแƒแƒ‘แƒแƒ›แƒ˜แƒกแƒ˜ แƒ›แƒฃแƒ“แƒ›แƒ˜แƒ•แƒ˜ แƒ”แƒฌแƒแƒ“แƒ”แƒ‘แƒ แƒฎแƒกแƒœแƒแƒ“แƒแƒ‘แƒ˜แƒก แƒžแƒ แƒแƒ“แƒฃแƒฅแƒขแƒ˜แƒ“แƒ แƒแƒฆแƒ˜แƒœแƒ˜แƒจแƒœแƒ”แƒ‘แƒ แƒแƒกแƒแƒ”แƒ‘แƒ˜แƒ— PR.

PR (AgCl) =

แƒคแƒแƒ แƒ›แƒฃแƒšแƒ˜แƒก แƒ”แƒ แƒ—แƒ”แƒฃแƒšแƒจแƒ˜ แƒ™แƒแƒ—แƒ˜แƒแƒœแƒ”แƒ‘แƒ˜แƒกแƒ แƒ“แƒ แƒแƒœแƒ˜แƒแƒœแƒ”แƒ‘แƒ˜แƒก แƒ—แƒแƒœแƒแƒคแƒแƒ แƒ“แƒแƒ‘แƒ˜แƒ“แƒแƒœ แƒ’แƒแƒ›แƒแƒ›แƒ“แƒ˜แƒœแƒแƒ แƒ”, แƒฎแƒกแƒœแƒแƒ“แƒแƒ‘แƒ˜แƒก แƒ›แƒฃแƒ“แƒ›แƒ˜แƒ•แƒ˜แƒก แƒ’แƒแƒ›แƒแƒฎแƒแƒขแƒฃแƒšแƒ”แƒ‘แƒ (แƒฎแƒกแƒœแƒแƒ“แƒแƒ‘แƒ˜แƒก แƒžแƒ แƒแƒ“แƒฃแƒฅแƒขแƒ˜) แƒจแƒ”แƒ˜แƒซแƒšแƒ”แƒ‘แƒ แƒ’แƒแƒœแƒกแƒฎแƒ•แƒแƒ•แƒ”แƒ‘แƒฃแƒšแƒ˜ แƒ˜แƒงแƒแƒก, แƒ›แƒแƒ’แƒแƒšแƒ˜แƒ—แƒแƒ“:

แƒ–แƒแƒ’แƒ˜แƒ”แƒ แƒ—แƒ˜ แƒชแƒฃแƒ“แƒแƒ“ แƒฎแƒกแƒœแƒแƒ“แƒ˜ แƒœแƒ˜แƒ•แƒ—แƒ˜แƒ”แƒ แƒ”แƒ‘แƒ˜แƒก แƒฐแƒ˜แƒ“แƒ แƒแƒขแƒแƒชแƒ˜แƒ˜แƒก แƒ›แƒฃแƒ“แƒ›แƒ˜แƒ•แƒ”แƒ‘แƒ˜แƒก (แƒฎแƒกแƒœแƒแƒ“แƒแƒ‘แƒ˜แƒก แƒžแƒ แƒแƒ“แƒฃแƒฅแƒขแƒ”แƒ‘แƒ˜แƒก) แƒ›แƒœแƒ˜แƒจแƒ•แƒœแƒ”แƒšแƒแƒ‘แƒ”แƒ‘แƒ˜ แƒ›แƒแƒชแƒ”แƒ›แƒฃแƒšแƒ˜แƒ แƒ“แƒแƒœแƒแƒ แƒ— 15-แƒจแƒ˜.

แƒฎแƒกแƒœแƒแƒ“แƒแƒ‘แƒ˜แƒก แƒžแƒ แƒแƒ“แƒฃแƒฅแƒขแƒ˜แƒก แƒชแƒแƒ“แƒœแƒ, แƒแƒ“แƒ•แƒ˜แƒšแƒ˜แƒ แƒ’แƒแƒ›แƒแƒ•แƒ—แƒ•แƒแƒšแƒแƒ— แƒœแƒ˜แƒ•แƒ—แƒ˜แƒ”แƒ แƒ”แƒ‘แƒ˜แƒก แƒ™แƒแƒœแƒชแƒ”แƒœแƒขแƒ แƒแƒชแƒ˜แƒ แƒ’แƒแƒฏแƒ”แƒ แƒ”แƒ‘แƒฃแƒš แƒฎแƒกแƒœแƒแƒ แƒจแƒ˜. แƒ›แƒแƒ’แƒแƒšแƒ˜แƒ—แƒ”แƒ‘แƒ˜:

1. BaSO 4cr Ba 2+ + SO 4 2-

PR (BaSO 4) \u003d \u003d 1.8. 10โ€“10 แƒ›แƒแƒšแƒ˜ 2/แƒš 2.

c(BaSO4) = = = = = 1.34. 10โ€“5 แƒ›แƒแƒš/แƒš.

2. Ca(OH) 2cr Ca 2+ + 2OH -

PR \u003d 2 \u003d 6.3. 10 โ€“6 แƒ›แƒแƒšแƒ˜ 3 /แƒš 3 .

2 PR = (2) 2 = 4 3

แƒ’ == = = 1.16. 10-2 แƒ›แƒแƒš/แƒš.

แƒ—แƒฃ แƒฅแƒ˜แƒ›แƒ˜แƒฃแƒ แƒ˜ แƒ แƒ”แƒแƒฅแƒชแƒ˜แƒ˜แƒก แƒ“แƒ แƒแƒก แƒฎแƒกแƒœแƒแƒ แƒจแƒ˜ แƒฉแƒœแƒ“แƒ”แƒ‘แƒ แƒ˜แƒแƒœแƒ”แƒ‘แƒ˜, แƒ แƒแƒ›แƒšแƒ”แƒ‘แƒ˜แƒช แƒชแƒฃแƒ“แƒแƒ“ แƒฎแƒกแƒœแƒแƒ“แƒ˜ แƒœแƒ˜แƒ•แƒ—แƒ˜แƒ”แƒ แƒ”แƒ‘แƒ˜แƒก แƒœแƒแƒฌแƒ˜แƒšแƒ˜แƒ, แƒ›แƒแƒจแƒ˜แƒœ, แƒแƒ› แƒœแƒ˜แƒ•แƒ—แƒ˜แƒ”แƒ แƒ”แƒ‘แƒ˜แƒก แƒฎแƒกแƒœแƒแƒ“แƒแƒ‘แƒ˜แƒก แƒžแƒ แƒแƒ“แƒฃแƒฅแƒขแƒ˜แƒก แƒชแƒแƒ“แƒœแƒ˜แƒ—, แƒแƒ“แƒ•แƒ˜แƒšแƒ˜แƒ แƒ˜แƒ›แƒ˜แƒก แƒ“แƒแƒ“แƒ’แƒ”แƒœแƒ, แƒ’แƒแƒฉแƒœแƒ“แƒ”แƒ‘แƒ แƒ—แƒฃ แƒแƒ แƒ แƒ˜แƒก แƒœแƒแƒšแƒ”แƒฅแƒ˜.
แƒ›แƒแƒ’แƒแƒšแƒ˜แƒ—แƒ”แƒ‘แƒ˜:

1. แƒ“แƒแƒ˜แƒšแƒ”แƒฅแƒ”แƒ‘แƒ แƒ—แƒฃ แƒแƒ แƒ แƒกแƒžแƒ˜แƒšแƒ”แƒœแƒซแƒ˜แƒก แƒฐแƒ˜แƒ“แƒ แƒแƒฅแƒกแƒ˜แƒ“แƒ˜, แƒ แƒแƒ“แƒ”แƒกแƒแƒช 100 แƒ›แƒš 0,01 แƒ› แƒ™แƒแƒšแƒชแƒ˜แƒฃแƒ›แƒ˜แƒก แƒฐแƒ˜แƒ“แƒ แƒแƒฅแƒกแƒ˜แƒ“แƒ˜แƒก แƒฎแƒกแƒœแƒแƒ แƒก แƒ”แƒ›แƒแƒขแƒ”แƒ‘แƒ 0,001 แƒ› แƒกแƒžแƒ˜แƒšแƒ”แƒœแƒซแƒ˜แƒก แƒกแƒฃแƒšแƒคแƒแƒขแƒ˜แƒก แƒ—แƒแƒœแƒแƒ‘แƒแƒ  แƒ›แƒแƒชแƒฃแƒšแƒแƒ‘แƒแƒก?

Cu 2+ + 2OH - Cu (OH) 2

แƒกแƒžแƒ˜แƒšแƒ”แƒœแƒซแƒ˜แƒก แƒฐแƒ˜แƒ“แƒ แƒแƒฅแƒกแƒ˜แƒ“แƒ˜แƒก แƒœแƒแƒšแƒ”แƒฅแƒ˜ แƒฌแƒแƒ แƒ›แƒแƒ˜แƒฅแƒ›แƒœแƒ”แƒ‘แƒ, แƒ—แƒฃ Cu 2+ แƒ“แƒ OH - แƒ˜แƒแƒœแƒ”แƒ‘แƒ˜แƒก แƒ™แƒแƒœแƒชแƒ”แƒœแƒขแƒ แƒแƒชแƒ˜แƒ”แƒ‘แƒ˜แƒก แƒžแƒ แƒแƒ“แƒฃแƒฅแƒขแƒ˜ แƒแƒฆแƒ”แƒ›แƒแƒขแƒ”แƒ‘แƒ แƒแƒ› แƒœแƒแƒ™แƒšแƒ”แƒ‘แƒแƒ“ แƒฎแƒกแƒœแƒแƒ“แƒ˜ แƒฐแƒ˜แƒ“แƒ แƒแƒฅแƒกแƒ˜แƒ“แƒ˜แƒก แƒฎแƒกแƒœแƒแƒ“แƒแƒ‘แƒ˜แƒก แƒžแƒ แƒแƒ“แƒฃแƒฅแƒขแƒก. แƒ—แƒแƒœแƒแƒ‘แƒแƒ แƒ˜ แƒ›แƒแƒชแƒฃแƒšแƒแƒ‘แƒ˜แƒก แƒฎแƒกแƒœแƒแƒ แƒ”แƒ‘แƒ˜แƒก แƒฉแƒแƒ›แƒแƒกแƒฎแƒ›แƒ˜แƒก แƒจแƒ”แƒ›แƒ“แƒ”แƒ’, แƒฎแƒกแƒœแƒแƒ แƒ˜แƒก แƒ›แƒ—แƒšแƒ˜แƒแƒœแƒ˜ แƒ›แƒแƒชแƒฃแƒšแƒแƒ‘แƒ แƒ’แƒแƒฎแƒ“แƒ”แƒ‘แƒ แƒแƒ แƒฏแƒ”แƒ  แƒ“แƒ˜แƒ“แƒ˜, แƒ•แƒ˜แƒ“แƒ แƒ” แƒ—แƒ˜แƒ—แƒแƒ”แƒฃแƒšแƒ˜ แƒกแƒแƒฌแƒงแƒ˜แƒกแƒ˜ แƒฎแƒกแƒœแƒแƒ แƒ˜แƒก แƒ›แƒแƒชแƒฃแƒšแƒแƒ‘แƒ, แƒจแƒ”แƒกแƒแƒ‘แƒแƒ›แƒ˜แƒกแƒแƒ“, แƒ—แƒ˜แƒ—แƒแƒ”แƒฃแƒšแƒ˜ แƒ›แƒแƒซแƒ แƒแƒ•แƒ˜ แƒœแƒ˜แƒ•แƒ—แƒ˜แƒ”แƒ แƒ”แƒ‘แƒ˜แƒก แƒ™แƒแƒœแƒชแƒ”แƒœแƒขแƒ แƒแƒชแƒ˜แƒ (แƒ แƒ”แƒแƒฅแƒชแƒ˜แƒ˜แƒก แƒ“แƒแƒฌแƒงแƒ”แƒ‘แƒแƒ›แƒ“แƒ”) แƒ’แƒแƒœแƒแƒฎแƒ”แƒ•แƒ แƒ“แƒ”แƒ‘แƒ. แƒ™แƒแƒœแƒชแƒ”แƒœแƒขแƒ แƒแƒชแƒ˜แƒ แƒกแƒžแƒ˜แƒšแƒ”แƒœแƒซแƒ˜แƒก แƒ˜แƒแƒœแƒ”แƒ‘แƒ˜แƒก แƒ›แƒ˜แƒฆแƒ”แƒ‘แƒฃแƒš แƒฎแƒกแƒœแƒแƒ แƒจแƒ˜

c(Cu 2+) \u003d (0,001 แƒ›แƒแƒš/แƒš): 2 \u003d 0,0005 แƒ›แƒแƒš/แƒš.

แƒฐแƒ˜แƒ“แƒ แƒแƒฅแƒกแƒ˜แƒ“แƒ˜แƒก แƒ˜แƒแƒœแƒ”แƒ‘แƒ˜แƒก แƒ™แƒแƒœแƒชแƒ”แƒœแƒขแƒ แƒแƒชแƒ˜แƒ -

c (OH -) \u003d (2. 0.01 แƒ›แƒแƒš / แƒš): 2 \u003d 0.01 แƒ›แƒแƒš / แƒš.

แƒกแƒžแƒ˜แƒšแƒ”แƒœแƒซแƒ˜แƒก แƒฐแƒ˜แƒ“แƒ แƒแƒฅแƒกแƒ˜แƒ“แƒ˜แƒก แƒฎแƒกแƒœแƒแƒ“แƒแƒ‘แƒ˜แƒก แƒžแƒ แƒแƒ“แƒฃแƒฅแƒขแƒ˜

PR \u003d 2 \u003d 5.6. 10โ€“20 แƒ›แƒแƒšแƒ˜ 3/แƒš 3.

c(Cu 2+) . ( แƒ’(OH -)) 2 \u003d 0,0005 แƒ›แƒแƒš / แƒš. (0,01 แƒ›แƒแƒš/แƒš) 2 \u003d 5. 10โ€“8 แƒ›แƒแƒšแƒ˜ 3 / แƒš 3 .

แƒžแƒ แƒแƒ“แƒฃแƒฅแƒขแƒ˜แƒก แƒ™แƒแƒœแƒชแƒ”แƒœแƒขแƒ แƒแƒชแƒ˜แƒ แƒ›แƒ”แƒขแƒ˜แƒ แƒฎแƒกแƒœแƒแƒ“แƒแƒ‘แƒ˜แƒก แƒžแƒ แƒแƒ“แƒฃแƒฅแƒขแƒ–แƒ”, แƒแƒ›แƒ˜แƒขแƒแƒ› แƒฌแƒแƒ แƒ›แƒแƒ˜แƒฅแƒ›แƒœแƒ”แƒ‘แƒ แƒœแƒแƒšแƒ”แƒฅแƒ˜.

2. 0,02 แƒ› แƒœแƒแƒขแƒ แƒ˜แƒฃแƒ›แƒ˜แƒก แƒกแƒฃแƒšแƒคแƒแƒขแƒ˜แƒก แƒฎแƒกแƒœแƒแƒ แƒ˜แƒก แƒ“แƒ 0,04 แƒ› แƒ•แƒ”แƒ แƒชแƒฎแƒšแƒ˜แƒก แƒœแƒ˜แƒขแƒ แƒแƒขแƒ˜แƒก แƒฎแƒกแƒœแƒแƒ แƒ˜แƒก แƒ—แƒแƒœแƒแƒ‘แƒแƒ แƒ˜ แƒ›แƒแƒชแƒฃแƒšแƒแƒ‘แƒ”แƒ‘แƒ˜แƒก แƒฉแƒแƒ›แƒแƒกแƒฎแƒ›แƒ˜แƒกแƒแƒก แƒœแƒแƒšแƒ”แƒฅแƒ˜ แƒ’แƒแƒ›แƒแƒ•แƒ แƒ•แƒ”แƒ แƒชแƒฎแƒšแƒ˜แƒก แƒกแƒฃแƒšแƒคแƒแƒขแƒ˜?

2Ag + + SO 4 2- Ag 2 SO 4

แƒ™แƒแƒœแƒชแƒ”แƒœแƒขแƒ แƒแƒชแƒ˜แƒ แƒ•แƒ”แƒ แƒชแƒฎแƒšแƒ˜แƒก แƒ˜แƒแƒœแƒ”แƒ‘แƒ˜แƒก แƒ›แƒ˜แƒฆแƒ”แƒ‘แƒฃแƒš แƒฎแƒกแƒœแƒแƒ แƒจแƒ˜

c (Ag +) \u003d (0.04 แƒ›แƒแƒš / แƒš): 2 \u003d 0.02 แƒ›แƒแƒš / แƒš.

แƒกแƒฃแƒšแƒคแƒแƒขแƒ˜แƒก แƒ˜แƒแƒœแƒ”แƒ‘แƒ˜แƒก แƒ›แƒ˜แƒฆแƒ”แƒ‘แƒฃแƒš แƒฎแƒกแƒœแƒแƒ แƒจแƒ˜ แƒ™แƒแƒœแƒชแƒ”แƒœแƒขแƒ แƒแƒชแƒ˜แƒ

c(SO 4 2-) \u003d (0,02 แƒ›แƒแƒš/แƒš): 2 \u003d 0,01 แƒ›แƒแƒš/แƒš.

แƒ•แƒ”แƒ แƒชแƒฎแƒšแƒ˜แƒก แƒกแƒฃแƒšแƒคแƒแƒขแƒ˜แƒก แƒฎแƒกแƒœแƒแƒ“แƒแƒ‘แƒ˜แƒก แƒžแƒ แƒแƒ“แƒฃแƒฅแƒขแƒ˜

PR (Ag 2 SO 4) \u003d 2. \u003d 1.2. 10โ€“5 แƒ›แƒแƒšแƒ˜ 3/แƒš 3.

แƒฎแƒกแƒœแƒแƒ แƒจแƒ˜ แƒ˜แƒแƒœแƒ”แƒ‘แƒ˜แƒก แƒ™แƒแƒœแƒชแƒ”แƒœแƒขแƒ แƒแƒชแƒ˜แƒ˜แƒก แƒžแƒ แƒแƒ“แƒฃแƒฅแƒขแƒ˜

{แƒ’(แƒแƒ’ +)) 2. แƒ’(SO 4 2-) \u003d (0,02 แƒ›แƒแƒš/แƒš) 2. 0,01 แƒ›แƒแƒš/แƒš \u003d 4. 10 โ€“6 แƒ›แƒแƒšแƒ˜ 3 /แƒš 3 .

แƒ™แƒแƒœแƒชแƒ”แƒœแƒขแƒ แƒแƒชแƒ˜แƒ˜แƒก แƒžแƒ แƒแƒ“แƒฃแƒฅแƒขแƒ˜ แƒœแƒแƒ™แƒšแƒ”แƒ‘แƒ˜แƒ แƒฎแƒกแƒœแƒแƒ“แƒแƒ‘แƒ˜แƒก แƒžแƒ แƒแƒ“แƒฃแƒฅแƒขแƒ–แƒ”, แƒแƒ›แƒ˜แƒขแƒแƒ› แƒœแƒแƒšแƒ”แƒฅแƒ˜ แƒแƒ  แƒฌแƒแƒ แƒ›แƒแƒ˜แƒฅแƒ›แƒœแƒ”แƒ‘แƒ.

20.6. แƒ’แƒแƒ แƒ“แƒแƒฅแƒ›แƒœแƒ˜แƒก แƒฎแƒแƒ แƒ˜แƒกแƒฎแƒ˜ (แƒžแƒ แƒแƒขแƒแƒšแƒ˜แƒ–แƒ˜แƒก แƒฎแƒแƒ แƒ˜แƒกแƒฎแƒ˜, แƒ“แƒ˜แƒกแƒแƒชแƒ˜แƒแƒชแƒ˜แƒ˜แƒก แƒฎแƒแƒ แƒ˜แƒกแƒฎแƒ˜, แƒฐแƒ˜แƒ“แƒ แƒแƒšแƒ˜แƒ–แƒ˜แƒก แƒฎแƒแƒ แƒ˜แƒกแƒฎแƒ˜)

แƒ แƒ”แƒแƒฅแƒชแƒ˜แƒ˜แƒก แƒ”แƒคแƒ”แƒฅแƒขแƒฃแƒ แƒแƒ‘แƒ แƒฉแƒ•แƒ”แƒฃแƒšแƒ”แƒ‘แƒ แƒ˜แƒ• แƒคแƒแƒกแƒ“แƒ”แƒ‘แƒ แƒ แƒ”แƒแƒฅแƒชแƒ˜แƒ˜แƒก แƒžแƒ แƒแƒ“แƒฃแƒฅแƒขแƒ˜แƒก แƒ›แƒแƒกแƒแƒ•แƒšแƒ˜แƒแƒœแƒแƒ‘แƒ˜แƒก แƒ’แƒแƒ›แƒแƒ—แƒ•แƒšแƒ˜แƒ— (แƒœแƒแƒฌแƒ˜แƒšแƒ˜ 5.11). แƒ—แƒฃแƒ›แƒชแƒ, แƒ—แƒฅแƒ•แƒ”แƒœ แƒแƒกแƒ”แƒ•แƒ” แƒจแƒ”แƒ’แƒ˜แƒซแƒšแƒ˜แƒแƒ— แƒจแƒ”แƒแƒคแƒแƒกแƒแƒ— แƒ แƒ”แƒแƒฅแƒชแƒ˜แƒ˜แƒก แƒ”แƒคแƒ”แƒฅแƒขแƒฃแƒ แƒแƒ‘แƒ แƒ˜แƒ›แƒ˜แƒก แƒ“แƒแƒ“แƒ’แƒ”แƒœแƒ˜แƒ—, แƒ—แƒฃ แƒ แƒแƒ›แƒ”แƒšแƒ˜ แƒœแƒแƒฌแƒ˜แƒšแƒ˜ แƒ’แƒแƒ“แƒแƒ˜แƒฅแƒชแƒ แƒงแƒ•แƒ”แƒšแƒแƒ–แƒ” แƒ›แƒœแƒ˜แƒจแƒ•แƒœแƒ”แƒšแƒแƒ•แƒแƒœแƒ˜ (แƒฉแƒ•แƒ”แƒฃแƒšแƒ”แƒ‘แƒ แƒ˜แƒ•, แƒงแƒ•แƒ”แƒšแƒแƒ–แƒ” แƒซแƒ•แƒ˜แƒ แƒ˜) แƒœแƒ˜แƒ•แƒ—แƒ˜แƒ”แƒ แƒ”แƒ‘แƒ˜แƒก แƒกแƒแƒ›แƒ˜แƒ–แƒœแƒ” แƒ แƒ”แƒแƒฅแƒชแƒ˜แƒ˜แƒก แƒžแƒ แƒแƒ“แƒฃแƒฅแƒขแƒแƒ“, แƒ›แƒแƒ’แƒแƒšแƒ˜แƒ—แƒแƒ“, SO 2-แƒ˜แƒก แƒ แƒแƒ›แƒ”แƒšแƒ˜ แƒœแƒแƒฌแƒ˜แƒšแƒ˜ แƒ’แƒแƒ“แƒแƒ˜แƒฅแƒชแƒ SO 3-แƒแƒ“ แƒ’แƒแƒ’แƒ˜แƒ แƒ“แƒ˜แƒก แƒฌแƒแƒ แƒ›แƒแƒ”แƒ‘แƒ˜แƒก แƒ“แƒ แƒแƒก. แƒ›แƒŸแƒแƒ•แƒ, แƒแƒœแƒฃ แƒ˜แƒžแƒแƒ•แƒ” แƒ™แƒแƒœแƒ•แƒ”แƒ แƒขแƒแƒชแƒ˜แƒ˜แƒก แƒฎแƒแƒ แƒ˜แƒกแƒฎแƒ˜แƒแƒ แƒ˜แƒ’แƒ˜แƒœแƒแƒšแƒฃแƒ แƒ˜ แƒœแƒ˜แƒ•แƒ—แƒ˜แƒ”แƒ แƒ”แƒ‘แƒ.

Cl 2 + 2KOH \u003d KCl + KClO + H 2 O

แƒฅแƒšแƒแƒ แƒ˜ (แƒ แƒ”แƒแƒ’แƒ”แƒœแƒขแƒ˜) แƒ—แƒแƒœแƒแƒ‘แƒ แƒแƒ“ แƒ’แƒแƒ แƒ“แƒแƒ˜แƒฅแƒ›แƒœแƒ”แƒ‘แƒ แƒ™แƒแƒšแƒ˜แƒฃแƒ›แƒ˜แƒก แƒฅแƒšแƒแƒ แƒ˜แƒ“แƒแƒ“ แƒ“แƒ แƒ™แƒแƒšแƒ˜แƒฃแƒ›แƒ˜แƒก แƒฐแƒ˜แƒžแƒแƒฅแƒšแƒแƒ แƒ˜แƒขแƒแƒ“. แƒแƒ› แƒ แƒ”แƒแƒฅแƒชแƒ˜แƒแƒจแƒ˜, แƒ—แƒฃแƒœแƒ“แƒแƒช KClO-แƒก 100%-แƒ˜แƒแƒœแƒ˜ แƒ’แƒแƒ›แƒแƒกแƒแƒ•แƒšแƒ˜แƒแƒœแƒแƒ‘แƒ˜แƒ—, แƒฅแƒšแƒแƒ แƒ˜แƒก แƒ›แƒแƒกแƒจแƒ˜ แƒ’แƒแƒ“แƒแƒฅแƒชแƒ”แƒ•แƒ˜แƒก แƒฎแƒแƒ แƒ˜แƒกแƒฎแƒ˜ 50%-แƒ˜แƒ.

แƒ—แƒฅแƒ•แƒ”แƒœแƒ—แƒ•แƒ˜แƒก แƒชแƒœแƒแƒ‘แƒ˜แƒšแƒ˜ แƒ แƒแƒแƒ“แƒ”แƒœแƒแƒ‘แƒ - แƒžแƒ แƒแƒขแƒแƒšแƒ˜แƒ–แƒ˜แƒก แƒฎแƒแƒ แƒ˜แƒกแƒฎแƒ˜ (แƒžแƒฃแƒœแƒฅแƒขแƒ˜ 12.4) - แƒแƒ แƒ˜แƒก แƒ™แƒแƒœแƒ•แƒ”แƒ แƒขแƒแƒชแƒ˜แƒ˜แƒก แƒฎแƒแƒ แƒ˜แƒกแƒฎแƒ˜แƒก แƒ’แƒแƒœแƒกแƒแƒ™แƒฃแƒ—แƒ แƒ”แƒ‘แƒฃแƒšแƒ˜ แƒจแƒ”แƒ›แƒ—แƒฎแƒ•แƒ”แƒ•แƒ:

TED-แƒ˜แƒก แƒคแƒแƒ แƒ’แƒšแƒ”แƒ‘แƒจแƒ˜ แƒ›แƒกแƒ’แƒแƒ•แƒกแƒ˜ แƒ แƒแƒแƒ“แƒ”แƒœแƒแƒ‘แƒ˜แƒ— แƒ”แƒซแƒแƒฎแƒ˜แƒแƒœ แƒ“แƒ˜แƒกแƒแƒชแƒ˜แƒแƒชแƒ˜แƒ˜แƒก แƒฎแƒแƒ แƒ˜แƒกแƒฎแƒ˜แƒ›แƒŸแƒแƒ•แƒ”แƒ‘แƒ˜ แƒแƒœ แƒคแƒฃแƒซแƒ”แƒ”แƒ‘แƒ˜ (แƒแƒกแƒ”แƒ•แƒ” แƒ›แƒแƒฎแƒกแƒ”แƒœแƒ˜แƒ”แƒ‘แƒฃแƒšแƒ˜, แƒ แƒแƒ’แƒแƒ แƒช แƒžแƒ แƒแƒขแƒแƒšแƒ˜แƒ–แƒ˜แƒก แƒฎแƒแƒ แƒ˜แƒกแƒฎแƒ˜). แƒ“แƒ˜แƒกแƒแƒชแƒ˜แƒแƒชแƒ˜แƒ˜แƒก แƒฎแƒแƒ แƒ˜แƒกแƒฎแƒ˜ แƒ“แƒแƒ™แƒแƒ•แƒจแƒ˜แƒ แƒ”แƒ‘แƒฃแƒšแƒ˜แƒ แƒ“แƒ˜แƒกแƒแƒชแƒ˜แƒแƒชแƒ˜แƒ˜แƒก แƒ›แƒฃแƒ“แƒ›แƒ˜แƒ•แƒ—แƒแƒœ แƒแƒกแƒขแƒ•แƒแƒšแƒ“แƒ˜แƒก แƒ’แƒแƒœแƒ–แƒแƒ•แƒ”แƒ‘แƒ˜แƒก แƒ™แƒแƒœแƒแƒœแƒ˜แƒก แƒ›แƒ˜แƒฎแƒ”แƒ“แƒ•แƒ˜แƒ—.

แƒแƒ›แƒแƒ•แƒ” แƒ—แƒ”แƒแƒ แƒ˜แƒ˜แƒก แƒคแƒแƒ แƒ’แƒšแƒ”แƒ‘แƒจแƒ˜ แƒฐแƒ˜แƒ“แƒ แƒแƒšแƒ˜แƒ–แƒ˜แƒก แƒฌแƒแƒœแƒแƒกแƒฌแƒแƒ แƒแƒ‘แƒ แƒฎแƒแƒกแƒ˜แƒแƒ—แƒ“แƒ”แƒ‘แƒ แƒฐแƒ˜แƒ“แƒ แƒแƒšแƒ˜แƒ–แƒ˜แƒก แƒฎแƒแƒ แƒ˜แƒกแƒฎแƒ˜ (แƒ—), แƒจแƒ”แƒ›แƒ“แƒ”แƒ’แƒ˜ แƒ’แƒแƒ›แƒแƒœแƒแƒ—แƒฅแƒ•แƒแƒ›แƒ”แƒ‘แƒ˜แƒก แƒ’แƒแƒ›แƒแƒงแƒ”แƒœแƒ”แƒ‘แƒ˜แƒกแƒแƒก, แƒ แƒแƒ›แƒšแƒ”แƒ‘แƒ˜แƒช แƒ›แƒแƒก แƒฃแƒ™แƒแƒ•แƒจแƒ˜แƒ แƒ”แƒ‘แƒ”แƒœ แƒœแƒ˜แƒ•แƒ—แƒ˜แƒ”แƒ แƒ”แƒ‘แƒ˜แƒก แƒกแƒแƒฌแƒงแƒ˜แƒก แƒ™แƒแƒœแƒชแƒ”แƒœแƒขแƒ แƒแƒชแƒ˜แƒแƒก ( แƒ—แƒแƒœ) แƒ“แƒ แƒฐแƒ˜แƒ“แƒ แƒแƒšแƒ˜แƒ–แƒ˜แƒก แƒ“แƒ แƒแƒก แƒฌแƒแƒ แƒ›แƒแƒฅแƒ›แƒœแƒ˜แƒšแƒ˜ แƒกแƒฃแƒกแƒขแƒ˜ แƒ›แƒŸแƒแƒ•แƒ”แƒ‘แƒ˜แƒก (K HA) แƒ“แƒ แƒกแƒฃแƒกแƒขแƒ˜ แƒคแƒฃแƒซแƒ”แƒ”แƒ‘แƒ˜แƒก แƒ“แƒ˜แƒกแƒแƒชแƒ˜แƒแƒชแƒ˜แƒ˜แƒก แƒ›แƒฃแƒ“แƒ›แƒ˜แƒ•แƒ”แƒ‘แƒ˜ ( แƒ™ MOH):

แƒžแƒ˜แƒ แƒ•แƒ”แƒšแƒ˜ แƒ’แƒแƒ›แƒแƒ—แƒฅแƒ›แƒ แƒ›แƒแƒฅแƒ›แƒ”แƒ“แƒ”แƒ‘แƒก แƒกแƒฃแƒกแƒขแƒ˜ แƒ›แƒŸแƒแƒ•แƒแƒก แƒ›แƒแƒ แƒ˜แƒšแƒ˜แƒก แƒฐแƒ˜แƒ“แƒ แƒแƒšแƒ˜แƒ–แƒ˜แƒกแƒ—แƒ•แƒ˜แƒก, แƒ›แƒ”แƒแƒ แƒ” แƒกแƒฃแƒกแƒขแƒ˜ แƒคแƒฃแƒซแƒ˜แƒก แƒ›แƒแƒ แƒ˜แƒšแƒ˜แƒกแƒ—แƒ•แƒ˜แƒก แƒ“แƒ แƒ›แƒ”แƒกแƒแƒ›แƒ” แƒกแƒฃแƒกแƒขแƒ˜ แƒ›แƒŸแƒแƒ•แƒ˜แƒกแƒ แƒ“แƒ แƒกแƒฃแƒกแƒขแƒ˜ แƒคแƒฃแƒซแƒ˜แƒก แƒ›แƒแƒ แƒ˜แƒšแƒ˜แƒกแƒ—แƒ•แƒ˜แƒก. แƒงแƒ•แƒ”แƒšแƒ แƒ”แƒก แƒ’แƒแƒ›แƒแƒœแƒแƒ—แƒฅแƒ•แƒแƒ›แƒ˜ แƒจแƒ”แƒ˜แƒซแƒšแƒ”แƒ‘แƒ แƒ’แƒแƒ›แƒแƒงแƒ”แƒœแƒ”แƒ‘แƒฃแƒš แƒ˜แƒฅแƒœแƒแƒก แƒ›แƒฎแƒแƒšแƒแƒ“ แƒ’แƒแƒœแƒ–แƒแƒ•แƒ”แƒ‘แƒฃแƒšแƒ˜ แƒฎแƒกแƒœแƒแƒ แƒ”แƒ‘แƒ˜แƒกแƒ—แƒ•แƒ˜แƒก, แƒฐแƒ˜แƒ“แƒ แƒแƒšแƒ˜แƒ–แƒ˜แƒก แƒฎแƒแƒ แƒ˜แƒกแƒฎแƒ˜แƒ— แƒแƒ แƒแƒฃแƒ›แƒ”แƒขแƒ”แƒก 0,05 (5%).

แƒ›แƒแƒกแƒ˜แƒก แƒ›แƒแƒฅแƒ›แƒ”แƒ“แƒ”แƒ‘แƒ˜แƒก แƒ™แƒแƒœแƒแƒœแƒ˜, แƒฐแƒแƒ›แƒแƒคแƒแƒ–แƒฃแƒ แƒ˜ แƒ แƒ”แƒแƒฅแƒชแƒ˜แƒ”แƒ‘แƒ˜, แƒฐแƒ”แƒขแƒ”แƒ แƒแƒคแƒแƒ–แƒฃแƒ แƒ˜ แƒ แƒ”แƒแƒฅแƒชแƒ˜แƒ”แƒ‘แƒ˜, แƒ›แƒงแƒแƒ แƒ˜ แƒคแƒแƒ–แƒ˜แƒก แƒ แƒ”แƒแƒฅแƒชแƒ˜แƒ”แƒ‘แƒ˜, แƒแƒ•แƒขแƒแƒžแƒ แƒแƒขแƒแƒšแƒ˜แƒ–แƒ˜แƒก แƒ›แƒฃแƒ“แƒ›แƒ˜แƒ•แƒ˜ (แƒ˜แƒแƒœแƒฃแƒ แƒ˜ แƒžแƒ แƒแƒ“แƒฃแƒฅแƒขแƒ˜), แƒ“แƒ˜แƒกแƒแƒชแƒ˜แƒแƒชแƒ˜แƒ˜แƒก (แƒ˜แƒแƒœแƒ˜แƒ–แƒแƒชแƒ˜แƒ˜แƒก) แƒ›แƒฃแƒ“แƒ›แƒ˜แƒ•แƒ˜, แƒ“แƒ˜แƒกแƒแƒชแƒ˜แƒแƒชแƒ˜แƒ˜แƒก แƒฎแƒแƒ แƒ˜แƒกแƒฎแƒ˜ (แƒ˜แƒแƒœแƒ˜แƒ–แƒแƒชแƒ˜แƒ), แƒฌแƒงแƒแƒšแƒ‘แƒแƒ“แƒ˜แƒก แƒ˜แƒœแƒ“แƒ”แƒฅแƒกแƒ˜, แƒฐแƒ˜แƒ“แƒ แƒแƒฅแƒกแƒ˜แƒ“แƒ˜แƒก แƒ˜แƒœแƒ“แƒ”แƒฅแƒกแƒ˜, แƒฐแƒ˜แƒ“แƒ แƒแƒšแƒ˜แƒ–แƒ˜แƒก แƒ›แƒฃแƒ“แƒ›แƒ˜แƒ•แƒ˜, แƒฎแƒกแƒœแƒแƒ“แƒแƒ‘แƒ˜แƒก แƒ›แƒฃแƒ“แƒ›แƒ˜แƒ•แƒ˜ (แƒฎแƒกแƒœแƒแƒ“แƒแƒ‘แƒ˜แƒก แƒžแƒ แƒแƒ“แƒฃแƒฅแƒขแƒ˜) , แƒ™แƒแƒœแƒ•แƒ”แƒ แƒขแƒแƒชแƒ˜แƒ˜แƒก แƒฎแƒแƒ แƒ˜แƒกแƒฎแƒ˜ .

  1. แƒฉแƒแƒ›แƒแƒ—แƒ•แƒแƒšแƒ”แƒ— แƒคแƒแƒฅแƒขแƒแƒ แƒ”แƒ‘แƒ˜, แƒ แƒแƒ›แƒšแƒ”แƒ‘แƒ˜แƒช แƒชแƒ•แƒšแƒ˜แƒก แƒฅแƒ˜แƒ›แƒ˜แƒฃแƒ  แƒฌแƒแƒœแƒแƒกแƒฌแƒแƒ แƒแƒ‘แƒแƒก แƒ“แƒ แƒชแƒ•แƒšแƒ˜แƒก แƒฌแƒแƒœแƒแƒกแƒฌแƒแƒ แƒแƒ‘แƒ˜แƒก แƒ›แƒฃแƒ“แƒ›แƒ˜แƒ•แƒแƒก.
  2. แƒ แƒ แƒคแƒแƒฅแƒขแƒแƒ แƒ”แƒ‘แƒ˜ แƒ˜แƒซแƒšแƒ”แƒ•แƒ แƒฅแƒ˜แƒ›แƒ˜แƒฃแƒ แƒ˜ แƒฌแƒแƒœแƒแƒกแƒฌแƒแƒ แƒแƒ‘แƒ˜แƒก แƒ’แƒแƒ“แƒแƒขแƒแƒœแƒแƒก แƒฌแƒแƒœแƒแƒกแƒฌแƒแƒ แƒแƒ‘แƒ˜แƒก แƒ›แƒฃแƒ“แƒ›แƒ˜แƒ•แƒ˜แƒก แƒจแƒ”แƒชแƒ•แƒšแƒ˜แƒก แƒ’แƒแƒ แƒ”แƒจแƒ”?
  3. 1 แƒšแƒ˜แƒขแƒ แƒจแƒ˜ แƒแƒฃแƒชแƒ˜แƒšแƒ”แƒ‘แƒ”แƒšแƒ˜แƒ 0,5 แƒ›แƒแƒšแƒ˜ NaCl, 0,16 แƒ›แƒแƒšแƒ˜ KCl แƒ“แƒ 0,24 แƒ›แƒแƒšแƒ˜ K 2 SO 4 แƒจแƒ”แƒ›แƒชแƒ•แƒ”แƒšแƒ˜ แƒฎแƒกแƒœแƒแƒ แƒ˜แƒก แƒ›แƒแƒ›แƒ–แƒแƒ“แƒ”แƒ‘แƒ. แƒ แƒแƒ’แƒแƒ  แƒ’แƒแƒ•แƒแƒ™แƒ”แƒ—แƒแƒ— แƒ”แƒก, แƒ แƒแƒชแƒ แƒ—แƒฅแƒ•แƒ”แƒœแƒก แƒ’แƒแƒœแƒ™แƒแƒ แƒ’แƒฃแƒšแƒ”แƒ‘แƒแƒจแƒ˜แƒ แƒ›แƒฎแƒแƒšแƒแƒ“ แƒœแƒแƒขแƒ แƒ˜แƒฃแƒ›แƒ˜แƒก แƒฅแƒšแƒแƒ แƒ˜แƒ“แƒ˜, แƒ™แƒแƒšแƒ˜แƒฃแƒ›แƒ˜แƒก แƒฅแƒšแƒแƒ แƒ˜แƒ“แƒ˜ แƒ“แƒ แƒœแƒแƒขแƒ แƒ˜แƒฃแƒ›แƒ˜แƒก แƒกแƒฃแƒšแƒคแƒแƒขแƒ˜?
  4. แƒ’แƒแƒœแƒกแƒแƒ–แƒฆแƒ•แƒ แƒ”แƒ— แƒซแƒ›แƒแƒ แƒ›แƒŸแƒแƒ•แƒแƒก, แƒฐแƒ˜แƒ“แƒ แƒแƒชแƒ˜แƒแƒœแƒฃแƒ แƒ˜ แƒ“แƒ แƒแƒ–แƒแƒขแƒ˜แƒก แƒ›แƒŸแƒแƒ•แƒ”แƒ‘แƒ˜แƒก แƒžแƒ แƒแƒขแƒแƒšแƒ˜แƒ–แƒ˜แƒก แƒฎแƒแƒ แƒ˜แƒกแƒฎแƒ˜ แƒ“แƒ”แƒชแƒ˜แƒ›แƒแƒšแƒแƒ แƒฃแƒš, แƒชแƒ”แƒœแƒขแƒ›แƒแƒšแƒแƒ แƒฃแƒš แƒ“แƒ แƒ›แƒ˜แƒšแƒ˜แƒ›แƒแƒšแƒแƒ แƒฃแƒš แƒฎแƒกแƒœแƒแƒ แƒ”แƒ‘แƒจแƒ˜.
  5. แƒ‘แƒฃแƒขแƒ˜แƒ แƒ˜แƒก แƒ›แƒŸแƒแƒ•แƒแƒก แƒžแƒ แƒแƒขแƒแƒšแƒ˜แƒ–แƒ˜แƒก แƒฎแƒแƒ แƒ˜แƒกแƒฎแƒ˜ 0,2 M แƒฎแƒกแƒœแƒแƒ แƒจแƒ˜ แƒแƒ แƒ˜แƒก 0,866%. แƒ’แƒแƒœแƒกแƒแƒ–แƒฆแƒ•แƒ แƒ”แƒ— แƒแƒ› แƒœแƒ˜แƒ•แƒ—แƒ˜แƒ”แƒ แƒ”แƒ‘แƒ˜แƒก แƒ›แƒŸแƒแƒ•แƒ˜แƒแƒœแƒแƒ‘แƒ˜แƒก แƒ›แƒฃแƒ“แƒ›แƒ˜แƒ•แƒ˜.
  6. แƒฎแƒกแƒœแƒแƒ แƒ˜แƒก แƒ แƒแƒ›แƒ”แƒš แƒ™แƒแƒœแƒชแƒ”แƒœแƒขแƒ แƒแƒชแƒ˜แƒแƒจแƒ˜ แƒ˜แƒฅแƒœแƒ”แƒ‘แƒ แƒแƒ–แƒแƒขแƒ˜แƒก แƒ›แƒŸแƒแƒ•แƒแƒก แƒžแƒ แƒแƒขแƒแƒšแƒ˜แƒ–แƒ˜แƒก แƒฎแƒแƒ แƒ˜แƒกแƒฎแƒ˜ 0,2?
  7. แƒ แƒแƒ›แƒ“แƒ”แƒœแƒ˜ แƒฌแƒงแƒแƒšแƒ˜ แƒฃแƒœแƒ“แƒ แƒ“แƒแƒ”แƒ›แƒแƒขแƒแƒก 300 แƒ›แƒš 0,2 แƒ› แƒซแƒ›แƒแƒ แƒ›แƒŸแƒแƒ•แƒแƒก แƒฎแƒกแƒœแƒแƒ แƒก, แƒ แƒแƒ› แƒ’แƒแƒแƒ แƒ›แƒแƒ’แƒ“แƒ”แƒก แƒ›แƒŸแƒแƒ•แƒ แƒžแƒ แƒแƒขแƒแƒšแƒ˜แƒ–แƒ˜แƒก แƒฎแƒแƒ แƒ˜แƒกแƒฎแƒ˜?
  8. แƒ’แƒแƒœแƒกแƒแƒ–แƒฆแƒ•แƒ แƒ”แƒ— แƒฐแƒ˜แƒžแƒแƒฅแƒšแƒแƒ แƒ›แƒŸแƒแƒ•แƒแƒก แƒžแƒ แƒแƒขแƒแƒšแƒ˜แƒ–แƒ˜แƒก แƒฎแƒแƒ แƒ˜แƒกแƒฎแƒ˜, แƒ—แƒฃ แƒ›แƒ˜แƒก แƒฎแƒกแƒœแƒแƒ แƒจแƒ˜ pH = 6. แƒ แƒแƒ’แƒแƒ แƒ˜แƒ แƒ›แƒŸแƒแƒ•แƒแƒก แƒ™แƒแƒœแƒชแƒ”แƒœแƒขแƒ แƒแƒชแƒ˜แƒ แƒแƒ› แƒฎแƒกแƒœแƒแƒ แƒจแƒ˜?
  9. แƒฎแƒกแƒœแƒแƒ แƒ˜แƒก pH แƒแƒ แƒ˜แƒก 3. แƒ แƒแƒ’แƒแƒ แƒ˜ แƒฃแƒœแƒ“แƒ แƒ˜แƒงแƒแƒก แƒแƒ›แƒ˜แƒกแƒแƒ—แƒ•แƒ˜แƒก แƒ) แƒแƒ–แƒแƒขแƒ˜แƒก, แƒ‘) แƒซแƒ›แƒแƒ แƒ›แƒŸแƒแƒ•แƒแƒก แƒ™แƒแƒœแƒชแƒ”แƒœแƒขแƒ แƒแƒชแƒ˜แƒ?
  10. แƒ แƒแƒ’แƒแƒ  แƒฃแƒœแƒ“แƒ แƒจแƒ”แƒ˜แƒชแƒ•แƒแƒšแƒแƒก แƒฎแƒกแƒœแƒแƒ แƒจแƒ˜ แƒ) แƒแƒฅแƒกแƒแƒœแƒ˜แƒฃแƒ›แƒ˜แƒก แƒ˜แƒแƒœแƒ”แƒ‘แƒ˜แƒก, แƒ‘) แƒฐแƒ˜แƒ“แƒ แƒแƒฅแƒกแƒ˜แƒ“แƒ˜แƒก แƒ˜แƒแƒœแƒ”แƒ‘แƒ˜แƒก แƒ™แƒแƒœแƒชแƒ”แƒœแƒขแƒ แƒแƒชแƒ˜แƒ แƒ˜แƒกแƒ”, แƒ แƒแƒ› แƒฎแƒกแƒœแƒแƒ แƒ˜แƒก pH แƒ’แƒแƒ˜แƒ–แƒแƒ แƒ“แƒแƒก แƒ”แƒ แƒ—แƒ˜แƒ—?
  11. แƒ แƒแƒ›แƒ“แƒ”แƒœ แƒแƒฅแƒกแƒแƒœแƒ˜แƒฃแƒ›แƒ˜แƒก แƒ˜แƒแƒœแƒก แƒจแƒ”แƒ˜แƒชแƒแƒ•แƒก 1 แƒ›แƒš แƒฎแƒกแƒœแƒแƒ แƒ˜ pH = 12-แƒ–แƒ”?
  12. แƒ แƒแƒ’แƒแƒ  แƒจแƒ”แƒ˜แƒชแƒ•แƒšแƒ”แƒ‘แƒ แƒฌแƒงแƒšแƒ˜แƒก pH, แƒ—แƒฃ แƒ›แƒแƒก 10 แƒšแƒ˜แƒขแƒ แƒก แƒ“แƒแƒ”แƒ›แƒแƒขแƒ”แƒ‘แƒ 0,4 แƒ’ NaOH?
  13. แƒ’แƒแƒ›แƒแƒ—แƒ•แƒแƒšแƒ”แƒ— แƒแƒฅแƒกแƒแƒœแƒ˜แƒฃแƒ›แƒ˜แƒก แƒ˜แƒแƒœแƒ”แƒ‘แƒ˜แƒก แƒ“แƒ แƒฐแƒ˜แƒ“แƒ แƒแƒฅแƒกแƒ˜แƒ“แƒ˜แƒก แƒ˜แƒแƒœแƒ”แƒ‘แƒ˜แƒก แƒ™แƒแƒœแƒชแƒ”แƒœแƒขแƒ แƒแƒชแƒ˜แƒ”แƒ‘แƒ˜, แƒแƒ’แƒ แƒ”แƒ—แƒ•แƒ” แƒฌแƒงแƒแƒšแƒ‘แƒแƒ“แƒ˜แƒก แƒ“แƒ แƒฐแƒ˜แƒ“แƒ แƒแƒฅแƒกแƒ˜แƒ“แƒ˜แƒก แƒ˜แƒœแƒ“แƒ”แƒฅแƒกแƒ”แƒ‘แƒ˜แƒก แƒ›แƒœแƒ˜แƒจแƒ•แƒœแƒ”แƒšแƒแƒ‘แƒ”แƒ‘แƒ˜ แƒจแƒ”แƒ›แƒ“แƒ”แƒ’ แƒฌแƒงแƒแƒšแƒฎแƒกแƒœแƒแƒ แƒ”แƒ‘แƒจแƒ˜: แƒ) 0,01 แƒ› HCl แƒฎแƒกแƒœแƒแƒ แƒ˜; แƒ‘) CH 3 COOH 0,01 แƒ› แƒฎแƒกแƒœแƒแƒ แƒ˜; แƒ’) 0,001 M NaOH แƒฎแƒกแƒœแƒแƒ แƒ˜; แƒ“) 0,001 แƒ› NH 3 แƒฎแƒกแƒœแƒแƒ แƒ˜.
  14. แƒ“แƒแƒœแƒแƒ แƒ—แƒจแƒ˜ แƒ›แƒแƒชแƒ”แƒ›แƒฃแƒšแƒ˜ แƒฎแƒกแƒœแƒแƒ“แƒแƒ‘แƒ˜แƒก แƒžแƒ แƒแƒ“แƒฃแƒฅแƒขแƒ”แƒ‘แƒ˜แƒก แƒ›แƒœแƒ˜แƒจแƒ•แƒœแƒ”แƒšแƒแƒ‘แƒ”แƒ‘แƒ˜แƒก แƒ’แƒแƒ›แƒแƒงแƒ”แƒœแƒ”แƒ‘แƒ˜แƒ— แƒ’แƒแƒœแƒ•แƒกแƒแƒ–แƒฆแƒ•แƒ แƒแƒ— แƒฎแƒกแƒœแƒแƒ แƒ˜แƒก แƒ™แƒแƒœแƒชแƒ”แƒœแƒขแƒ แƒแƒชแƒ˜แƒ แƒ“แƒ แƒ›แƒแƒกแƒฃแƒ แƒ˜ แƒฌแƒ˜แƒšแƒ˜ แƒ) แƒ•แƒ”แƒ แƒชแƒฎแƒšแƒ˜แƒก แƒฅแƒšแƒแƒ แƒ˜แƒ“แƒ˜แƒก, แƒ‘) แƒ™แƒแƒšแƒชแƒ˜แƒฃแƒ›แƒ˜แƒก แƒกแƒฃแƒšแƒคแƒแƒขแƒ˜แƒก, แƒ’) แƒแƒšแƒฃแƒ›แƒ˜แƒœแƒ˜แƒก แƒคแƒแƒกแƒคแƒแƒขแƒ˜แƒก แƒฎแƒกแƒœแƒแƒ แƒจแƒ˜.
  15. แƒ’แƒแƒœแƒกแƒแƒ–แƒฆแƒ•แƒ แƒ”แƒ— แƒฌแƒงแƒšแƒ˜แƒก แƒ›แƒแƒชแƒฃแƒšแƒแƒ‘แƒ, แƒ แƒแƒ›แƒ”แƒšแƒ˜แƒช แƒกแƒแƒญแƒ˜แƒ แƒแƒ แƒ‘แƒแƒ แƒ˜แƒฃแƒ›แƒ˜แƒก แƒกแƒฃแƒšแƒคแƒแƒขแƒ˜แƒก แƒ“แƒแƒกแƒแƒจแƒšแƒ”แƒšแƒแƒ“ 1 แƒ’ แƒ›แƒแƒกแƒ˜แƒ— 25 o C แƒขแƒ”แƒ›แƒžแƒ”แƒ แƒแƒขแƒฃแƒ แƒแƒ–แƒ”.
  16. แƒ แƒแƒ›แƒ“แƒ”แƒœแƒ˜แƒ แƒ˜แƒแƒœแƒ”แƒ‘แƒ˜แƒก แƒกแƒแƒฎแƒ˜แƒ— แƒ•แƒ”แƒ แƒชแƒฎแƒšแƒ˜แƒก แƒ›แƒแƒกแƒ 25 o C แƒขแƒ”แƒ›แƒžแƒ”แƒ แƒแƒขแƒฃแƒ แƒแƒ–แƒ” แƒ’แƒแƒฏแƒ”แƒ แƒ”แƒ‘แƒฃแƒš แƒ•แƒ”แƒ แƒชแƒฎแƒšแƒ˜แƒก แƒ‘แƒ แƒแƒ›แƒ˜แƒ“แƒ˜แƒก 1 แƒšแƒ˜แƒขแƒ  แƒฎแƒกแƒœแƒแƒ แƒจแƒ˜?
  17. 25 o C แƒขแƒ”แƒ›แƒžแƒ”แƒ แƒแƒขแƒฃแƒ แƒแƒ–แƒ” แƒ’แƒแƒฏแƒ”แƒ แƒ”แƒ‘แƒฃแƒšแƒ˜ แƒ•แƒ”แƒ แƒชแƒฎแƒšแƒ˜แƒก แƒกแƒฃแƒšแƒคแƒ˜แƒ“แƒ˜แƒก แƒฎแƒกแƒœแƒแƒ แƒ˜แƒก แƒ แƒ แƒ›แƒแƒชแƒฃแƒšแƒแƒ‘แƒ แƒจแƒ”แƒ˜แƒชแƒแƒ•แƒก 1 แƒ›แƒ’ แƒ’แƒแƒฎแƒกแƒœแƒ˜แƒš แƒœแƒ˜แƒ•แƒ—แƒ˜แƒ”แƒ แƒ”แƒ‘แƒแƒก?
  18. แƒฌแƒแƒ แƒ›แƒแƒ˜แƒฅแƒ›แƒœแƒ”แƒ‘แƒ แƒ—แƒฃ แƒแƒ แƒ แƒœแƒแƒšแƒ”แƒฅแƒ˜, แƒ—แƒฃ 0,4 M KCl แƒฎแƒกแƒœแƒแƒ แƒ˜แƒก แƒ—แƒแƒœแƒแƒ‘แƒแƒ แƒ˜ แƒ›แƒแƒชแƒฃแƒšแƒแƒ‘แƒ แƒ“แƒแƒ”แƒ›แƒแƒขแƒ”แƒ‘แƒ 0,05 M Pb(NO 3) 2 แƒฎแƒกแƒœแƒแƒ แƒก?
  19. แƒ“แƒแƒแƒ“แƒ’แƒ˜แƒœแƒ”แƒ—, แƒฌแƒแƒ แƒ›แƒแƒ˜แƒฅแƒ›แƒœแƒ”แƒ‘แƒ แƒ—แƒฃ แƒแƒ แƒ แƒœแƒแƒšแƒ”แƒฅแƒ˜ 5 แƒ›แƒš 0,004 M CdCl 2 แƒฎแƒกแƒœแƒแƒ แƒ˜แƒก แƒ“แƒ 15 แƒ›แƒš 0,003 M KOH แƒฎแƒกแƒœแƒแƒ แƒ˜แƒก แƒฉแƒแƒ›แƒแƒกแƒฎแƒ›แƒ˜แƒก แƒจแƒ”แƒ›แƒ“แƒ”แƒ’.
  20. แƒ—แƒฅแƒ•แƒ”แƒœแƒก แƒ’แƒแƒœแƒ™แƒแƒ แƒ’แƒฃแƒšแƒ”แƒ‘แƒแƒจแƒ˜แƒ แƒจแƒ”แƒ›แƒ“แƒ”แƒ’แƒ˜ แƒœแƒ˜แƒ•แƒ—แƒ˜แƒ”แƒ แƒ”แƒ‘แƒ”แƒ‘แƒ˜: NH 3 , KHS, Fe, Al(OH) 3 , CaO, NaNO 3 , CaCO 3 , N 2 O 5 , LiOH, Na 2 SO 4 . 10H2O, Mg (OH) Cl, Na, Ca (NO 2) 2. 4H2O, ZnO, NaI. 2H2O, CO2, N2, Ba(OH)2. 8H2O, AgNO3. แƒ—แƒ˜แƒ—แƒแƒ”แƒฃแƒš แƒแƒ› แƒœแƒ˜แƒ•แƒ—แƒ˜แƒ”แƒ แƒ”แƒ‘แƒแƒ–แƒ”, แƒชแƒแƒšแƒ™แƒ” แƒ‘แƒแƒ แƒแƒ—แƒ–แƒ”, แƒฃแƒžแƒแƒกแƒฃแƒฎแƒ”แƒ— แƒจแƒ”แƒ›แƒ“แƒ”แƒ’ แƒ™แƒ˜แƒ—แƒฎแƒ•แƒ”แƒ‘แƒก:

1) แƒ แƒแƒ’แƒแƒ แƒ˜แƒ แƒแƒ› แƒœแƒ˜แƒ•แƒ—แƒ˜แƒ”แƒ แƒ”แƒ‘แƒ˜แƒก แƒกแƒขแƒ แƒฃแƒฅแƒขแƒฃแƒ แƒ แƒœแƒแƒ แƒ›แƒแƒšแƒฃแƒ  แƒžแƒ˜แƒ แƒแƒ‘แƒ”แƒ‘แƒจแƒ˜ (แƒ›แƒแƒšแƒ”แƒ™แƒฃแƒšแƒฃแƒ แƒ˜ แƒ—แƒฃ แƒแƒ แƒแƒ›แƒแƒšแƒ”แƒ™แƒฃแƒšแƒฃแƒ แƒ˜)?
2) แƒแƒ’แƒ แƒ”แƒ’แƒแƒชแƒ˜แƒ˜แƒก แƒ แƒ แƒ›แƒ“แƒ’แƒแƒ›แƒแƒ แƒ”แƒแƒ‘แƒแƒจแƒ˜แƒ แƒ”แƒก แƒœแƒ˜แƒ•แƒ—แƒ˜แƒ”แƒ แƒ”แƒ‘แƒ แƒแƒ—แƒแƒฎแƒ˜แƒก แƒขแƒ”แƒ›แƒžแƒ”แƒ แƒแƒขแƒฃแƒ แƒแƒ–แƒ”?
3) แƒ แƒ แƒขแƒ˜แƒžแƒ˜แƒก แƒ™แƒ แƒ˜แƒกแƒขแƒแƒšแƒ”แƒ‘แƒก แƒฌแƒแƒ แƒ›แƒแƒฅแƒ›แƒœแƒ˜แƒก แƒ”แƒก แƒœแƒ˜แƒ•แƒ—แƒ˜แƒ”แƒ แƒ”แƒ‘แƒ?
4) แƒแƒฆแƒฌแƒ”แƒ แƒ”แƒ— แƒฅแƒ˜แƒ›แƒ˜แƒฃแƒ แƒ˜ แƒ‘แƒ›แƒ แƒแƒ› แƒœแƒ˜แƒ•แƒ—แƒ˜แƒ”แƒ แƒ”แƒ‘แƒแƒจแƒ˜.
5) แƒ แƒ แƒ™แƒšแƒแƒกแƒก แƒ›แƒ˜แƒ”แƒ™แƒฃแƒ—แƒ•แƒœแƒ”แƒ‘แƒ แƒ”แƒก แƒœแƒ˜แƒ•แƒ—แƒ˜แƒ”แƒ แƒ”แƒ‘แƒ แƒขแƒ แƒแƒ“แƒ˜แƒชแƒ˜แƒฃแƒšแƒ˜ แƒ™แƒšแƒแƒกแƒ˜แƒคแƒ˜แƒ™แƒแƒชแƒ˜แƒ˜แƒก แƒ›แƒ˜แƒฎแƒ”แƒ“แƒ•แƒ˜แƒ—?
6) แƒ แƒแƒ’แƒแƒ  แƒฃแƒ แƒ—แƒ˜แƒ”แƒ แƒ—แƒฅแƒ›แƒ”แƒ“แƒ”แƒ‘แƒก แƒ”แƒก แƒœแƒ˜แƒ•แƒ—แƒ˜แƒ”แƒ แƒ”แƒ‘แƒ แƒฌแƒงแƒแƒšแƒ—แƒแƒœ? แƒ—แƒฃ แƒ˜แƒก แƒ˜แƒฎแƒกแƒœแƒ”แƒ‘แƒ แƒแƒœ แƒ แƒ”แƒแƒ’แƒ˜แƒ แƒ”แƒ‘แƒก, แƒ›แƒ˜แƒ”แƒชแƒ˜แƒ— แƒฅแƒ˜แƒ›แƒ˜แƒฃแƒ แƒ˜ แƒ’แƒแƒœแƒขแƒแƒšแƒ”แƒ‘แƒ. แƒจแƒ”แƒ’แƒ•แƒ˜แƒซแƒšแƒ˜แƒ แƒแƒ› แƒžแƒ แƒแƒชแƒ”แƒกแƒ˜แƒก แƒจแƒ”แƒ‘แƒ แƒฃแƒœแƒ”แƒ‘แƒ? แƒ—แƒฃ แƒ•แƒแƒ™แƒ”แƒ—แƒ”แƒ‘แƒ—, แƒ›แƒแƒจแƒ˜แƒœ แƒ แƒ แƒžแƒ˜แƒ แƒแƒ‘แƒ”แƒ‘แƒจแƒ˜? แƒ แƒ แƒคแƒ˜แƒ–แƒ˜แƒ™แƒฃแƒ แƒ›แƒ แƒ แƒแƒแƒ“แƒ”แƒœแƒแƒ‘แƒแƒ› แƒจแƒ”แƒ˜แƒซแƒšแƒ”แƒ‘แƒ แƒ“แƒแƒแƒฎแƒแƒกแƒ˜แƒแƒ—แƒแƒก แƒฌแƒแƒœแƒแƒกแƒฌแƒแƒ แƒแƒ‘แƒ˜แƒก แƒ›แƒ“แƒ’แƒแƒ›แƒแƒ แƒ”แƒแƒ‘แƒ แƒแƒ› แƒžแƒ แƒแƒชแƒ”แƒกแƒจแƒ˜? แƒ—แƒฃ แƒœแƒ˜แƒ•แƒ—แƒ˜แƒ”แƒ แƒ”แƒ‘แƒ แƒฎแƒกแƒœแƒแƒ“แƒ˜แƒ, แƒ แƒแƒ’แƒแƒ  แƒจแƒ”แƒ˜แƒซแƒšแƒ”แƒ‘แƒ แƒ’แƒแƒ˜แƒ–แƒแƒ แƒ“แƒแƒก แƒ›แƒ˜แƒกแƒ˜ แƒฎแƒกแƒœแƒแƒ“แƒแƒ‘แƒ?
7) แƒจแƒ”แƒกแƒแƒซแƒšแƒ”แƒ‘แƒ”แƒšแƒ˜แƒ แƒ—แƒฃ แƒแƒ แƒ แƒแƒ› แƒœแƒ˜แƒ•แƒ—แƒ˜แƒ”แƒ แƒ”แƒ‘แƒ˜แƒก แƒ แƒ”แƒแƒฅแƒชแƒ˜แƒ แƒ›แƒแƒ แƒ˜แƒšแƒ›แƒŸแƒแƒ•แƒแƒกแƒ—แƒแƒœ? แƒ—แƒฃ แƒจแƒ”แƒกแƒแƒซแƒšแƒ”แƒ‘แƒ”แƒšแƒ˜แƒ, แƒ แƒ แƒžแƒ˜แƒ แƒแƒ‘แƒ”แƒ‘แƒจแƒ˜? แƒ›แƒ˜แƒ”แƒชแƒ˜แƒ— แƒ แƒ”แƒแƒฅแƒชแƒ˜แƒ˜แƒก แƒ’แƒแƒœแƒขแƒแƒšแƒ”แƒ‘แƒ. แƒ แƒแƒขแƒแƒ› แƒฎแƒ“แƒ”แƒ‘แƒ แƒ”แƒก แƒ แƒ”แƒแƒฅแƒชแƒ˜แƒ? แƒแƒ แƒ˜แƒก แƒ˜แƒก แƒจแƒ”แƒฅแƒชแƒ”แƒ•แƒแƒ“แƒ˜? แƒ—แƒฃ แƒจแƒ”แƒฅแƒชแƒ”แƒ•แƒแƒ“แƒ˜แƒ, แƒ›แƒแƒจแƒ˜แƒœ แƒ แƒ แƒžแƒ˜แƒ แƒแƒ‘แƒ”แƒ‘แƒจแƒ˜? แƒ แƒแƒ’แƒแƒ  แƒ’แƒแƒ•แƒ–แƒแƒ แƒ“แƒแƒ— แƒ›แƒแƒกแƒแƒ•แƒšแƒ˜แƒแƒœแƒแƒ‘แƒ แƒแƒ› แƒ แƒ”แƒแƒฅแƒชแƒ˜แƒแƒจแƒ˜? แƒ แƒ แƒจแƒ”แƒ˜แƒชแƒ•แƒšแƒ”แƒ‘แƒ, แƒ—แƒฃ แƒ›แƒแƒ แƒ˜แƒšแƒ›แƒŸแƒแƒ•แƒแƒก แƒœแƒแƒชแƒ•แƒšแƒแƒ“ แƒ’แƒแƒ›แƒแƒ•แƒ˜แƒงแƒ”แƒœแƒ”แƒ‘แƒ— แƒ›แƒจแƒ แƒแƒš แƒฌแƒงแƒแƒšแƒ‘แƒแƒ“แƒ˜แƒก แƒฅแƒšแƒแƒ แƒ˜แƒ“แƒก? แƒ›แƒ˜แƒ”แƒชแƒ˜แƒ— แƒจแƒ”แƒกแƒแƒ‘แƒแƒ›แƒ˜แƒกแƒ˜ แƒ แƒ”แƒแƒฅแƒชแƒ˜แƒ˜แƒก แƒ’แƒแƒœแƒขแƒแƒšแƒ”แƒ‘แƒ.
8) แƒจแƒ”แƒกแƒแƒซแƒšแƒ”แƒ‘แƒ”แƒšแƒ˜แƒ แƒ—แƒฃ แƒแƒ แƒ แƒแƒ› แƒœแƒ˜แƒ•แƒ—แƒ˜แƒ”แƒ แƒ”แƒ‘แƒ˜แƒก แƒ แƒ”แƒแƒฅแƒชแƒ˜แƒ แƒœแƒแƒขแƒ แƒ˜แƒฃแƒ›แƒ˜แƒก แƒฐแƒ˜แƒ“แƒ แƒแƒฅแƒกแƒ˜แƒ“แƒ˜แƒก แƒฎแƒกแƒœแƒแƒ แƒ˜แƒ—? แƒ—แƒฃ แƒจแƒ”แƒกแƒแƒซแƒšแƒ”แƒ‘แƒ”แƒšแƒ˜แƒ, แƒ แƒ แƒžแƒ˜แƒ แƒแƒ‘แƒ”แƒ‘แƒจแƒ˜? แƒ›แƒ˜แƒ”แƒชแƒ˜แƒ— แƒ แƒ”แƒแƒฅแƒชแƒ˜แƒ˜แƒก แƒ’แƒแƒœแƒขแƒแƒšแƒ”แƒ‘แƒ. แƒ แƒแƒขแƒแƒ› แƒฎแƒ“แƒ”แƒ‘แƒ แƒ”แƒก แƒ แƒ”แƒแƒฅแƒชแƒ˜แƒ? แƒแƒ แƒ˜แƒก แƒ˜แƒก แƒจแƒ”แƒฅแƒชแƒ”แƒ•แƒแƒ“แƒ˜? แƒ—แƒฃ แƒจแƒ”แƒฅแƒชแƒ”แƒ•แƒแƒ“แƒ˜แƒ, แƒ›แƒแƒจแƒ˜แƒœ แƒ แƒ แƒžแƒ˜แƒ แƒแƒ‘แƒ”แƒ‘แƒจแƒ˜? แƒ แƒแƒ’แƒแƒ  แƒ’แƒแƒ•แƒ–แƒแƒ แƒ“แƒแƒ— แƒ›แƒแƒกแƒแƒ•แƒšแƒ˜แƒแƒœแƒแƒ‘แƒ แƒแƒ› แƒ แƒ”แƒแƒฅแƒชแƒ˜แƒแƒจแƒ˜? แƒ แƒ แƒจแƒ”แƒ˜แƒชแƒ•แƒšแƒ”แƒ‘แƒ, แƒ—แƒฃ แƒœแƒแƒขแƒ แƒ˜แƒฃแƒ›แƒ˜แƒก แƒฐแƒ˜แƒ“แƒ แƒแƒฅแƒกแƒ˜แƒ“แƒ˜แƒก แƒฎแƒกแƒœแƒแƒ แƒ˜แƒก แƒœแƒแƒชแƒ•แƒšแƒแƒ“ แƒ’แƒแƒ›แƒแƒงแƒ”แƒœแƒ”แƒ‘แƒฃแƒšแƒ˜ แƒ˜แƒฅแƒœแƒ”แƒ‘แƒ แƒ›แƒจแƒ แƒแƒšแƒ˜ NaOH? แƒ›แƒ˜แƒ”แƒชแƒ˜แƒ— แƒจแƒ”แƒกแƒแƒ‘แƒแƒ›แƒ˜แƒกแƒ˜ แƒ แƒ”แƒแƒฅแƒชแƒ˜แƒ˜แƒก แƒ’แƒแƒœแƒขแƒแƒšแƒ”แƒ‘แƒ.
9) แƒ›แƒ˜แƒ”แƒชแƒ˜แƒ— แƒ—แƒฅแƒ•แƒ”แƒœแƒ—แƒ•แƒ˜แƒก แƒชแƒœแƒแƒ‘แƒ˜แƒšแƒ˜ แƒงแƒ•แƒ”แƒšแƒ แƒ›แƒ”แƒ—แƒแƒ“แƒ˜ แƒแƒ› แƒœแƒ˜แƒ•แƒ—แƒ˜แƒ”แƒ แƒ”แƒ‘แƒ˜แƒก แƒ›แƒ˜แƒกแƒแƒฆแƒ”แƒ‘แƒแƒ“.
10) แƒ›แƒ˜แƒ”แƒชแƒ˜แƒ— แƒแƒ› แƒœแƒ˜แƒ•แƒ—แƒ˜แƒ”แƒ แƒ”แƒ‘แƒ˜แƒก แƒ—แƒฅแƒ•แƒ”แƒœแƒ—แƒ•แƒ˜แƒก แƒชแƒœแƒแƒ‘แƒ˜แƒšแƒ˜ แƒงแƒ•แƒ”แƒšแƒ แƒ“แƒแƒกแƒแƒฎแƒ”แƒšแƒ”แƒ‘แƒ.
แƒแƒ› แƒ™แƒ˜แƒ—แƒฎแƒ•แƒ”แƒ‘แƒ–แƒ” แƒžแƒแƒกแƒฃแƒฎแƒ˜แƒก แƒ’แƒแƒชแƒ”แƒ›แƒ˜แƒกแƒแƒก แƒจแƒ”แƒ’แƒ˜แƒซแƒšแƒ˜แƒแƒ— แƒ’แƒแƒ›แƒแƒ˜แƒงแƒ”แƒœแƒแƒ— แƒœแƒ”แƒ‘แƒ˜แƒกแƒ›แƒ˜แƒ”แƒ แƒ˜ แƒกแƒแƒชแƒœแƒแƒ‘แƒแƒ แƒ แƒšแƒ˜แƒขแƒ”แƒ แƒแƒขแƒฃแƒ แƒ.

  • แƒกแƒแƒ˜แƒขแƒ˜แƒก แƒกแƒ”แƒฅแƒชแƒ˜แƒ”แƒ‘แƒ˜